USMLE Step 3: Part IV

Ace your homework & exams now with Quizwiz!

40 YO M smoker with hx of asthma is admitted to the inpatient service with fever, cough, brownish-green sputum, and malaise. PE shows RR 15, no use of accessory ms, and b/l polyphonic wheezes throughout the lung fields. There is no clubbing or skin lesions. You consider a dx of allergic bronchopulmonary aspergillosis. All of the following clinical features are c/w allergic bronchopulmonary aspergillosis, except: A: b/l, peripheral cavitary lung infiltrates B: elevated IgE C: peripheral eosinophilia D: positive serum ABS to Aspergillus E: positive skin testing for Aspergillus

A Aspergillus has many clinical manifestations. Invasive aspergillosis typically occurs in immunocompromised patients and presents as rapidly progressive pulmonary infiltrates. Infection progresses by direct extension across tissue planes. Cavitation may occur. Allergic bronchopulmonary aspergillosis (ABPA) is a different clinical entity. It often occurs in patients with preexisting asthma or CF. It is characterized by an allergic reaction to Aspergillus. Clinically, it is characterized by intermittent wheezing, b/l pulmonary infiltrates, brownish sputum, and peripheral eosinophilia. IgE may be elevated, suggesting an allergic process, and a specific reaction to Aspergillus that is manifested by serum ABS or skin testing is common. Although central bronchiectasis is common in ABPA, the presence of peripheral cavitary lung lesions is not a common feature

A sputum cx from a patient with CF showing which of the following organisms has been a/w a rapid decline in pulmonary function and poor clinical prognosis? A: Burkholderia cepacia B: Pseudomonas aeruginosa C: S aureus D: S epidermidis E: Stenotrophomonas maltophilia

A B cepacia is an opportunistic pathogen that has been responsible for nosocomial outbreaks. It also colonizes and infects the lower RT of patients with CF, chronic granulomatous disease, and SCD. In patients with CF it portends a rapid decline in PFT and a poor clinical prognosis. It also may cause a resistant necrotizing PNA. It is often intrinsically resistant to a variety of antimicrobials, including many B-lactams and Aminoglycosides. TMP-SMX is usually the first line treatment. Pseudomonas and Staph aureus are common colonizes and pathogens in patients with CF. Stentrophomonas is the pathogen, particularly in cancer, transplant, or critically ill patients. It is a cause of PNA, UTI, wound infection, and bacteremia. TMP-SMX is the DOC for Stenotrophomonas

A previously healthy 28 YO M describes several episodes of fever, myalgia, and HA that have been followed by ABD pain and diarrhea. He has experienced up to 10 bowel movements per day. PE is unremarkable. Labs are notable only for a slightly elevated WBC and an elevated ESR. Wright's stain of a fecal sample reveals the presence of PMNs. Colonoscopy reveals inflamed mucosa. Bx of an affected area discloses mucosal infiltration with PMNs, monocytes, and eosinophils; epithelial damage, including loss of mucus; glandular degeneration; and crypt abscesses. The patient notes that several months ago he was at a church BBQ where several people contracted a diarrheal illness. Although this patient could have IBD, which of the following pathogens is most likely to be responsible? A: Campylobacter B: E coli C: Norwalk virus D: S aureus E: Salmonella

A Campy are motile, curved gram-negative rods. The principal diarrheal pathogen is C jejuni. This organism is found in the GIT of many animals used for food production and is usually transmitted to humans in raw or undercooked food or through direct contact with infected animals. Over half of the cases are due to insufficiently cooked contaminated poultry. Campy is a common cause of diarrheal illness in the US. The illness usually occurs within 2-4 days after exposure to the organism in food or water. Bx of an affected jejunum, ileum, or colon reveals findings indistinguishable from those of Crohns disease and UC. Although the diarrheal illness is usually self-limited, it may be a/w constitutional ssx, lasts more than 1 week, and recurs in 5-10% of untreated patients. Cx include pancreatitis, cystitis, arthritis, meningitis, and GBS. The ssx of Campy are similar to those resulting from infection with salmonella, shigella, and yersinia; all of these cause fever and fecal WBCs. The dx is made by isolating it from the stool, which requires selective media. E coli (ETEC) is not a/w fecal WBCs, nor is Norwalk virus. It is far more common cause of a recurrent relapsing diarrheal illness that could be pathologically confused with IBD than Yersinia, Salmonella, Shigella, and EPEC

Capsofungin is a 1st line age for which of these? A: Candidemia B: Histoplasmosis C: Invasive aspergillosis D: Mucormycosis E: Paracoccidiomycosis

A Caspofungin and the other echinocandins (Anidulafungin, Micafungin) inhibit fungal synthesis of B-1,3-glucan synthase, a necessary enzyme for fungal cell wall synthesis that does not have a human correlate. These agents are available only IV, no po. They are fungicidal for Candida, and fungistatic for Aspergillus. Caspofungin is at least equivalent effective as Amphotericin B for disseminated candidiasis and is as effective as fluconazole for candidate esophagitis. It is not a first line therapy for Aspergillus but may be used as salvage. The echinocandins, including Caspofungin, have an extremely high safety profile. They do not have activity against Mucor, Paracoccidio, or Histoplasmosis

HIV+ patient with CD4 110 who is not taking meds presents to urgent care with c/o HA for the past week. He also notes nausea and intermittent blurry vision. Exam is notable for normal VS w/o fever, but mild papilledema. Head CT does not show dilated ventricles. The ddx test for the patient is: A: CSF cx B: MRI with gadolinium C: ophthalmologic exam including visual field testing D: serum Cryptococcal antigen test E: urine cx

A Cryptococcal meningoencephalitis presents with early manifestations of HA, nausea, gait disturbance, confusion and visual changes. Fever and nuchal rigidity are often mild or absent. Papilledema is present in ~30% of cases. Asymmetric CN palsies occur in 25%. Neuroimaging is often wnl. If there are FND findings, an MRI may be used to dx in the basal ganglia or caudate nucleus, although they are more common in immunocompetent patients with C neoformans var. gattii. Imaging does not make the dx. The ddx diagnosis remains CSF cx. However, capsular antigen testing in both serum and CSF is very sensitive and can provide presumptive dx. Approximately 90% of patients, including all with a positive CSF smear, and the majority of AIDS patients have detectable cryptococcal antigen. The result is often negative in patients with pulmonary disease. However, because of a very small FP rate in antigen testing. CSF cx remains the ddx diagnosis. In this condition it often can also be cx'd from the urine; however, other testing methods are more rapid and useful

24 YO M seeks evaluation for a painless penile ulceration. He noted the lesion 2 weeks ago, and since that time, two adjacent areas have also developed ulceration. He states that there has been blood staining his underwear from slight oozing of the ulcers. He has no PMH and takes no meds. He returned 5 weeks ago from a vacation in Brazil where he did have unprotected sexual intercourse with a local woman. He denies other high risk sexual behaviors and has never had sex with a prostitute. He was last tested for HIV 2 years ago. He has never had Chlamydia or Gonorrhea. On exam, there are (3) well defined red, friable lesions measuring 5-mm or less on the penile shaft. They bleed easily with any manipulation. There is no pain with palpation. There is shotty inguinal LAD. On bx of one lesion, there is a prominent intracytoplasmic inclusion of bipolar organisms in an enlarged mononuclear cell. Additionally, there is epithelial cell proliferation with an increased number of plasma cells and few PMNs. A rapid RPR is negative. Cx grows no organism. What is the most likely cause? A: Calymmatobacterium granulomatis (donovanosis) B: Chlamydia trachomatis (lymphogranuloma venereum) C: H ducreyi (chancroid) D: Leishmania amazonensis (cutaneous leishmaniasis) E: Treponema pallidum (secondary syphilis)

A Donovanosis is caused by the intracellular organism Calymmatobacterium granulomatis and most often presents as a painless erythematous genital ulceration after a 1-4 week incubation period. However, incubation periods can be as long as 1 year. The infection is predominantly sexually transmitted, and auto inoculation can lead to formation of new lesions by contact with adjacent infected skin. Typically the lesion is painless but bleed easily. Cx include phimosis in men and pseudo elephantiasis of the labia in women. If the infection is untreated, it can lead to progressive destruction of the penis or other organs. Dx is made by demonstration of Donovan bodies within large mononuclear cells on smears from the lesion. Donovan bodies refer to the appearance of multiple intracellular organisms within the cytoplasm of mononuclear cells. These organisms are bipolar and have an appearance similar to a safety pin. On histology, there is an increase in the number of plasma cells with few PMNs; additionally, epithelial hyperplasia is present and can resemble neoplasia. A variety of abx can be used to treat donovanosis including macrolides, tetracyclines, tmp/smx, and chloramphenicol. Treatment should be continued until the lesion has healed, often requiring >=5 weeks of treatment. All of the choices above are in the ddx of penile ulcerations. LV is endemic to the Caribbean. The ulcer of primary infection heals spontaneously, and the second phase of the infection results in markedly enlarged inguinal LAD, which may drain spontaneously. H ducreyi results in painful genital ulcerations, and the organism can be cx'd from the lesion. The painless ulcerations of cutaneous leishmaniasis can appear similarly to those of donovanosis but usually occur on exposed skin. Histology of intracellular parasites can distinguish leishmaniasis definitively from donovanosis. Finally, it is unlikely that the patient has syphilis in the setting of a negative RPR, and the histology is inconsistent with this dx.

In the developed world, seroprevalence of H pylori infection is currently: A: decreasing B: increasing C: staying the same D: unknown

A H pylori is a disease of overcrowding. Transmission has therefore decreased in the US as the standard of living has increased. It is predicated that the % of duodenal ulcers due to factors other than H pylori (e.g., NSAIDs) will increase over the upcoming decades. Controversial, but increasing, evidence suggests its colonization may provide some protection from GERD. Therefore, the health implications of H pylori eradication may not be simple.

64 YO M in Wisconsin develops a high fever and malaise over 2 days. He has spent his weekends over the past month chopping wood in his backyard. Initial labs show PMNs 1000/uL, PLT 84, AST 140, and ALT 183. A PBS reveals prominent morulae in PMNs. What is the likely diagnosis? A: Human granulocytic anaplasmosis B: Human monocytotropic ehrlichiosis C: Lyme Disease D: Rocky Mtn Spotted Fever E: SLE

A Human granulocytotropic anaplasmosis occurs mostly in the NE and Upper Midwest USA. It shares the Ixodes tick vector with Lyme disease. It is typically a disease of older males (median 51 YO). Because seroprevalence rates are high in endemic areas, subclinical infection is likely common. The disease typically presents with fever (>90%), myalgia, HA, and malaise. Thrombocytopenia, leukopenia, and elevated LFTs are common. ARDS, toxic-like syndrome, and opportunistic infections may occur, particularly in the elderly. Human granuolcytotropic anaplasmosis should be considered on the ddx of a flulike illness during May thru December in endemic regions. Morulae, intracytoplasmic inclusions, are seen in PMNs of up to 80% of cases of anaplasmosis on PBS and are diagnostic in the appropriate clinical context. This patient has high epidemiology risk based on long periods of time outside in an endemic region. Ehrlichiosis, which can be a more severe illness, has morulae in mononuclear cells (not PMNs) in a minority of cases. Lyme disease, which may be difficult to distinguish from anaplasmosis or ehrlichiosis, will not cause morulae. Treatment of Anaplasmosis is with Doxycycline

39 YO healthy man plans to travel to Malaysia and comes to clinic for appropriate vaccinations. He cannot recall which vaccines he has had in the past, but reports having had "all the usual ones" in childhood. Which of the following represents the MC vaccine-preventable infection in travelers? A: influenza B: measles C: rabies D: tetanus E: yellow fever

A INFLUENZA occurs year round in the tropics and is the most common vaccine-preventable infection in travelers. Documentation of vaccination against yellow fever is required in many countries. Measles is prevalent in much of the developing world, and all travelers should have documented vaccination. Tetanus should be UTD for international travelers, and rabies should be discussed.

Several family members present to a local ED 2 days after a large family summer picnic where deli meats and salads were served. They all complain of profuse diarrhea, HA, fevers, and myalgia. Their symptoms began ~24h after the picnic. It appears that everyone who ate Aunt Emma's bologna surprise was afflicted. Routine cx of blood and stool are negative to date. Which is true about Listeria gastroenteritis? A: abx is not necessary for uncomplicated cases B: carriers are asymptomatic but can easily spread via fecal:oral route C: GIT illness can result from ingestion of single organism D: illness is toxin-mediated, and organisms are not present at the time of infection E: person-person spread is a common cause of outbreaks

A Listeria causes GIT illness via ingestion of food that has been contaminated with high concentrations of bacteria. The bacteria may survive and multiply at refrigeration temperatures, therefore deli meats, soft cheeses, hot dogs, and milk are common sources. The attack rate is very high, with close to 100% of exposed patients experiencing ssx. SSX develop within 48 hours of exposure, and there is no prolonged asymptomatic carrier state. Person-person spread (other than vertical from mother to fetus) does not appear to occur during outbreaks. While the bacteria have several virulence factors that lead to clinical ssx, the organism, and not a specific toxin, mediates infection. A large inoculum is necessary to produce ssx. Surveillance studies show that <5% of asymptomatic adults have positive stool cx, and fecal:oral spread is not common. Typical ssx, including fever, are as described in the case above. Isolated GI illness does not require abx.

Borrelia burgdorferi serology is indicated for which of the following patients, all of whom reside in Lyme-endemic regions? A: 19 YO F camp counselor who presents with her 2nd episode of inflamed, red and tender L knee and R ankle B: 23 YO M house painter who present with a primary erythema migrans lesion at the site of the witnessed tick bite C: 36 YO F state park ranger who presents with malar rash, diffuse arthralgia/arthritis of her shoulders, knees, MCP and PIP joints; pericarditis; and glomerulonephritis D: 42 YO F with chronic fatigue, myalgia, and arthralgia E: 46 YO M gardener who presents with fever, malaise, migratory arthralgia/myalgia, and 3 erythema migrans lesions

A Lyme serology should be done only in patients with an intermediate pretest prob of having Lyme disease. The presence of erythema migrans in both patients B and E is diagnostic of Lyme in the correct epidemiology context. The diagnosis is entirely clinical. Patient C sounds more like SLE, and initial labs should focus on this dx. Patients with chronic fatigue, myalgia, and cognitive change are occasionally concerned about Lyme disease as a potential etiology for their ssx. However, the pretest prob of Lyme is low in these patients, assuming the absence of antecedent erythema migrans, and a positive serology is unlikely to be a true positive test. Lyme arthritis typically occurs months after the initial infection and occurs in ~60% of untreated patients. The typical attack is large joint, oligoarticular, and intermittent, lasting weeks at a time. Oligoarticular arthritis carries a broad ddx including sarcoidosis, spondyloarthropathy, RA, psoriatic arthritis, and Lyme. Lyme serology is appropriate here. Patients usually have the highest IgG antibody responses seen in this infection

45 YO patient with HIV/AIDS presents to the ED. He c/o a rash that has been slowly spreading up his R arm and is now evident on his chest and back. The rash consists of small nodules that have a reddish-blue appearance. Some of them are ulcerated, but there is minimal fluctuance or drainage. He is unsure when these began. He notes no foreign travel or unusual exposures. He is homeless and unemployed, but occasionally gets work as a day laborer doing landscaping and digging. A cx of a skin lesion grows a Mycobacterium in 5 days. Which is likely? A: M. abscessus B: M. avium C: M. kansasii D: M. marinum E: M. ulcerans

A Nontuberculous mycobacteria (NTM) were originally classified into "fast-growers" and "slow growers" based on the length of time they took to grow in cx. While more sophisticated tests have been developed, this classification scheme is still used and is of some benefit to the clinician. Fast-growing NTM include M. abscessus, foruitum, and chelonae. They will typically take 7 days or less to grow on media, allowing relatively fast ID and drug-resistance testing. Slow growers include M avium, marinum, ulcerans, and kansasii. They often require special growth media and therefore a high pretest suspicion. The patient described likely has a cutaneous infection from a "fast-growing" NTM, which could be diagnosed with tissue bx, gram stain, and cx.

Rifampin lowers serum levels of all of the following meds, except: A: amiodarone B: anticonvulsants C: cyclosporine D: hormonal contraceptives E: protease inhibitors F: warfarin

A Rifampin is considered the most potent and important anti-TB drug. It is also effective against other organisms, including some gram-positive and gram-negative organisms, as well as Legionella, Myocbacterium marinum, and kansasii. It is notable for turning body fluids a red-orange color. Its use should be avoided or carefully monitored in patients with severe hepatic disease, but it does not need to be dose-adjusted in renal failure. Rifampin is a potent CY3PA inducer and may lower the half-life and therefore effective levels of many important drugs, such as anticonvulsants, cyclosporine, OCPs, protease inhibitors, narcotics, TCAs, azoles, beta blockers, and many ABX. Patients need to be monitored for the effects of sub-therapeutic levels whether by directly measuring drug levels (anticonvulsants, cyclosporine), direct effects of the drug (warfarin), or with clinical adjustment (OCPs, protease inhibitors). While not studied extensively, rifabutin has a similar, although likely lesser, effect on the same meds as Rifampin. Amiodarone is not metabolized by CY3PA.

63 YO M from MS comes to your office for evaluation of a chronic sore on his thigh. He has an open sore on his anterior thigh that has been draining purulent material for many months. The thigh is NT but is warm to touch. The material is purulent and foul-smelling. He has been given multiple abx courses and recently finished a course of Itraconazole w/o relief of his ssx. He has an intact neuro exam of his LE. His ESR is 64, WBC is 15, and Hgb 8. A plain XR of the affected thigh shows a periosteal reaction of the femur and the skin. A gram stain of the pus shows broad-based budding yeast and you make a presumptive dx of blastomyces osteomyelitis. What is the treatment of choice for this patient? A: Amphotericin B B: Caspofungin C: Itraconazole D: Moxifloxacin E: Voriconazole

A although spontaneous cures of pulmonary infection with Blastomyces have been well documented, almost all patients with blastomycosis should be treated since there is no way to distinguish which patients will progress or disseminate. Extrapulmonary disease should always be treated, especially if the patient is immunocompromised. Itraconazole is indicated for non-central nervous system extra pulmonary disease in mild-moderate disease. Otherwise, Amphotericin B is the DOC, especially if there has been treatment failure with Itraconazole. The echinocandins have variable activity against B. dermatitidis and are not recommended for blastomycosis. The triazole antifungals have not been studied extensively in human cases of blastomycosis. Fluroquinolones have activity against many mycobacterial spp, but do not have activity against fungi, including B dermatitidis

Which single clinical feature has the most specificity in ddx Pseudomonas aeuriginosa sepsis from other causes of severe sepsis in a hospitalized patient? A: echthyma gangrenosum B: hospitalization for severe burns C: profound bandemia D: recent abx exposure E: recent ventilation for >14 days

A ecthyma gangrenous is a disseminated collection of geographic, painful, reddish, maculopapular lesion that rapidly progresses from pink to purple and finally to a black, dry necrosis. They are teeming with causative bacteria. In reviews on ecthyma, Pseudomonas aeruginosa is the MC isolate from blood and skin lesions. However, many organisms can cause this foreboding rash. Neutropenic patients and AIDS patients are at highest risk, but DM and ICU patients are also affected. Pseudomonal sepsis is severe with a high mortality. Its presentation is otherwise difficult to discern from other sepsis syndromes. with hypothermia, fever, hypotension, organ damage, encephalopathy, bandemia, and shock being the MC findings. Though abx use, severe burns, and long ICU stays increase the risk for Pseudomonas infection, these exposures are also RF for other bacterial infections, many of which also carry daunting resistance profiles. Because of its propensity for MDR, two agents (usually an anti-pseudomonas B lacteal plus an amino glycoside or ciprofloxacin) are warranted until cx data return confirming sens to one or both. At this point the choice to narrow 1 abx or not is still debated and is largely physician preference.

42 YO Nigerian M comes to the ED because of fevers, fatigue, weight loss, and cough for 3 weeks. He complains of fever and a 4.5-kg weight loss. He described his sputum as yellow in color. It has rarely been blood streaked. He emigrated to the US 1 year ago and is an undocumented alien. HE has never been treated for TB, has never had a PPD skin test placed, and does not recall receiving BCG vaccination. He denies HIV risk factors. He is married and reports no ill contacts. He smokes 1 ppd and drinks 1 pint of Vodka on a daily basis. On PE, he appears chronically ill with temporal wasting. BMI is 21. VS are: BP 122/68, HR 89, RR 22, SaO2 95% on RA, and Temperature 37.9C. There are amphoric breath sounds posteriorly in the RUL field with a few scattered crackles in this area. No clubbing is present. The exam is otherwise unremarkable. CT scan shows a large cavitary lesion in the RUL. A stain for AFB is negative. What is the most appropriate approach to the ongoing care of the patient? A: admit on airborne isolation until (3) expectorated sputum show no evidence of AFB B: admit w/o isolation as he is unlikely to be infectious with a negative AFB C: perform a bx of the lesion and consult oncology D: place a PPD on his forearm and have him return for evaluation in 3 days E: start a 6-week course of ABX for anaerobic bacterial abscess

A in this man from an endemic area for TB, with CT findings should be treated as active pulmonary TB until proven otherwise. In addition, his ssx suggest a chronic illness with low-grade fevers, wt loss, and temporal wasting that would be c/w active pulmonary TB. If a patient is suspected of having active pulmonary TB, the initial management should include documentation of disease while protecting workers and the general population. This patient should be hospitalized in a negative-pressure room on airborne isolation until (3) expectorated sputum samples have been demonstrated to be negative. The samples should preferably be collected in the morning as the burden of organisms is expected to be higher on a more concentrated sputum. The sens of a single sputum for the detection of TB is only 40-60%. Thus, a single sputum is inadequate to determine infectivity and the presence of active pulmonary TB. Skin testing with a PPD of the TB mycobacteria is used to detect latent infection with TB and has no role in determining whether active disease is present. The cavitary lesion on CT could represent malignancy or bacterial lung abscess, but given that he is from a high-risk area for TB, it should be considered the most likely dx until r/o by sputum.

50 YO M is admitted for active pulmonary TB with positive sputum and AFB. He is HIV positive with CD4 85, and is not on ART. In addition to pulmonary disease, he is found to have disease in L4 vertebrae. what is the most appropriate initial therapy? A: INH, Rifampin, ethambutol, and pyrazinamide (RIPE) B: RIPE + ART C: RIPE + Streptomycin D: RIE E: Withhold therapy until sensitivities are available

A initial treatment of active TB a/w HIV disease does not differ from that of a non-HIV person. The standard treatment regimen includes (4) drugs termed RIPE. These drugs are given for a total of 2 months in combination with Vitamin B6 (pyridoxine) to prevent neurotoxicity from INH. Following the initial 2 months, patients c/w INH and Rifampin to complete 6 months total. These recommendations are the same as those of non-HIV patients. If the sputum cx remains positive for TB after 2 months, the total course of antimycobacterial therapy is increased to 6-9 months. If an individual is already on ART at the time of dx, it may be continued, but often Rifabutin is substituted for Rifampin because of drug interactions with Rifampin and protease inhibitors. In individuals not on ART at the time of diagnosis, it is not recommended to start it concurrently because of the risk of immune reconstitution inflammatory syndrome (IRIS) and an increased risk of med AE. IRIS occurs as the immune system improves with ART and causes an intense inflammatory reaction directed against the infecting organism. There have been fatal cases of IRIS in a/w TB and initiation of ART. In addition, both ART and anti-TB drugs have many AE. It can be difficult for the clinician to decide which medication is the cause of the AE and may lead to unnecessary alterations in the anti-TB regimen. For these reasons, CDC recommends to await a response to treatment to anti-TB before initiation of ART. Three-drug regimens are a/w a higher relapse rate if used as standard 6 month course and, if used, require a total of 9 months. Situations in which 3 drug treatment is used are pregnancy, intolerance to a specific drug, and in the setting of resistance. A five drug regimen of RIPE + Streptomycin is recommended as the standard re-treatment regimen. Streptomycin and pyrazinamide are d/c'd after 2 months if susceptibility testing is unavailable. If susceptibility testing is available, the treatment should be based on the pattern. In no instance is it appropriate to withhold treatment in the setting of active TB to await susceptibility testing.

A patient admitted with fevers, malaise, and diffuse joint pain. His initial blood cx reveals MRSA in all cx bottles. He has no arthritis on exam, and his renal function is normal. ECHO shows a 5-mm vegetation on the aortic valve. He is initiated on IV-Vancomycin at 15 mg/kg q/12 hours. Four days later, the patient remains febrile and cx remain positive for MRSA. In addition to search for embolic foci of infection, which of the following changes should be made to treatment plan? A: no change B: add gentamicin C: add rifampin D: check Vancomycin serum peak and trough levels and consider TID dosing E: d/c Vancomycin, start Daptomycin

A patients with IE on abx can be expected to demonstrate clinical improvement within 5-7 days. Blood cx will frequently remain positive for 3-5 days for S aureus treated with B-lactam ABX and 7-9 days with Vancomycin. Neither Rifampin nor Gentamicin has been shown to provide clinical benefit in the scenario above. Vanc trough and peak levels have not been shown to improve drug efficacy in IE. It is too early in therapy to consider changing due to Vancomycin failure. The efficacy of Daptomycin or Linezolid as an alternative to Vanc for L-sided MRSA has not been established

53 YO M with hx of ETOH abuse presents with an enlarging mass at the angle of the jaw. The patient describes the mass slowly enlarging over a period of 6 weeks with occasional associated pain. He has also noted intermittent fevers throughout the period. Recently, he has developed yellowish drainage from the inferior portion of the mass. He takes no meds and has no PMH. He drinks 6 beers daily. On PE, the T 37.9 (100.2). His dentition is poor. There is diffuse soft tissue swelling and induration at the angle of the mandible on the left. It is mildly tender, and no discrete mass is palpable. The area of swelling is ~8x8 cm. An aspirate is sent from gram stain and cx. The culture grows Eikenella corrodens. After 7 days you receive a call reporting growth of a gram-positive bacillus branching at acute angles on anaerobic media. What organism is causing this? A: Actinomyces B: Eikenella corrodens C: Mucormycosis D: Nocardia E: Peptostreptococcus What is the best treatment?

A the MC site of Actinomycosis infection is the craniofacial area. Often the injection is a/w poor dentition, facial trauma, or tooth extraction. Clinically this presents as a chronic cellulitis of the face, often with drainage through sinus tracts. The infection may spread w/o regard for tissue planes, and adjacent bony structures may become involved. Dx requires a high degree of suspicion. The drainage is frequently contaminated with other organisms, especially gram negative rods. The characteristic sulfur granules may not be seen unless deep tissue is sampled. On gram's stain, the appearance shows an intense gram positive center and branching rods at the periphery. As opposed to the strictly aerobic Nocardia spp, Actinomyces grows slowly in anaerobic and micro aerobic conditions. Therapy requires a long course of abx, even though the organism is sensitive to PCN. This is presumed to be due to the difficulty of using abx to penetrate the thick-walled masses and sulfur granules. Current recommendations are for PCN IV for 2-6 weeks followed by po therapy for a total of 6-12 months. Surgery is reserved for patients who are not responsive to medical therapy.

Which of the following is the MC source of fever in travelers returning from SE Asia? A: dengue fever B: malaria C: mononucleosis D: salmonella E: yellow fever

A the causes of fever in travelers vary by geography. In general, all febrile travelers returning from malaria-endemic regions should be assumed to have malaria until r/o or another dx established, since falciparum malaria may be life-threatening and effective tx is available. Dengue is particularly common in SE Asia. Most cases are self-limited and require supportive therapy. A small proportion, however, can developed hemorrhagic fever or a shock syndrome. Sub-saharan Africa: malaria Caribbean: dengue Central America: malaria South America: Dengue South Central Asia: Salmonella

Sens and spec serum or urine diagnostic tests exist for all of the following invasive fungal infections, except: A: blastomycosis B: coccidiomycosis C: cryptococcosis D: histoplasmosis

A the ddx of an invasive fungal infection generally requires histologic demonstration of fungus invading tissues along with an inflammatory response. However, coccidioides serum complement fixation, cryptococcal serum and CSF antigen, and urine/serum histoplasma antigen are all tests with good performance characteristics, occasionally allowing for presumptive diagnosis. There is no such widely used serology for blastomycosis. Serum testing for galactomannan is approved for the diagnosis of Aspergillus. However, FN may occur, and further studies are necessary

79 YO M has had a DM foot ulcer overlying his 3rd metatarsal head for 3 months but has not been compliant with his physician's request to offload the affected foot. He presents with dull, throbbing foot pain and subjective fevers. Exam reveals a putrid-smelling wound notable also for a pus-filled 2.5-cm wide ulcer. A metal probe is used to probe the wound and it detects bone as well as 3-cm deep cavity. Gram stain of the pus shows gram-positive cocci in chains, gram-positive rods, gram-negative diplococci, enter-appearing gram-negative rods, tiny pleomorphic gram-negative rods, and a predominance of neutrophils. Which of the following empirical ABX regimens is recommended while blood and drainage cx are processed? A: Ampicillin/sulbactam, 1.5 g IV q/4h B: Clindamycin, 600 mg po tid C: Linezolid, 600 mg IV bid D: Metronidazole, 500 mg po qid E: Vancomycin, 1 g IV bid

A the gram stain is polymicrobial and the putrid smell is specific for anaerobic organisms. The dx of acute osteomyelitis is also very likely based on the positive probe to bone test and wide ulcer. Broad-spectrum abx are indicated. Vancomycin and Linezolid cover MRSA and strep isolates but would miss gram-negative rods and anaerobic bacteria. Metronidazole covers only anaerobes, missing gram positives that are key in the initiation of DM foot infections. Clindamycin covers gram-positives and anaerobes, but misses gram-negative rods. Ampicillin-sulbactam (Unasyn) is broad-spectrum and covers all (3) classes of organisms. If the patient has a hx of MRSA or MRS RF, then the addition of Vancomycin or Linezolid is a strong consideration

24 YO M with advanced HIV presents to the ED with a tan painless nodule on the LE. He is afebrile and has no other lesions. He does not take ART, and his last CD4 was 20. He lives with a friend who has cats and kittens. A biopsy shows lobular proliferation of blood vessels lined by enlarged endothelial cells and a mixed acute and chronic inflammatory infiltrate. Tissue stains show gram-negative bacilli. Which of these is most likely to be effective for the lesion? A: Azithromycin B: Cephazolin C: IFN alpha D: Penicillin E: Vancomycin

A the patient has bacillary angiomatosis due to cutaneous infection with Bartonella Quintana or Bartolenna henselae. Kittens are the likely source of the infection in this case. Bacillary angiomatosis occurs in HIV infected patients with CD4 <100. The cutaneous lesions of bacillary angiomatosis are typically painless cutaneous lesions but may appear as SubQ nodules, ulcerated plaques, or verrucous growths. They may be single or multiple. The ddx includes Kaposi, pyogenic granuloma, and tumors. Bx findings are as described in this case, and the dx is made by histology. Tx is with azithromycin or doxycycline. Oxacillin or Vancomycin is the treatment for Staphylococcal or Streptococcal skin infections.

testing for latent TB infection is indicated in HIV patients at the time of initial diagnosis for all of the following reasons, except: A: active TB treatment success rates are lower in HIV infected patients compared to uninfected patients B: drug interactions between drug regimens for active TB and HART are challenging to manage C: HIV-assoc active TB is more likely to be extra pulmonary and can be diagnostically challenging D: HIV infected patients with active TB have high 6-month MR E: the rate of progression from latent TB to active TB is higher in HIV patients compared to uninfected patients

A the purpose of testing for latent TB (with either PPD or IFN) in any person is to detect latent TB and treat at that stage to avoid development of active TB. This strategy benefits the individual patient and the PUBH. These issues are more pressing in persons with HIV infection. The progression from latent to active TB in HIV is estimated as high as 10% per year rather than 10% per lifetime in uninfected. In HIV, active TB is clinically present in extra pulmonary sites (kidney, CNS) and may be diagnostically challenging. TB infection appears to accelerate HIV disease. The 6-month MR among co-infected patients is higher than with HIV alone. Rifamycin derivatives, used for active TB, have fairly complex drug-drug interactions with ART agents often necessitating ART regimen exchange or dose adjustment. Appropriate tx for active TB has similar efficacy rates for mycobacterial eradication in HIV infected and uninfected persons

Glucocorticoids have been shown to be of benefit for treatment for all of the following infections, except: A: Aspergillus PNA B: MTB pericarditis C: PCP PNA D: severe typhoid fever E: S pneumo meningitis

A the role of immune modulators in serious infections has received increased attention in recent years as understanding of the cytokine and inflammatory systems has evolved. In several models, an aggressive host anti-inflammatory response may increase organ damage. Steroids are useful adjuncts for several infections as ABX alone can increase inflammation and cytokine release due to lysis of intact organism and release of pro-inflammatory intracellular content. However, mold infections, such as Aspergillus, are apt to worsen in the setting of glucocorticoids

38 YO homeless male presents to ED with TIA characterized by a facial droop and L arm weakness lasting 20 mins, and LUQ pain. He reports intermittent subjective fevers, diaphoresis, and chills for the past 2 weeks. He has had no recent travel or contact with animals. He has taken no recent abx. PE reveals a slightly distressed man with disheveled appearance. His T is 38.2C, HR 90, BP 127/74. He has poor dentition. Cardiac exam reveals an early diastolic murmur over the left 3rd ICS. His spleen is tender and 2 cm descended below the costal margin. He has tender painful red nodules on the tips of his 3rd finger of his R hand and on the 4th finger of his L hand that are new. He has nits evident on his clothes, c/w body louse infection. WBC 14.5 with 15% bands and 98% PMNs. Blood cx are drawn followed by empiric Vancomycin. These cx remain negative for growth 5 days later. He remains febrile but HD stable but does develop a new lesion on his toe similar to those on his fingers on hospital Day 3. A TTE reveals a 1-cm mobile vegetation on the cusp of his aortic valve and moderate AR. A CT scan of the ABD shows an enlarged spleen with wedge-shaped splenic and renal infarctions. What test should be sent to confirm? A: Bartonella serology B: EBV heterophile antibody C: HIV PCR D: PBS E: Q fever serology

A this patient has culture-negative endocarditis, a rare entity defined as clinical evidence of IE in the absence of positive blood cultures. In this case, evidence for subacute bacterial endocarditis includes valvular regurgitation, aortic valve vegetation, and embolic phenomena on the extremities., spleen, and kidneys. A common reason for negative blood cultures is prior antibiotics. In the absence of this, the two most common pathogens (both of which are technically difficult to isolate in blood cx bottles) are Q fever, or Coxiella burnettii (typically a/w close contact with livestock), and Bartonella. In this case, the patient's homelessness and body louse infection are clues for Bartonella Quintana infection. Dx is made by blood cultures about 25% of the time. Otherwise, direct PCR of valvular tissue, if available, or acute and convalescent serologies. Empiric treatment for culture negative endocarditis usually includes ceftriaxone and gentamicin, with or without doxycycline. For confirmed Bartonella endocarditis, optimal therapy is gentamicin plus doxycycline. EBV and HIV do not cause endocarditis. A PBS would not be diagnostic.

26 YO F during a clinic is found to have a positive RPR (1:4) and a positive fluorescent treponema antibody absorption test (FTA-ABS). She has never been treated for syphilis. She recalls a large painless ulcer on her labia 9 months prior, followed about 2 months later by a diffuse rash and oral lesions that also resolved. She has had 5 sexual contacts in the past year. In addition to treating the patient, all of the following additional interventions should be considered, except: A: ECHO looking at the aortic arch B: HIV counseling and testing C: pregnancy testing D: screening and treatment of recent contacts E: screening for other STDs

A this patient has syphilis of <1 year duration (early latent). HIV counseling and testing are important for all patients diagnosed with syphilis, given the shared high risk behaviors a/w transmission of both infectious as well as increased risk of neurosyphilis in co-infected patients. Pregnancy status is important to know as congenital syphilis is a concern. All recent sexual contacts should be screened and treated empirically for syphilis, regardless of serologic or clinical status, as the latency period between exposure and primary syphilis can last as long as 3 months. Individuals with syphilis are at higher risk of other STDs such as C/G. Syphliis aortitis is not a concern in a patient with early latent syphilis.

38 YO F is seen in clinic for decrease in cognitive and executive function. Her husband is concerned as she no longer pays bills, keeps appointments, or remembers important dates. She also seems to derive considerably less pleasure from caring for her children and her hobbies. She is unable to concentrate for long enough to enjoy movies. This is a clear change from her functional status 6 months ago. A workup reveals a positive HIV antibody by ELISA and Western blot. Her CD4 378, with viral load 78,000/mL. She is afebrile with normal VS. Her affect is blunted, and she seems disinterred in the medical interview. Neuro exam for strength, sensation, cerebellar function, and CN function is nonfocal. Fundoscopy is normal. MMSE is 22/30. RPR is negative. MRI of the brain shows only cerebral atrophy disproportionate to her age but no focal lesions. What is the next step? A: ART B: CSF JV virus PCR C: CSF mycobacterial PCR D: CSF VDRL test E: Serum Cryptococcal antigen F: Toxoplasma IgG

A this patient most likely has HIV encephalopathy of moderate severity. Other neurologic conditions a/w HIV may be considered with a broad initial workup, but her reasonably high CD4 count, lack of FNDs, and lack of mass lesions on MRI makes toxoplasmosis, CNS tuberculoma, PML, or CNS lymphoma are less likely. Immediate ART is the treatment of choice, and she warrants this despite the CD4 count placing her in a gray zone according to recent guidelines. A LP looking for VDRL is unnecessary as a serum RPR is a good screening test for syphilis; JC virus in the CSF would suggest PML, but her pretest prob for this is low because it usually affects patients with low CD4. Serum Cryptococcal antigen has excellent performance characteristics, but there is little reason to suspect cryptococcal meningitis in the absence of HA or elevated ICP

Which of the following serology patterns places a transplant recipient at the lowest risk of developing CMV infection after renal transplant? A: donor CMV IgG negative, recipient IgG negative B: donor IgG negative, recipient IgG positive C: donor IgG positive, recipient IgG negative D: donor IgG positive, recipient IgG positive E: risk is equal regardless of serology

A when the donor is CMV IgG positive and the recipient is negative, there is a very high risk of primary CMV infection in the recipient. However, if the recipient is positive, CMV occurs as ar reactivation infection. When both donor and recipient are seronegative, then the risk of any CMV infection is lowest, but not zero, as a contact with an infected host could prompt primary CMV infection. Unlikely nearly all other transplant patients, many donor and recipient seronegative patients do not receive chemoppx with ganciclovir. In patients who are Cmv IgG negative and received a CMV IgG negative transplant, transfusions should be from IgG negative donors or WBC filtered products administered to reduce the risk of primary CMV infection

An otherwise healthy 5-year old child presents with low grade fevers, sore throat, and red, itchy eyes. He attends summer camp, where several other campers were ill. On exam, the patient is noted to have pharyngitis and b/l conjunctivitis. Which is the most likely diagnosis? A: adenovirus B: enterovirus C: influenza D: metapneumovirus E: rhinovirus

A while most of the choices listed can cause pharyngitis in children, Adenovirus classically presents with b/l granular conjunctivitis as well as pharyngitis, and is frequently the cause of an outbreak among children who are in close contact with one another. Symptom-based and supportive therapies are indicated for all infections other than disseminated infections in immunocompromised patients. Rhinovirus infections manifest clinically as a common cold with ST and rhinorrhea. Human metapneumovirus is a recently described respiratory pathogen. Infections usually occur in winter, and antibodies are present in most children by age 5 YO. Clinically HMPV appears similar to human RSV, with upper and lower respiratory symptoms. Serious infections may occur in immunocompromised patients. Parainfluenza predominantly is a mild coldlike illness in older children and adults, presenting with hoarseness often w/o cough. Enterovirus most frequently causes an acute undifferentiated febrile illness but may cause rhinitis, pharyngitis, and PNA.

All of the following are clinical manifestations of Ascaris lumbricoides infection, except: A: asymptomatic carriage B: fever, HA, photophobia, nuchal rigidity, and eosinophilia C: NP cough and pleurisy with eosinophilia D: RUQ pain and fever E: small bowel obstruction

B Ascaris is the longest nematode (15-40 cm) parasite of humans. It resides in tropical and subtropical regions. In the US, it is found mostly in the rural SE. Transmission is through fecally contaminated soil. MC the worm burden is low and is causes no ssx. Clinical disease is related to larval migration to the lungs or to adult worms in the GIT. The MC cx occur due to high GIT adult worm burden leading to SBO (most often in children with narrow caliber SI lumen) or migration leading to obstruction complications such as cholangitis, pancreatitis, or appendicitis. Rarely, adult worms can migrate to the esophagus and be orally expelled. During the lung phase of larval migration (9-12 days s/p egg ingestion) patients may develop NP cough, fever, eosinophilia, and pleuritic CP. Eosinophilia pneumonia syndrome (Loffler's syndrome) is characterized by symptoms and lung infiltrates. Meningitis is not a known cx of ascariasis but can occur with disseminated strongyloides in an immunocompromised host.

Current CDC recommendations are that screening for HIV be performed in: A: all high-risk groups (IVDA, MSM, high-risk heterosexual women) B: all US adults C: IVDA D: MSM E: women who have sex with more than 2 men per year

B CDC states that all adults should receive HIV testing with the availability of a patient opt-out mechanism rather than informed consent. The basis for this is that ~25% of the 1 M Americans infected with HIV are unaware of their status, there is good available treatment for HIV that serves to extend the lifespan and decrease HIV transmission, and HIV testing is shown to correlate with a decrease in risk taking behaviors. Cost-benefit analysis has suggest this has advantages to current approaches focusing on screening high-risk populations. Pretest counseling is desirable but not always built into the testing process so physicians provide some degree of prep for a positive test. If the dx is made, support should be activated that may include trained nurses, SW, or community support centers.

38 YO M with HIV/AIDS presents with 4 weeks of diarrhea, fever, and weight loss. Which of the following makes the diagnosis of CMV colitis? A: CMV IgG B: Colonoscopy with biopsy C: Serum CMV PCR D: Stoom CMV antigen E: Stool CMV cx

B CMV colitis should are considered in AIDS patients with CD4 <50, fevers, and diarrhea. Diarrhea is often bloody but can be watery. Initial evaluation often involves stool studies to r/o other parasitic or bacterial causes of diarrhea in AIDS. A standard panel will include some or all of the following depending on epidemiology and history: C diff stool antigen, stool cx, stool MAC, stool O + P, and special stains for Crypto, Isospora, Cyclospora, and Microsporidia. There is no stool or serum test that is useful for CMV colitis eval in an HIV patient. A positive CMV IgG is merely a marker of past infection. If this test is negative, then the pretest prob of active CMV decreases substantially. Serum CMV PCR has gained utility in solid organ and BMT patients for following treatment response for invasive CMV infection. However, in HIV patients, CMV viremia correlates imprecisely with colitis. Further, because it is a latent-lytic herpesvirus, a positive serum PCR does not imply disease unless drawn in the right clinical context, for which there is none in HIV. Colonic history is sens/spec for CMV colitis, with large-cell inclusion bodies being diagnostic.

38 YO F with frequent hospital admissions related to ETOH comes to the ED after being bitten by a dog. There are open wounds on her arms and R hand that are purulent and have necrotic borders. She is hypotensive and admitted to the ICU. She is found to have disseminated intravascular coagulation and soon develops MOF. Which of the following is the most likely organism to have caused her rapid decline? A: Aeromonas B: Capnocytophaga C: Eikenella D: Haemophilus E: Staphylococcus

B Capnocytophaga canimorsus is the MC organism to have caused fulminant disease in this ETOH patient following a dog bite. Eikenella and Haemophilus are common mouth flora in humans but not in dogs. Staphylococcus can cause sepsis but is less likely in this scenario

What is the MC manifestation of Coccidiodes infection in the immunocompetent host? A: acute PNA B: asymptomatic seroconversion C: HSN phenomena such as erythema nodosum D: meningitis E: self limited flu-like illness

B Coccidiodomycosis is a very diverse disease in terms of its scope of clinical presentation, as well as severity. 60% of infections as determined by serologic conversion are asymptomatic. The MC syndrome in the other 40% is an acute respiratory illness characterized by fever, cough, and pleuritic pain. Skin manifestations, such as erythema nodosum, are also common with Coccidioides. It can cause a severe and difficult to treat meningitis in AIDS patients, other immunocompromised patients, and occasionally immunocompetent hosts, and can occasionally cause ARDS and fatal multi lobar PNA. Coccidioides is confined to the western hemisphere. Endemic regions include S Arizona, the Central Valley of California, and N Mexico, where it resides in soil. The risk of symptomatic infection increases with age.

25 YO F presents to clinic c/o several days of worsening burning and pain with urination. She describes an increase in urinary frequency and suprapubic tenderness but no fever or back pain. She has no PMH with the exception of two prior episodes similar to this in the past 2 years. UA shows moderate WBC. Which is the most likely organism? A: Candida B: E coli C: Enterobacter D: Klebsiella E: Proteus

B E coli is the causative agent in 85-95% of uncomplicated UTIs that occur in premenopausal women. Uncomplicated cystitis is the MC UTI syndrome. 20% of women will develop a recurrence in 1 year after the initial UTI. Pregnant women are at high risk of cystitis developing into pyelonephritis. Proteus represents only 1-2% of uncomplicated UTI. Proteus causes 20-45% of UTIs in patients with long term bladder catheterization. Klebsiella also accounts for only 1-2% of uncomplicated UTIs; however, it is responsible for 5-17% of complicated UTIs. Enterobacter is a rare cause of infection outside of the hospital. Candida is most often a GU colonizer in health patients and is rarely the cause of infection

23 YO F develops CMV pneumonitis 5 months s/p lung transplant. She developed severe AE from Ganciclovir while receiving ppx. Foscarnet is RX'd for this episode. Which AE is most likely? A: BM suppression B: -lyte wasting C: embryotoxic D: lethargy and tremors E: hyperkalemia

B Foscarnet is a potent agent used for drug-resistance CMV, HSV, and VZV, or for patients who are intolerant of 1st line agents. It may cause AKI. It also binds divalent metals, commonly causing hypokalemia, hypocalcemia, hypophosphatemia, and hyperphosphatemia. It is often poorly tolerated as a result of nausea and malaise. Ganciclovir commonly causes BM suppression and significant neutropenia when used for CMV, necessitating a switch to Foscarnet. Foscarnet commonly causes AKI. Acylovir (HSV, VZV) is generally very well tolerated but may cause lethargy and tremors. Aerosolized Ribavirin is used to treat RSV in infants. It is a mutagen and teratogen and is embryo toxic.

26 YO F presents late in the 3rd trimester of her pregnancy with high fevers, myalgia, backache, and malaise. She is admitted and started on empirical broad-spectrum abx. Blood cx return positive for Listeria. She delivers a 5-lb infant 24 hours after admission. Which is true about abx for this infection? A: clindamycin should be used in patients with PCN allergy B: neonates should receive weight-based Ampicillin and Gentamicin C: PCN plus Gentamicin is 1st line for the mother D: Quinolones should be used for Listeria in late-stage pregnancy E: TMP/SMX has no efficacy against Listeria

B Listeria bacteremia in pregnancy is a relatively rare but serious infection for both mother and fetus. Vertical transmission may occur, with 70-90% of fetuses developing infection from mother. Preterm labor is common. Pre-partum treatment of the mother increases the chance of healthy delivery. Mortality among fetuses approaches 50% and is much lower in neonates receiving appropriate abx. First-line therapy is with Ampicillin, with Gentamicin often added for synergy. This recommendation is the same for mother and baby. In patients with a true PCN allergy, the therapy of choice is TMP-SMX. Quinolones have shown animal model and in vitro efficacy against Listeria, but there is not enough evidence to recommend these as first line.

All of these are clinical findings c/w the diagnosis of molloscum contagious except: A: involvement of the genitals B: involvement of the soles of the feet C: lack of inflammation or necrosis at the site of the rash D: rash a/w eczematous eruption E: rash spontaneously resolving over 3-4 months

B MC isa. cutaneous poxvirus infection with a distinctive cutaneous appearance. The rash typically consists of collections of 2-5 mm umbilicated papule that can occur anywhere on the Boyd except the palms and soles. It can be accompanied by an eczematous reaction. MC is transmitted through close contact including sexual contact, which will cause genital involvement. Unlike other poxvirus lesions MC is not a/w inflammation or necrosis. In immunocompetent patients, the disease is usually self-limiting; rash will subside within several months time. Systemic involvement does not occur.

76 YO F is brought to clinic by her son. She c/o chronic NP cough and fatigue. Her son adds that she has a low-grade fever, progressive weight loss over months, and "just does not seem like herself." Chest CT reveals bronchiectasis and small (<5 mm) nodules scattered throughout the parenchyma. She had a distant hx of treated TB. A sputum sample is obtained, as are blood cx. Two weeks later, both cx sets grow AFB c/w MAC. Which is the best treatment? A: bronchodilators and pulmonary toilet B: clarithromycin and ethambutol C: clarithromycin and rifampin D: moxifloxacin and rifampin E: pyrazinamide, INH, rifampin, and ethambutol

B Non-TB Mycobacteria, such as MAC, may cause chronic pulmonary infections in normal hosts and those with underlying pulmonary disease immunosuppression. In immunocompetent persons without underlying disease, treatment of pulmonary infection with MAC is considered on an individual basis based on ssx, radiology, and bacteriology. Treatment should be initiated in the presence of progressive pulmonary disease or ssx. In patients w/o prior lung disease and who do not demonstrate progressive clinical decline, MAC pulmonary infection can be managed conservatively. Patients with underlying lung disease, such as COPD or CF, or those with hx of pulmonary TB should receive ABX. In the vignette, the patient has both clinical and historical reasons for ABX. The appropriate regimen is clarithromycin and ethambutol. The combination of RIPE is less effective for MTB, which is not presented here. Quinolones have shown promise but are not first line therapy. Rifampin has no role.

You are a physician working on a cruise ship traveling from Miami to the Yucatan. In the course of 24 hours, 23 people are seen with an acute GIT that is marked by vomiting and watery diarrhea. The most likely cause is: A: enterhemorrhagic E coli B: norovirus C: rotavirus D: Shigella E: Salmonella What is the best way to diagnose?

B Norovirus, or Norwalk-like agent, was initially described as a cause of forborne illness in Norwalk, Ohio, in 1968. Since that time the virus has been ID'd as a small RNA virus of the Calciviridae family. The initial detection was poor, relying on E Microscopy or immune electron microscopy. Using these techniques, it was ID'd as the causative agent in 19-42% of nonbacterial diarrheal outbreaks. With the development of more sensitive assays (RT PCR, ELISA), they are being found as increasingly frequent causes of diarrheal outbreaks. Treatment is supportive as ssx improve within 10-51 hours. Rotavirus is the MC cause of viral diarrhea in infants but is uncommon in adults. Salmonella, shigella, and E coli present with more colonic and systemic manifestations.

A patient who has undergone prosthetic valve surgery 6 weeks ago is readmitted with ssx c/w Infective Endocarditis. Which of the following is the most likely etiologic organism? A: Candida albicans B: Coagulase-negative Staphylococci C: Enterococcus D: E coli E: Pseudomonas aeruginosa

B Prosthetic cardiac valves are at high risk of developing endocarditis after bacteremia. Patients who develop endocarditis within 2 months of valve surgery most likely have acquired their infection nosocomially as a result of intra-operative contamination of the prosthesis or of a bacteremic post-op event. Coagulase-negative staphylococci are the MC (33%) nosocomial pathogens during this time frame, followed by S aureus (22%), facultative gram-negative bacilli (13%), enterococci (8%), diphtheroids (6%), and fungi (6%). The modes of infection and typical organisms causing prosthetic valve endocarditis >12 months after surgery are similar to those in CA-endocarditis. Both sets must be considered in the intermediate 2-12 months s/p surgery.

17 YO M in Arkansas presents to clinic in August with fever, HA, myalgia, nausea, and anorexia 8 days after returning from 1-week camping trip. PE is remarkable for a T 38.6C and a generally fatigued but nontoxic appearing, well-developed young man. He does not have a rash, and orthostatic VS measurements are negative. What would be reasonable? A: Initiate Ceftriaxone, 1 g IM x1 B: Initiate Doxycycline, 100 mg po bid C: Initiate Oseltamavir, 75 mg po qd D: Reassure the patient and order a heterophile antibody titer (mono spot) E: reassure the patient and order rickettsial serologies

B Rocky Mtn Spotted Fever, caused by Ricetssia rickettsii occurs throughout the US, Canada, Mexico, Central America, and South America. It is transmitted by the dog tick in the Eastern 2/3 of the US and by the wood tick in the Western US. Humans are typically infected during tick season from May - September. In the preantibiotic era, the case fatality rate approached 25%. Currently, the MR is ~5%, mostly due to delayed recognition and therapy. The incubation period after a tick bite is approximately 1 week. The initial ssx are entirely nonspecific, and the typical rash is often not seen in early disease. It may be difficult to distinguish from many self-limited viral syndromes. Only 60% of patients recall a tick bite, and only 3% of them have the classic hx of tick bite, fever, and rash. Therefore, assessment for this potentially deadly disease should be evaluated. His recent high risk travel period for tick bite in a highly endemic area makes this a high pretest prob. Empirical treatment is warranted and will not affect the diagnostic workup. A diagnostic indirect immunoflourescent antibody test will not be positive (>=1:64 titer) until 7-10 days after symptoms. The only diagnostic test that is useful during the acute illness is IHC for Rickettsia of a biopsy from a skin rash. Doxycycline is an effective therapy and should not be continued until the patient is afebrile and improving clinically for 2-3 days.

A person with liver disease caused by Schistosoma mansoni would be most likely to have: A: ascites B: esophageal varices C: gynecomastia D: jaundice E: spider nevi

B Schistosoma mansoni infection of the liver causes cirrhosis from vascular obstruction resulting from periportal fibrosis but relatively little heptocellular injury. HSM, hypersplenism, and esophageal varices develop quite commonly, and schistosomiasis is usually a/w eosinophilia. Spider nevi, gynecomastia, jaundice and ascites are less commonly observed than they are in ETOH and post-necrotic fibrosis

68 YO F has been in the MICU for 10 days with COPD flare and PNA, including the initial 6 days on a mech vent. She just finished a course of Moxifloxacin and GC taper when she develops ABD discomfort over 2 days. VS reveal T 38C, HR 94, BP 162/94, RR 18, and O2 Sat 90%. On exam, she is in moderate distress. She is not usually accessory ms, but she is tachypneic. She has a slight b/l wheeze with good air movement. Heart sounds are distant and unchanged. Her ABD is moderately distended and tense, with scant BS present. There is no guarding or rebound, but she is tender throughout. Review of her records reveals no bowel movement over the past 72 hours and no stool is palpable in the rectal vault. WBC has increased from 7.1 to 38 over the past 2 days. ABD plain XR shows what is read as probable ileus in the RLQ. Aside from NG tube with suction and NPO, which of the following should be included? A: IV-Ig B: Metronidazole, 500 mg IV TID C: Pip/Tazo, 3.37 g IV q/6h D: Restart Moxifloxacin, 400 po qd E: Vancomycin, 500 mg po QID

B Severe C diff may mimic a surgical ABD and patients may not have diarrhea. The lack of diarrhea should not overshadow the other signs and RF that are suggestive of C diff, including the WBC, long hospitalization, prior ABX, and probable enteral tube feeds on the vent. Dynamic ileus is a serious and well-known cx of C diff. All potentially serious manifestations that could be C diff should be empirically treated as such until stool antigen tests are negative and an alternative clinical explanation is found. IV-Metronidazole may be less optimal than po Vancomycin for severe cases, and this patient may fail. However, po medicines are less likely to reach the target organ in the presence of an adynamic ileus, necessitating IV Metronidazole. Some advocate combination of po Vanc by NGT with IV Metro. All potentially offending ABX should be stopped (if possible, as is the case here with the patient having recovered from her PNA) rather than continued. Surgical colectomy may be necessary in fulminant cases when there is no response to medical therapy. IV-Ig, which may provide ABS to C diff toxin, are reserved for severe or multiple recurrent cases

While attending UGA, a group of friends go on a 5-day canoeing and camping trip in Southern GA. A few weeks later, one of the campers develops a serpiginous, raise, pruritic, erythematous eruption on his buttocks. Strongyloides larvae are found in his stool. Three of his companions, who are asymptomatic, are also found to have strongyloides larvae in their stool. Which of the following is indicated for asymptomatic carriers? A: fluconazole B: ivermectin C: mebendazole D: mefloquine E: treat only symptomatic illness

B Strongyloides is the only helminth that can replicated in the human host, allowing auto infection. Humans acquire stronglyoides when larvae in fecally contaminated soil penetrate the skin or mucous membranes. The larvae migrate to the lungs via the bloodstream, break through the alveolar spaces, ascending the respiratory airways, and are swallowed to reach the SI where they mature into adult worms. Adult worms may penetrate the mucosa of the SI. Strongyloides is endemic in SE Asia, Sub-saharan Africa, Brazil, and the Southern USA. Many patients are asymptomatic or have mild GIT ssx or the characteristic cutaneous eruption, larval currens, as described in this case. Small bowel obstruction may occur with early heavy infection. Eosinophilia is common with all clinical manifestations. In patients with impaired immunity, particularly glucocorticoid therapy, hyper infection or dissemination may occur. This may lead to colitis, enteritis, meningitis, peritonitis, and AKI. Bacteremia or gram negative sepsis may develop due to bacterial translocation through disrupted enteric mucosa. Because of the risk of hyper infection, all patients with Strongyloides, even asymptomatic carriers, should be treated with Ivermectin, which is better than Albendazole. Fluconazole is used to treat candida. Mebendazole is used to treat trichuris, enterobiasis, ascariasis, and hookworm. Mefloquine is used for malaria.

28 YO F returns from a 6-week trip to Tanzania in March. She calls your office 2 weeks later c/o new symptoms of fever, mild ABD pain, and HA. She feels like she has the flu. What is the next step? A: ask her to come to clinic in the next 24 hours B: emergently refer her to the ED C: write a Rx for oseltamivir and call her in 24 hours to ensure improvement D: write a Rx for respiratory fluroquinolone

B any returning traveler to a region where Plasmodium falciparum is endemic who develops a fever warrants emergent evaluation for the MC and dangerous infection in the returning traveler: malaria. Falciparum is the potentially fatal form of malaria which can lead to overwhelming sepsis, renal failure, and cerebral edema; it is also the MC form of malaria in Africa. This patient should be referred to the ED for thick and thin smear. If a smear cannot be performed and interpreted in an expeditious fashion, then empirical doxycycline and quinine should be started. SSX of malaria are nonspecific but include fever, HA, ABD pain, jaundice, myalgia, and AMS.

51 YO F is diagnosed with Plasmodium falciparum malaria after returning from safari in Tanzania. Her parasitemia is 6%, HCT 21, bilirubin 7.8, and Cr 2.7. She is still making 60 mL of urine per hour. She rapidly becomes obtunded. Intensive care is initiated, with frequent Cr checks, close monitoring for hypoglycemia, infusion of phenobarbital for seizure ppx, mechanical vent for airway protection, and exchange transfusion to address her high parasitemia. Which is recommended as first-line treatment for her infection? A: chloroquine B: IV Artsenuate C: IV Quinine D: IV Quinidine E: Mefloquine

B artemisinin-containing regimens are now recommended by WHO as first line agents for falciparum malaria. In severe falciparum malaria, IV Artesunate reduced mortality by 35% compared to IV quinine. Artemether and Artemotil are given IM and are not as effective. Although safter and more effective than quinine, Artesunate is not available in the US. In the US, quinidine or quinine is used as a necessary 2nd choice. IV quinine is as effective and safer than IV quinidine, but is often not available in US hospital pharmaceuticals. Chloroquine is only effective for vivax and ovale infections and falciparum in certain pockets of the Middle East and Caribbean where resistance has not yet developed. Mefloquine comes only as oral formulation. It is MC used as prophylactic but is also used for MDR malaria.

Which type of bite represents a potential medical emergency in an asplenic patient? A: cat bite B: dog bite C: fish bite D: human bite

B cat bites are the MC animal bites which lead to cellulitis due to deep inoculation and the frequent presence of Pastueurella multicoda. In the immunocompetent host, only cat bites warrant empiric antibiotics. Often the first dose is given IV. Ampicillin/sulbactam followed by po amoxicillin/clavulanate is effective empirical therapy for cate bites. However, in an asplenic patient, a dog bite can lead to rapid overwhelming sepsis as a result of Capnocytophaga canimorsus bacteremia. These patients should be followed closely and given 3rd generation Cephalopsporins early in the course of infection. Empirical therapy should also be considered for dog bites in the elderly, for deep bites, and for bites on the hand.

56 YO M with hx of HTN and cigarette smoking is admitted to ICU after 1 week of fever and NP cough. Imaging shows a new pulmonary infiltrate, and urine antigen test is positive for Legionella. Each of these will be an effective ABX, except: A: Azithromycin B: Aztreonam C: Levofloxacin D: Tigecycline E: TMP/SMX

B despite ABX treatment, PNA from all causes remains a major source of mortality in the US. Mortality from Legionella varies from 0-11% in treated immunocompetent patients to ~30% in improperly treated patients. Because Legionella is intracellular, ABX that reach intracellular MICs are more likely to be effective. Newer macrolides and quinolone are ABX of choice and are as effective as monotherapy. Doxycycline and tigecycline are active in vitro. Anecdotal reports have described both success and failure with TMP/SMX and clindamycin. Aztreonam, most B-lactams, and Cephalosporins cannot be considered effective for Legionella PNA. For severe cases, Rifampin may be added to Azithromycin or a Fluroquinolone initially

9 YO M is brought to pediatric ER by his father. He has had 2 days of HA, neck stiffness, and photophobia and this morning has a T 38.9C (102F). He has also had several episodes of vomiting and diarrhea overnight. LP is performed, which reveals pleocytosis in the CSF. Which is true regarding enteroviruses as a cause of aseptic meningitis? A: elevated CSF protein rules out enteroviruses as cause B: enteroviruses are responsible for up to 90% of aseptic meningitis in children C: lymphocytes will predominate in the CSF early on, with a shift to PMNs at 24 hours D: SSX are more severe in children than adults E: they occur more commonly in winter and spring

B enteroviruses are responsible for up to 90% of aseptic meningitis in which an etiologic agent can be ID'd. SSX are typically more severe in adults than children. Illness is more frequent in the summer and fall in temperate climates, whereas other causes of viral meningitis are MC in winter and spring. CSF shows an elevated (usually <1000) WBC. Early, there may be PMNs predominance; however, this typically shifts toward lymphocyte predominance by 24 hours. CSF glucose and protein are usually normal, though the latter can sometimes be elevated. The illness is typically self-limiting and prognosis is excellent

the human enterovirus family includes poliovirus, coxsackievirus, enteroviruses, and echovirus. Which of the following statements regarding viral infections with one of the members of this group is true? A: among children infected with poliovirus, paralysis is common B: enteroviruses cannot be transmitted via blood transfusions and insect bites C: in utero exposure to maternal enteroviral antibodies is not protective D: infections are MC in adolescents and adults, though serious illness is MC in young children E: paralysis from poliovirus was more commonly seen in developing countries

B enteroviruses are ssRNA viruses that multiply in the GIT but rarely cause GI illness. Typical person-person spread occurs via the fecal:oral route; enterovirus are not known to spread via blood transfusions or insect vectors. Infection is MC among infants and small children; serious illness occurs in neonates, older children, and adults. Most infections with polio are symptomatic or cause a minor illness. Prior to the implementation of polio vaccines, paralysis was a rare clinical presentation of polio infection and was less frequent in developing countries, likely due to earlier exposure. Paralytic disease due to polio is more common in older adults, pregnant women, or persons exercising strenuously or with trauma at the time of CNS ssx. Exposure to maternal abs leads to lower risk of ssx neonatal infection

19 YO M plans on traveling through Central America by bus. He comes to clinic interested in travel advice and any vaccinations he may need. He has no medical hx and takes no meds. In addition to DEET and mosquito netting, which of the following recommendations would be important for ppx against malaria? A: atovaquone B: chloroquine C: doxycycline D: mefloquine E: primaquine

B malaria ppx recommendations vary by region. Currently the recommended malaria ppx for Central America is chloroquine. In contrast, due to resistance of falciparum malaria, ppx in India and most areas in Africa is with Atovaquone/Proguanil, Doxycycline, or Mefloquine.

Which of the following statements regarding liver abscesses is true? A: amebic liver abscesses should be r/o only by direct sampling and cx of pus B: ALP is the most likely LFT to be abnormal in the presence of the liver abscess C: Candida are most commonly isolated from patients with abscesses that develop as a result of peritoneal or pelvic pathology D: patients with liver abscesses nearly always have RUQ pain E: all are true

B microbiology data is critical in establishing the source of a liver abscess. Polymicrobial samples of pus or blood cx with gram negative rods, enterococcus, and anaerobes suggest an ABD or pelvic source. Hepatosplenic candidiasis once commonly occurred in leukemia or SCT patients not receiving anti fungal ppx. Fungemia was though to develop in the portal vasculature with poor CL of yeast during neutropenia. The rejuvenation of PMNs correlated with ssx of hepatic abscesses. Hepatosplenic candidiasis is now quite rare, given the widespread use of fluconazole ppx in patients with prolonged neutropenia. Certain spp such as Strep milleri or S aureus likely indicate a primary bacteremia and warrant a search for the source of this, depending on the typical ecologic niche of the organism. Amebic abscesses should be considered in the context of the host epidemiology; those with low-medium pretest prob based on travel hx, who also have a negative amebic serology, are effectively r/o for disease, w/o needing to sample the abscess percutaneously. Fever is the MC presenting ssx of liver abscess. Only 50% with abscess have RUQ pain, hepatomegaly, or jaundice. Therefore, half of patients may have no signs localizing to the liver. An elevated ALP is the most sensitive labs in liver abscess, present in ~70% of cases. Other LFT abnormal are less commonly.

Which of the following represents a rare but serious extra pulmonary cx of Influenza? A: diffuse eczematous rash B: myositis C: oligoarthritis D: purulent conjunctivitis E: secondary bacterial PNA caused by S aureus

B myositis and subsequent rhabdomyolysis and myoglobinuria represent a rare but severe cx of Influenza. Renal failure may occur. Myalgias are a prominent ssx of influenza infection, but myositis characterized by elevated CPK and marked tenderness of ms is very infrequent. The pathogenesis is unknown. Other cx include encephalitis, transverse myelitis, and GBS, but the etiologic relationship is uncertain. Myocarditis and pericarditis were reported during the 1918-1919 influenza pandemic. The most serious cx is secondary bacterial PNA, such as from s aureus. Arthritis, conjunctivitis, and eczematous rashes have not been described as cx.

47 YO F with known HIV/AIDS (CD4 106, viral load 35,000/mL) presents with painful growths on the side of her tongue. What is the most likely diagnosis? A: aphthous ulcers B: hairy leukoplakia C: herpes stomatitis D: oral candidiasis E: oral kaposi sarcoma

B oral hairy leukoplakia is due to severe overgrowth of EBV in T-cell def patients. It is not premalignant, is often unrecognized by the patient, but is sometimes a cosmetic, symptomatic, and therapeutic nuisance. The white thickened folds on the side of the tongue can be pruritic or painful and sometimes resolve with acyclovir derivatives or topical podophyllin resin. Ultimate resolution occurs after immune reconstitution with ART. Oral candida or thrush is very common, relatively easy to treat, and takes on the appearance of white plaques on the tongue, palate, and buccal mucosa that bleed with blunt removal. HSV recurrences or aphthous ulcers present as painful ulcerating lesions. The latter should be considered when oral ulcers persist, do not respond to acyclovir, and do not cx HSV. Kaposi sarcoma uncommon in the oropharynx and takes on a. violet hue, suggesting its highly vascularized content

What is the MC AE of oral Ribavirin when used with pegylated IFN for the treatment of HCV? A: drug-associated lupus B: hemolytic anemia C: hyperthyroidism D: leukopenia E: rash

B po Ribavirin combined with peg-IFN appears to be the most effective regimen for treated HCV. It does not exert antiviral effect but may be an immune modulator in combination with the IFN. Hemolytic anemia occurs in nearly 25% of patients receiving this therapy. Common approaches to this problem are dose reduction, cessation of treatment, or use of RBC growth factors. Rash can occur but is less common. IFN has common AE as well, including flulike ssx, depression, sleep disturbances, personality changes, leukopenia, and thrombocytopenia

64 YO F is admitted to the hospital with AMS. She recently returned from a summer white-water rafting trip in CO. Her husband reports increasing confusion, alternating lethargy and agitation, and visual hallucinations over the past 3 days. There is no hx of IVDA or psychiatric illness. She takes no meds. Her PE is notable for T 39 (102.2F), myoclonic jerks, and hyperreflexia. She is delirious and oriented to person only when aroused. There is no nuchal rigidity. CSF exam reveals clear fluid with WBC 15 with 100% lymphocytes, protein 1.0 g/L (100 mg/dL), and glucose 4.4 (80 mg/dl). Gram stain shows no organisms. You suspect infection with WNV. Which study will be most useful in making the diagnosis? A: CSF cx B: CSF IgM antibodies C: CNS MRI D: CSF PCR E: Stool cx

B since its introduction to US in 1999, WNV causes ~1000-3000 cases of encephalitis with 300 deaths per year. It is a Flavivirus of the same family as the causative agents of St Louis and Japanese encephalitis. Cases typically occur in the summer, often in community outbreaks, a/w dead crows. It is estimated that 1% of infections cause encephalitis, with the remainder being subclinical or having self-limited WNV fever. The elderly, DM, and patients with prior CNS disease are at a greater risk of encephalitis. WNV cannot be cx'd, and there is not yet a PCR test. IgM normally do not cross the BBB, and so their presence in the CSF is due to intrathecal production during acute infection with WNV. MRI is abnormal in only 30% of cases, significantly less often than is the case in HSV encephalitis. Stool cx may be useful in the diagnostic evaluation of enteroviral meningitis or encephalitis but not in cases of WNV

29 YO M is being initiated on HIV ART because of a rising viral RNA. He has no significant PMH or PsychHX and has never received ART. His viral resistance screening shows no likely resistance mutations. Which of the following is now considered an acceptable first-line regimen of ART for patients being newly treated for HIV who have no viral resistance and and no other medical or psychiatric problem? A: Stavudine (d4T)Z, didanosine (ddI), efavirenz (EFV) B: tenofovir (TDF), emtricitabine (FTC), efavirenz (EFV) C: tenofovir (TDF), emtricitabine (FTC), indinavir D: tenofovir (TDF), lopinavir/ritonavir, atazanavir E: zidovudine (AZT), lamivudine (3TC), abacavir (ABC)

B suppression of HIV replication with combination ART is the cornerstone of management of patients with HIV infection. There are several possible starting regimens for HIV therapy. Tenofovir, emtricitabine, and efavirenz is a popular combination as it is potent, reasonably free of AE in the long term, and is available in a single pill called Atripla. The combination of stavudine and didanosine (A) is now strictly contraindicated as there is a synergistic effect of mitochondrial toxicity resulting in cases of pancreatitis, neuropathy, and lactic acidosis. Indinavir, an older protease inhibitor, has an unacceptably high toxicity (nephrolithiasis) and too frequent dosing (TID) for the current standard of care. Choice D has (2) protease inhibitors, which is atypical and would not be first line. Choice E is available in a combination and taken BID (called Trizivir). It consists of (3) nucleoside analogues and has been a/w unacceptably high levels of treatment failure compared to standard of care.

79 YO Filipino-American male with DM, CAD, and emphysema develops the acute onset of lower back pain and night sweats. Ten days prior, he underwent a prolonged lithotripsy procedure for septic ureteral stones. He was treated for a positive PPD 23 years ago. He moved to the USA x 20 years ago and was rice farmer in the Philippines before moving. Exam reveals tenderness over the lumbar spine. He has 5/5 strength in the LE. MRI shows osteomyelitis of L3 and L4, with narrowing of the disc space and a small contiguous epidural abscess that is not compressing his spinal cord. A needle cx of the epidural abscess drawn prior to administration of ABX will most likely reveal: A: Brucella melitensis B: E coli C: MTB D: S aureus E: Polymicrobial content with gram-positive cocci in chains, enteric gram-negative rods, and anaerobic pleomorphic forms

B the MC overall cause of acute bacterial osteomyelitis is S aureus, accounting for ~50% of cases due to a single organism, introduced via the bloodstream in patients at risk for bloodstream infections (IVDA, HD patients, open post-op wounds). However, in an older male patient with lumbar osteomyelitis, GU or Enteric pathogens such as E coli are common, particularly after recent UTI and/or urologic surgeries, accounting for up to 25% of cases of vertebral osteomyelitis. Pathogenesis may occur via retrograde induction of organism into the spine venous plexus. Polymicrobial osteomyelitis is most often due to contiguous infection, such as decubitus ulcer or diabetic foot infection, rather than bloodstream introductions that are more typical in the spine. TB (Potts disease) is always a consideration for osteomyelitis of the spine. However, the patient's presentation is likely too acute for TB, and the thoracic spine is slightly more typical location than the lumbar spine. Brucellosis commonly involves the spine, but this patient's potential exposure to Brucella is dated and the course of the infection is too acute for Brucellosis. Hypothetically each of the listed infections is possible, highlighting the importance of hold abx before cx the epidural space, provided that the patient does not have sepsis on original presentation

All of the following individuals receiving TB skin PPD reactions should be treated for latent TB, except: A: 23 YO IVDA who is HIV negative, with 12-mm PPD B: 38 YO 4th grade teacher has a 7-mm PPD and no known exposure to active TB. She has never been PPD tested before. C: 43 YO individual in the Peace Corps working in Sub-Saharan Africa has a 10-mm reaction. 18 months ago, it was 3 mm. D: 55 YO HIV positive M has a negative PPD. His partner is recently diagnosed with cavitary TB E: 72 YO M who is receiving CTX for NHL has a 16-mm PPD

B the aim of treatment of latent TB is to prevent the development of active disease, and the PPD is the MC means of ID'ing cases of latent TB in high-risk groups. To perform a PPD, 5 TB units of PPD are placed SQ in the forearm. The degree of induration is determined at 48-72 hours. Erythema only does not count as a positive reaction to PPD. The size of the reaction determines whether individuals should receive treatment for latent TB. In general, individuals in low-risk groups should not be tested. However, if tested, a reaction >15 mm is required to be positive. School teachers are low risk. Thus a reaction of 7 mm is not positive. A size of >=10 mm is considered positive in individuals who have been infected within 2 years or those with high risk medical conditions. The individual working in an area where TB is endemic has tested newly positive by skin testing and should be treated as a newly infected individual. High risk medical conditions for which treatment of latent TB is recommended include DM, IVDA, ESRD, rapid weight loss, and heme disorders. PPD >=5 mm are considered positive for latent TB in individuals with fibrotic lesions on CXR, those with close contact with an infected person, and those with HIV or who are otherwise immunocompromised. There are (2) situations in which treatment for latent TB is recommended regardless of the induration. First, infants and children who have had close contact with actively infected persons should be treated. After 2 months of treatment, a skin test should be performed. Treatment can be d/c'd if the skin test remains negative at that time. Also, individuals who are HIV positive and have had close contact with infected person should be treated regardless of skin test results.

18 YO M presents with a firm, nontender lesions around his anal orifice. The lesion is about 1.5-cm in diameter and has a cartilaginous feel on exam. The patient reports that it has progressed to this stage from a small papule. It is not tender. He reports recent unprotected anal intercourse. Bacterial cx of the lesion is negative. A RPR test is also negative. Therapeutic interventions should include: A: IM Ceftriaxone, 1 g B: IM PCN G benzathine, 2.4 M U C: po Acyclovir, 200 mg 5 times per day D: observation E: surgical resection with biopsy

B the patient's clinical exam is c/w primary syphilis and he should receive appropriate therapy. In primary syphilis, 25% of patients will have a negative nontreponemal test for syphilis (RPR or VDRL). A single dose of long-acting PCN G is the recommended treatment for primary, secondary, and early latent. Ceftriaxone is the treatment for gonorrhea, but the lesion is not c/w this dx. Ceftriaxone given daily for 7-10 days is an alternative treatment for primary and secondary. Acyclovir is the DOC for genital herpes. Herpetic lesions are classically multiple and painful. Observation is not an option because the chance it will resolve spontaneously w/o treatment is high and the patient will remain infected and infectious.

Patients with which of the following have the lowest risk of invasive pulmonary Aspergillus infection? A: allogeneic stem cell transplant with GVHD B: HIV infection C: long-standing high-dose glucocorticoids D: post-solid organ transplant with multiple episodes of rejection E: relapsed/uncontrolled leukemia

B the primary risk factor for invasive Aspergillus infection is neutropenia and glucocorticoid use. Risk is proportional to the degree and length of neutropenia and dose of GC. HIV patients rarely develop invasive aspergillosis, and if they do, it is in the context of prolonged neutropenia and/or advanced disease. Patients with GVHD and uncontrolled leukemia are at particularly elevated risk. The infection is seen in solid organ transplant patients, particularly those requiring high cumulative doses of GC for graft rejection.

Which of the following is true regarding influenza ppx? A: patients receiving an IM influenza vaccine should be warned of the increased risk of GBS B: patients with HSN to eggs should not receive the IM vaccine C: the IM influenza vaccine is a live, attenuated strain of influenza that is based on isolates from the previous year's strain of A and B D: the IM influenza vaccine should not be given to immunocompromised host E: the intranasal spray, "flu-mist" is an inactivated virus prep based on the previous year's strains of A and B

B there are inactivated and live, attenuated forms of influenza. The intranasal spray, marketed as "flu mist", is a live, attenuated virus and is not recommended for the elderly or immunocompromised. This vaccine has similar efficacy to the IM vaccine, which is an inactivated, or "killed" prep of the previous year's strains of A and B. The IM is manufactured using egg products; patients with true egg HSN should not receive it. It is safe for elderly or immunocompromised patients. In the past, influenza vaccines have been a/w GBS. This association has not been demonstrated in the past decade, despite close surveillance. Patients do not need to be warned of this AE.

25 YO F presents with 1 day of fever to 38.3C (101 F), sore throat, dysphagia, and a number of grayish-white papulovesicular lesions on the soft palate, uvula, and anterior pillars of the tonsils. The patient is likely infected with: A: Candida B: Coxsackie virus C: Herpesvirus D: HIV E: Staph lugdunensis

B these lesions are diagnostic of herpangina, which is caused by Coxsackie A. They are typically round and discrete, which helps ddx them from thrush. Unlike HSV stomatitis, herpangina lesions are not a/w gingivitis. Herpangina usually presents with dysphagia, odynophagia, and fever; these lesions can persist for several weeks. The do not ulcerate.

34 YO M IVDA presents with 2-day hx of slurred speech, blurry vision that is worse with b/l gaze deviation, dry mouth, and difficulty swallowing both liquids and solids. He states that his arms feel weak as well but denies any sensory deficits. He has had no recent illness but does describe a chronic ulcer on his L-LE that has felt slightly warm and tender of late. He frequently injects heroin into the edges of the ulcer. On ROS, he reports mild SOB, but denies any GIT ssx, urinary retention, or loss of bowel/bladder functions. PE reveals a frustrated, nontoxic appearing man who is alert and oriented but noticeably dysarthric. He is afebrile and stable VS. CN exam reveals b/l CN VI deficits and an inability to maintain medial gaze in both eyes. He has mild b/l ptosis, and both pupils are reactive but sluggish. His strength is 5/5 in all extremities except for his shoulder shrug, which is 4/5. Sensory exam and DTRs are within normal limits in all 4 extremities. His oropharynx is dry. Cardio and ABD exams are wnl. He has a 4 cm x 5 cm well-granulated LE ulcer with redness, warmth, and erythema noted on the upper margin of the ulcer. What is the treatment of choice? A: glucocorticoids B: equine antitoxin to C botulinum neurotoxin C: IV-Heparin D: Naltrexone E: plasmapheresis

B this patient most likely has wound botulism. The use of "black-tar" heroin has been ID'd as a form of botulism. Typically the wound appears benign, and unlike in other forms of botulism, GIT ssx are absent. Symmetric descending paralysis suggests botulism, as does CN involvement. This patient's ptosis, diplopia, dysarthria, dysphagia, lack of fevers, normal reflexes, and lack of sensory deficits are suggestive. Botulism can be easily confused with GBS, which is often characterized by an antecedent infection and rapid, symmetric ascending paralysis and treated with plasmapheresis. The miller Fischer variant of GBS is known for CN involvement with ophthalmoplegia, ataxia, and areflexia being the most prominent features. Elevated protein in the CSF also favors GBS over botulism. Both botulism and GBS can progress to respiratory failure, so making a dx by PE is critical. Other dx modalities that may be helpful are wound cx, serum assay for toxin, and exam for decreased compound ms APs on routine nerve stimulation. Patients are at risk of respiratory failure due to respiratory ms weakness or aspiration. They should be followed closely with O2 sat monitoring and serial measurements of FVC.

34 YO M seeks the advice of his PCP because of an asymptomatic rash on his chest. There are coalescing light brown to salmon colored macules present on his chest. A scraping of the lesion is viewed after a wet prep with 10% KOH. There are both hyphal and spore forms present, giving the appearance of "spaghetti and meatballs". In addition, the lesions fluoresce to yellow-green under a Wood's lamp. Tinea versicolor is dx. Which of the following microorganisms is responsible for this infection? A: Fusarium solani B: Malessezia furfur C: Sporothrix schenkii D: Trichophyton rubrum

B tinea versicolor is the MC superficial skin infection. It is caused by lipophilic yeasts of the genus Malassezia, most commonly, M furor. In tropical areas, the prevalence is 40-60%, whereas in temperate areas it is about 1%. In general, most individuals seek evaluation for cosmetic reasons as the lesions are asymptomatic or only mild pruritic. The lesions typically appear as patches of pink or copper-brown skin, but the areas may be hypo pigmented in dark skinned individuals. Dx can be made by demonstration of the organism on KOH prep, where a typical spaghetti and meatballs appearance can be seen. This is due to the presence of both spore forms and hyphal forms within the skin. Under a long wave UVA light (Woods lamp), the affected areas fluoresce to yellow-green. The organism is sensitive to a variety of antifungals. Selenium sulfide shampoo, topical azaleas, terbinafine, and ciclopirox have all been used with success. A 2-week treatment regimen typically shows good results, but the infection typically recurs within 2 years of treatment.

72 YO M is admitted to the hospital with bacteremia and pyelonephritis. He is HIV-negative and has no other PMH. Two weeks into his treatment with ABX for a fever evaluation reveals a blood cx positive for Candida. Exam is wnl. WBC is wnl. The central line is removed, and systemic anti fungal agents are initiated. What further evaluation is recommended? A: ABD CT scan to look for abscess B: CXR C: Fundoscopy D: Repeat blood cx E: TTE

C Candidemia may lead to seeding of other organs. Among non-neutropenic patients up to 10% develop retinal lesions; therefore, it is very important to perform thorough fundoscopy. Focal seeding can occur within 2 weeks of the onset of candidemia and may occur even if the patient is afebrile or infection clears. The lesions may be u/l or b/l and are typically small white retinal exudates. However, retinal infection may progress to retinal detachment, vitreous abscess, or extension into the anterior chamber of the eye. Patients may be asymptomatic initially but may also report blurring, ocular pain, or scotoma. ABD abscesses are possible but usually occur in patients recovering from profound neutropenia. Fungal endocarditis is also possible but is more common in IVDA and may have murmur on CVS exam. Fungal PNA and pulmonary abscess are very rare and not likely in this patient

28 YO M is diagnosed with HIV during a clinic visit. He has no ssx of opportunistic infection. His CD4 is 150. All of these are approved regimens for primary prophylaxis against PCP infection, except: A: aerosolized pentamidine, 300 mg monthly B: atovaquone, 1500 mg po daily C: clindamycin, 900 mg po q8h, plus primaquine, 30 mg po daily D: dapsone, 100 mg po daily E: TMP/SMX, 1 single-strength tablet po daily

C Clindamycin plus primaquine is a therapeutic, not prophylactic, regimen for mild-moderate disease due to PCP. TMP/SMX is usually given as a first line agent but carries a significant AE profile including hyperkalemia, renal insufficiency, elevation of serum Cr, granulocytopenia, hemolysis in G6PD def, and frequent allergic reactions, particularly in those with T cell def. Atovquone is a common alternative that is given at the same dose for PCP as for therapy. GIT ssx are common. Aerosolized pentamidine can be given on a monthly basis with a risk of bronchospasm and pancreatitis. Patients who develop PCP while receiving aerosolized pentamidine often have upper-lobe predominant disease. Dapsone is commonly used for PCP ppx; however, the physician should be aware of methemoglobinemia, G6PD hemolysis, rare hepatotoxicity, and rare HSN reaction

You are a physician for an undergrad health clinic in AZ. You evaluate 3 students with similar cx of fever, malaise, diffuse arthralgia, cough w/o hemoptysis, and chest discomfort, and one of the patients has a skin rash on her extremities c/w erythema multiforme. CXR is similar in all three, with hilar LAD and small pleural effusions. Upon further questioning you learn that all three are in the same archaeology class and participated in an excavation 1 week ago. Your leading dx is : A: mononucleosis B: primary pulmonary aspergillosis C: primary pulmonary coccidioidomycosis D: primary pulmonary histoplasmosis E: streptococcal pneumonia

C Coccidioides immitis is a mold that is found in the soil in the SW US and Mexico. Case clusters of primary disease may appear 10-14 days after exposure, and the activities with the highest risk include archaeologic excavation, rock hunting, military maneuvers, and construction work. Only 40% of primary pulmonary infections are symptomatic. SSX may include those of HSN reaction such as erythema nodosum, erythema multiforme, arthritis, or conjunctivitis. Dx can be made by cx of sputum; however, when the organism is suspected, the lab needs to be notified as it is a biohazard level 3. Serology may be helpful; however, seroconversion of primary disease may take up to 8 weeks. Skin testing is useful for epidemiology but not done for clinical practice

The MC cause of traveler's diarrhea in Mexico is: A: Campylobacter jejuni B: Entamoeba histolytica C: ETEC D: Giardia E: Vibrio cholerae

C ETEC is responsible for 50% of traveler's diarrhea in Latin America and 15% in Asia. ETEC and Enteroaggregative E coli are the MC isolates from persons with classic secretory traveler's diarrhea. Treatment of frequent watery stools due to presumed E coli infection may be with Ciprofloxacin, or because of concerns of increasing resistance to Cipro, Azithromycin. E histolytica and Cholera account for smaller % of traveler's diarrhea in Mexico. Campy is MC in Asian and during the winter in subtropical areas. Giardia is a/w contaminated water supplies and in campers who drink from freshwater streams.

A previously unvaccinated health care worker incurs a needle stick from a patient with known active HBV. What is the appropriate management for the worker? A: HBV immunoglobulin B: HBV vaccine C: HBV vaccine plus HBV immunoglobulins D: HBV vaccine plus Lamivudine E: Lamivudine plus Tenofovir

C HBV is efficient spread as a blood born pathogen. In approximately 1/3 of needle stick cases where the victim is not immunized (either by vaccine or previous clearance), HBV transmission will occur. This is in comparison to 3% for HCV and 0.3% for HIV-1. Moreover, HBV, because it is a DNA virus, can survive for a prolonged amount of time on sterilized surfaces. This speaks to the goal of 100% vaccination for all healthcare workers. Rapid administration of both HBV vaccine and immunoglobulins are the most effective way to prevent transmission if a high-risk stick occurs to a non immune health care worker. No data exist to support the use of antivirals for HBV needle sticks, though this strategy has proven effective for HIV-1 associated needle sticks.

All of the following are examples of an indication for checking for HIV-resistance genotype, except: A: 23 YO M with new diagnosis of HIV B: 34 YO M with HIV-1 was started on ART (TDF, FTC, EFV) 1 month ago. At that time CD4 213 and HIV-1 viral load 65,000. On recheck 1 month later, viral load is 37,000. He states he takes his meds 100% of the time C: 42 YO M with HIV/AIDS started on ART (TDF, FTC, and ritonavir-boosted Atazanavir) 1 year ago was LTFU. Originally HIV-1 viral load 197,000 and CD4 11. He was 100% compliant until he ran out of meds 2 months ago. Viral load recheck is 184,000 with CD4 138. D: 52 YO F who has had full viral suppression (viral load <30) and 100% medical compliance on ART (AZT, 3TC, EFV) for 2 years has relapsed on IV heroin over the past 3 months. She states she continued with her ART with a "few missed doses here and there". Repeat viral load is 3800 and CD4 is stable at 413

C HIV resistance testing is recommended in selecting initial ART where the prevalence of resistance is high (US and Europe) and in determining new therapy for patients with virologic failure while on ART. In the US, the predominant virus in up to 12% of new cases has one major genotypic resistance mutation (patient A). In the patient failing ART, a resistance genotype should be performed while the patient is on therapy. In the absence of ART, the majority of virus reverts to WT and the genotype appears normal (genotypes only sample the dominant viral form, though many exist); however, archived viruses in latent pools that are not accessible with current available assays may in fact harbor resistance. Therefore a genotype for patient C is likely to be of little value. Following the initiation of therapy the patient should have 1 log (tenfold) reduction in plasma HIV RNA levels within 1-2 months. Failure to achieve this response (patient B) may warrant a change in therapy. Patient D has breakthrough failure after a period of intermittent compliance. To determine if she has developed a new resistance pattern, she should have a genotype performed while on therapy to allow for adequate selection pressure from the antiviral agents to select the resistance virus leading to failure as the dominant strain

All of the following statements regarding human T cell lymphotropic virus 1 (HTLV-1) are true, except: A: acute T cell leukemia is a/w HTLV-1 infection B: HTLV-1 endemic regions include S Japan, the Caribbean, and S America C: HTLV-1 infection is a/w a gradual decline in T cell function and immunosuppression D: HTLV-1 is transmitted parenterally, sexually, and from mother to child E: tropical spastic parapesis is a/w HTLV-1 infection

C HTLV1 is a retrovirus that is a chronic infection like HIV, but does not cause similar sequelae. It was the first ID'd human retrovirus. Gradual decline of CD4 lymphocyte number and function is a feature of HIV, but not HTLV-1. While many people in endemic areas have serologic evidence of infection, most do not develop disease. The two major cx of HTLV-1 are tropical spastic paraparesis and acute T cell leukemia. Tropical spastic paraparesis is an UMN disease of insidious onset leading to weakness, LE stiffness, urinary incontinence, and eventually a thoracic myopathy which leads to a bedridden state in about 1/3 of patients after 10 years. It is more common in women than men. It can be easily confused with MS; this is why it is important to recall the geographic regions where HTLV-1 is endemic when evaluating myopathy. Acute T cell leukemia is a difficult to treat leukemia that is specific to chronic HTLV-1 infection. It is thought to be transmitted in a similar fashion to HIV

32 YO M presents with jaundice and malaise. He is found to have acute HBV with positive HBV DNA and E antigen. Which of the following antiviral agents are approved as part of the therapeutic regimen for mono-infection with HBV? A: Efavirenz B: Ganciclovir C: Lamivudine D: Rimantadine E: Tenofovir

C Lamivudine is a pyrimidine nucleoside analogue that has activity against HIV and HBV. In acute HBV, it results in suppression of HBV DNA and loss of E antigen in 30% of patients. Tenofovir is a nucleotide analogue with activity against HIV and HBV. It is not approved for initial treatment, but may be used for co-infection or for HBV resistant to Lamivudine. Efavirenz has activity only against HIV and not HBV. Ganciclovir has activity against herpes viruses and is mostly used for treatment of CMV. Rimantadine has antiviral activity only against Influenza

All of these are RF for developing Legionella PNA, except: A: glucocorticoids use B: HIV infection C: neutropenia D: recent surgery E: tobacco use

C Legionella is an intracellular pathogen which enters the body through aspiration or direct inhalation. Numerous studies have found it is one of the four most common causes of CA-PNA with Strep pneumoniae, Haemophilus influenzae, and Chlamydia pneumoniae accounting for 2-9% of cases. Post-op patients are at risk due to increased risk of aspiration. Cell-mediated immunity is the primary host defense against Legionella, and patients with HIV or those on glucocorticoids are at risk based on their depressed cell mediated immune function. Alveolar macrophages phagocytose Legionella. Smokers and those with CLD are at risk given their poor local immune response and decreased ability for widespread phagocytosis. Neutrophils play a comparatively small role in the host defense against Legionella, and those with neutropenia are not predisposed to Legionella infection

All of the following factors influence the likelihood of transmitting active TB, except: A: duration of contact with an infected person B: environment in which contact occurs C: presence of extra pulmonary TB D: presence of laryngeal TB E: probability of contact with an infectious person

C TB is most commonly transmitted from person-person via airborne droplets. Factors that affect likelihood of developing TB infection include the probability of contact with an infectious person, the intimacy and duration of the contact, the degree of infectiousness of the contact, and the environment in which the contact takes place. The most infectious patients are those with cavitary pulmonary or laryngeal TB with about 10^5 - 10^7 TB bacteria per mL of sputum. Individuals who have a negative AFB smear with a positive cx for TB are less infectious but may transmit the disease. However, individuals with only extra pulmonary (e.g., renal, skeletal) TB are considered non-infectious

Which of the following favors a diagnosis of acute bacterial epididymitis? A: a solid NT testicular mass B: absence of blood flow on Doppler US C: concurrent urethral d/c D: elevation of the testicle within the scrotal sac E: lack of response to Ceftriaxone plus Doxycycline therapy

C acute epididymitis almost always causes u/l painful swelling of the epididymis. In young men, it is usually an extension of a primary STI, and urethral d/c is therefore very suggestive of the dx. The ddx includes testicular torsion, which is a surgical emergency. An elevated testicle and lack of blood flow on Doppler study suggest this dx. Testicular cancer, unlike epididymitis, does not usually cause tenderness and pain. This is an important consideration in any male with a testicular mass. Response to abx should suggest bacterial epididymitis, rather than r/o

26 YO F college student presents with tender epitrochlear and axillary tender, firm, 3-cm LN on her L-side. She has a 0.5-cm painless nodule on her L-2nd finger. She reports low-grade fever and malaise x2 weeks. She enjoys gardening, exotic fish collection, and owns several pets including fish, kittens, and a puppy. She is sexually active with 1 partner. She traveled extensively throughout rural SE Asia 2 years before her illness. The ddx should include all of these, except: A: Bartonella henselae B: lymphoma C: sporothrix schenkii infection D: Staphylococcal infection

C although the patient's gardening puts her at risk for Sporothrix infection, this infection typically causes a more localized streaking nodular lymphadenitis affecting the forearm. The ddx for nodular adenitis includes Sporothrix, Nocardia, MAC, Leishmania, and Francisella and is based on direct inoculation of organism due to contact from the soil, marine, insect bite, or animal bite. This patient has regional lymphadenitis involving larger LN that drain the site of inoculation. Most likely in her case is cat scratch disease due to Bartonella, based on the kittens in her home, but lymphoma and Staphylococcal infection must also be considered and often a LN bx is required to make the distinction. Most cases of cat scratch fever will resolve w/o treatment. In immunocompetent patients, ABX have minimal benefit but may expedite resolution of LAD. Antimicrobial treatment, usually with Azithromycin, is indicated in immunosuppressed patients.

Which of the following is not a common feature of severe falciparum malaria? A: acute tubular necrosis B: HCT <15% C: hepatic necrosis D: hypoglycemia E: obtundation

C appropriately and promptly treated, falciparum malaria w/o cx has a MR of ~0.1%. The presence or development of organ dysfunction elevates mortality risk significantly. Hypoglycemia is a/w poor prognosis and is MC in children and pregnant women. It is caused by a failure of hepatic gluconeogenesis and increased glucose consumption by the host. Quinine and quinidine may also increase pancreatic insulin secretion. Obtundation and coma are worrisome in that they often represent cerebral edema, a feared cx of cerebral malaria. The MR of cerebral malaria is 15-20%. AKI and profound hemolytic anemia often accompany severe infection and are also poor prognostic features. Their presence may necessitate hemodialysis and exchange transfusion. Though liver enzyme elevations and hemolytic jaundice are common in malaria, hepatic necrosis is not. When severe live dysfunction occurs, it is usually in the context of MOF

regarding the epidemiology of influenza viruses, which is true? A: antigenic drift requires a change in both hemagglutinin (H) and neuraminidase (N) antigens B: antigenic shift is defined by an exchange of hemagglutinin (H) and neuraminidase (N) antigens between influenza A and B C: avian influenza outbreaks in humans occur when influenza A viruses undergo antigenic shifts with influenza A from poultry D: influenza C virus infections, while uncommon, are more virulent on a population basis due to its increased ability to undergo antigenic shift E: the lethality a/w avian influenza is related to its ability to spread via person-person

C avian influenza epidemics occur when human influenza A undergoes an antigenic exchange with influenza found in poultry. Recent outbreaks have not been a/w effective human-human spread; nearly all patients reported exposure to infected poultry. Past influenza pandemics, including the 1918-19 pandemic, appear to have originated from antigenic exchange between human and avian influenza viruses. Antigenic shifts are defined as major changes in the H and N antigens and occur only with Flu A. Minor antigenic changes are known as drifts and can occur with H alone or with both H and N. While Flu A and B are genetically and morphologically similar, the latter virus' inability to undergo shifts lessens its virulence and involvement in pandemic flu. Influenza C is a rare cause of disease in humans and is typically a mild, self-limited infection

40 YO M is admitted for 2-3 weeks of fever, tender LAD, and RUQ ABD pain. He reports progressive weight loss and malaise x1 year. On exam, he is found to be febrile and frail with temporal wasting and oral thrush. Matted, tender anterior cervical LAD <1cm and tender hepatomegaly are noted. He is dx with AIDS (CD4 12, HIV RNA 650,000/mL). Blood cx grow MAC. He is started on Rifabutin and Clarithromycin, as well as Dapsone for PCP ppx, and d/c home 2 weeks later after his fevers subside. He follows up with an HIV provider 4 weeks later and is started on Tenofovir, Emtricitabine, and Efavirenz. Two weeks later he returns to clinic with fevers, neck pain, and ABD pain. His T 39.2C, HR 110, BP 110/64, and O2 sats wnl. His cervical nodes are now 2-cm in size and extremely tender, and one has fistulized to his skin and is draining yellow pus that is AFB stain positive. What is the most likely explanation for this? A: Cryptococcal meningitis B: HIV treatment failure C: Immune reconstitution syndrome to MAC D: Kaposi sarcoma E: MAC treatment failure due to drug resistance

C immune reconstitution syndrome (IRIS) is commonly seen after the initiation of ART in patient with AIDS and a concomitant opportunistic infection (OI). It is syndrome where either a previously recognized OI worsens after ART despite an initial period of improvement after standard therapy, or when an OI that was not previously recognized is unmasked after ART. The latter occurs presumably as immune cells become reactivated and recognize the presence of a pathogen that disseminated in the absence of adequate T cell response with the patient remaining subclinical prior to ART. Many opportunistic pathogens are known to behave this way but Cryptococcus, Mycobacterium tb, and MAC are the most likely to be a/w IRIS. RF for IRIS are low CD4 at time of ART initiation, initiation of ART within 2 months of treatment for OI, adequate virologic response to ART, and increase in CD4 lymphocyte count as a result of ART. IRIS can be diagnostically challenging and is very diverse in terms of clinical presentation and severity. Depending on the organ system and pathogen, drug resistant OI and new oI must be considered, sometimes necessitating invasive bx and cx. In this case, the overlap of organ system with the original presentation, low likelihood of MAC drug resistance, and timing of the syndrome favor IRIS. Therapy is with NSAIDs and sometimes glucocorticoids. OI treatment is continued, and all efforts are made to continue ART as well, except under dire circumstance

30 YO healthy woman presents with severe SOB, confusion, productive cough, and fevers. She has been ill 1 week ago with a flulike illness characterized by fever, myalgia, HA, and malaise. Her illness almost entirely improved w/o medical intervention until 36 hours ago, when she developed new rigors followed by progression of her respiratory ssx. On exam, her T 39.6C, HR 130, BP 95/60, RR 40, and O2 sat 88% on 100% face mask. On exam, she is clammy, confused, and very SOB. Lung exams reveals amphoric breath sounds over her L lower lung fields. She is intubated and resuscitated with fluid and abx. Chest CT shows necrosis of the LL lobe. Blood and sputum cx grow S aureus. This isolate is likely resistant to which abx? A: Doxycycline B: Linezolid C: Methicillin D: TMP/SMX E: Vancomycin

C in recent years, the emergence of "community acquired" methicillin-resistant staph aureus (MRSA) in numerous populations has been well documented. This pathogen MC leads to pyogenic infection of the skin but has also been a/w necrotizing fasciitis, infectious polymyositis, endocarditis, and osteomyelitis. The most feared cx is a necrotizing PNA that often follows influenza URI and can affect previously healthy people. This pathogen produces the Panton-Valentine leukocidin protein that forms holes in the membranes of the PMNs as they arrive at the site of infection, and serves as a marker for this pathogen. An easy way to ID this strain of MRSA is its sensitivity profile. Unlike MRSA isolates of the past, which were sensitive only to Vancomycin, Daptomycin, Quinupristin/Dalfopristin, and Linezolid, CA_MRSA are almost uniformly susceptible to TMP/SMX and Doxycycline as well. The organism is also usually sensitive to Clindamycin. The term community-acquired has probably outlived its usefulness as this isolate has become the MC S aureus isolate causing infections in hospitals worldwide.

Which of the following is the MC manifestation of initial (primary) herpes simplex virus (HSV-1) infection? A: asymptomatic infection B: genital ulcers C: gingivostomatitis and pharyngitis D: orolabial ulcers E: trigeminal neuralgia

C infection with HSV-1 is acquired more frequently and earlier than with HSV-2. More than 90% of adults have serologic evidence of hSV-1 infection by 40. Primary HSV-1 usually causes pharyngitis and gingivostomatitis and MC occurs in children and young adults. Clinical manifestations of primary infection include fever, malaise, myalgia, and adenopathy. There are usually curative or ulcerative lesions of the pharynx or tonsils. It may be difficult to ddx from bacterial pharyngitis. Herpes labials is the MC manifestation of hSV-1 recurrence. Pharyngitis is uncommon with recurrence. HSV 1 resides in latency in the trigeminal ganglia, but trigeminal neuralgia is not a feature of primary infection. HSV 2 recurrence involves genital ulcers

All of these increase the risk for C diff, except: A: antacids B: antecedent ABX C: C diff colonization D: enteral tube feeds E: increasing LOS F: older age

C interestingly, a number of studies have found that colonization is NOT a risk factor for disease. This may be because strains that are apt to colonize may provide some immunity to the host or are less toxigenic than disease causing strains. In either case, this serves as a reminder that stool testing should be conducted only on symptomatic patients, as a positive test carries a totally different meaning if clinical suspicion is low. Patients should not be considered to have C diff based on cx alone. Additionally info to make a diagnosis in a patient with the appropriate clinical findings include demonstrating presence of Toxin A and B or demonstration of pseudomembranes at colonoscopy. RF for C diff disease are well-defined: the most important is antecedent abx, especially fluroquinolones, cephalosporins, and clindamycin. Age, high patient acuity, enteral feedings, antacids, and LOS in a health care facility are also predictive

41 YO M with HCV-associated ascites presents with acute ABD pain. PE is notable for T 38.3C, HR 115, BP 88/48, RR 16, and O2 sat 99% on RA. The patient is in moderate discomfort and is lying still. He is alert and oriented. Lungs are clear. Cardiac exam is unremarkable. His abdomen is diffusely tender with distant bowel sounds, mild guarding, and no rebound. Labs reveal WBC 11.6 with 94% PMNs, HCT 29, PLT 24. Paracentesis reveals 658 PMNs/uL, total protein 1.2, glucose 24, and gram stain showing gram-negative rods, gram-positive cocci in chains, gram-positive rods, and yeast forms. All of these are indicated, except: A: ABD XR B: broad-spectrum ABX C: Drotreocogin alfa D: IVF E: surgical consult

C it is important to distinguish between primary (spontaneous) and secondary peritonitis. Primary is a result of longstanding ascites, usually as a result of cirrhosis. The pathogenesis is poorly understood but may involve bacteremic spread or translocation across the gut wall of usually only a single spp of pathogenic bacteria. Secondary peritonitis is due to rupture of a hollow viscous or irritation of the peritoneum due to a contiguous abscess or pyogenic infection. It typically presents with peritoneal signs and in most cases represents a surgical emergency. Secondary peritonitis in a cirrhotic patient is difficult to distinguish on clinical grounds from primary (spontaneous) peritonitis. It is often overlooked because classic peritoneal signs are almost always lacking, and it is uniformly fatal in the absence of surgery. Suspicion for this dx should occur when ascites shows a protein >1 g/dl, LDH > serum LDH, glucose <50, and/or polymicrobial gram stain. Once this diagnosis is suspected, an ABD XR is indicated to r/o free air, and prompt surgical consultation is warranted. Unlike with primary (spontaneous) bacterial peritonitis, in cases of secondary peritonitis abx should include anaerobic coverage and often anti fungal agents. This patient requires IVF as he has hypotension and tachycardia due to sepsis. Drotrecogin alfa has been shown to reduce mortality in patients with sepsis; however, patients with thrombocytopenia, cirrhosis, and ascites were excluded from inclusion in phase III trials.

34 YO M recent immigrant from Burundi presents with fever, HA, severe myalgia, photophobia, conjunctival injection, and prostration. He lived in a refugee camp for 10 years. In the camp, he was treated for several unknown febrile illnesses. Since arriving in the US 7 years ago, he has worked as a computer analyst and lived only in a metropolitan NW city with no significant travel. Initial blood cx are negative. Five days into the illness he develops hypotension, pneumonitis, encephalopathy, and gangrene of his distal digits as well as petechial, hemorrhagic rash over his entire body except for his face. A bx of his rash reveals IHC changes c/w Rickettsial infection. Which pathogen is most likely? A: Coxiella burnetii (Q fever) B: Rickettsia Africae (African tick borne fever) C: Rickettsia prowazekii (louse-born typhus) D: Rickettsia rickettsia (RM Spotted fever) E: Rickettsia typhi (Murine typhus)

C only two rickettsial infections (Prowazekii and Coxiella burnetii) have a recrudescent or chronic stage. This patient has louse-borne (epidemic) typhus caused by R prowazekii. Louse-borne typhus occurs most commonly in outbreaks in overcrowded, poorly hygienic areas such as refugee camps. There was an outbreak of ~100,000 people living in refugee camps in Burundi in 1997. It is the second most severe form of Rickettsial disease and can recur years after acute infection, as in this patient. This is thought to occur as a result of waning immunity. RM spotted fever would be c/w this patient's presentation but he has no RF for this disease. African tick borne fever is considerably less severe and is often a/w a black eschar at the site of tick bite. Murine typhus is usually less severe and does not exist in a recrudescent form. Q fever can cause chronic disease but this is almost always in the form of endocarditis

52 YO F with ETOH cirrhosis, portal HTN, esophageal varices, and hx of hepatic encephalopathy presents with confusion over several days. Her husband remarkers that the patient has been adherent to her meds. These meds include Labetalol, furosemide, aldactone, and lactulose. PE is notable for T 38.3C, HR 115, BP 10/562, RR 12, and O2 sat 96% on RA. The patient is extremely drowsy, only intermittently able to answer questions, and disoriented. She has slight asterixis. Lungs are clear. CVS is wnl. Her ABD is distended and tense, but NT. She has 3+ LE edema extending to the thighs. She is guaiac negative. Her CN and extremity strength are symmetric and wnl. Labs show WBC 4.8, HCT 33 (baseline 30), and PLT 94. BMP is wnl. What is an essential component of the workup? A: CT head B: EGD C: Paracentesis D: Therapeutic trial of lactulose E: Serum ammonia level

C primary bacterial peritonitis is a cx of ascites a/w cirrhosis. Clinical presentation can be misleading as only 80% of patients have fever, and ABD ssx are only variably present. Therefore, when patients with known cirrhosis develop worsening encephalopathy, fever, and/or malaise, the dx should strongly be considered and r/o. In this case, a peritoneal PMN of >250 would be diagnostic of bacterial peritonitis even if Gram stain is negative. The paracentesis also might provide micro confirmation. CT head would be useful for dx of cerebral edema a/w severe hepatic encephalopathy or in the presence of focal neurologic findings suggesting an epidural bleed. Cirrhotic patients are at greatest risk of GIT bleeds and it may worsen hepatic encephalopathy by increasing the protein load in the colon. EGD would be reasonable, particularly if the stools were guaiac positive or there was gross evidence of hematemesis or melena. In this case, there is no evidence of GIT bleed and there is mild hemoconcentration, possibly from peritonitis. Lactulose, and possibly Neomycin or Rifaximin, is a logical therapeutic trial in this patient if peritonitis is present. Serum NH3 level may suggest hepatic encephalopathy, if elevated, but does not have sufficient predictive value on its own to r/o or r/I this diagnosis

55 YO M is admitted for aspiration PNA. Over the past 8 months, he has had a relentless neurologic decline characterized by dementia with severe memory loss and decline in intellect. These ssx were preceded by 2-3 months of labile mood, weight loss, and HA. Currently he is awake but unable to answer questions. Neuro exam is notable for normal CN and sensation. He has marked myoclonus provoked by startle or bright lights, but it also occurs spontaneously during sleep. Prior evaluation revealed normal serum chemistries, negative serology for syphilis, and normal CSF. Head CT is normal. The infectious agent which caused his neuro syndrome is likely: A: DNA virus B: fungus C: protein lacking nucleic acids D: protozoan E: RNA virus

C prions are infectious proteins which lack nucleic acids and cause neurodegenerative disease. The MC prion disease in humans is sporadic CJD. Others include familial, fatal familial insomnia, kuru, and iatrogenic CJD. Prions result when an abnormal prion protein binds to a normal isoform of the prion protein, stimulating its conversion into the abnormal isoform. Abnormal prion isoforms have a greater proportion of B-structure and less alpha-helix than normal isoforms. The alpha to beta structural transition underlies the etiology of the CNS degeneration. The patient described has a typical presentation of s-CJD with sleep disturbance, fatigue, and defects in higher cortical function. CJD progresses quickly to dementia. Over 90% of patients exhibit myoclonus during the illness. Typically the myoclonus is provoked by startle, loud noises, or bright lights and will occur during sleep. The diagnosis requires an appropriate clinical presentation and no other etiologies on CSF. There is no widely available lab test for diagnosis. Brain bx may demonstrate spongiform degeneration and the presence of prion proteins

Which of the following statements regarding VZV infection after hematopoietic SCT is true? A: Acyclovir ppx is not warranted for patients with positive VZV serologies pre-transplant as the rate of zoster reactivation is low following transplant B: Herpes zoster resistance is a common problem, and a change from acyclovir to foscarnet is often required C: multidermatomal and disseminated zoster can occur in transplant patients who do not receive appropriate antiviral therapy D: zoster occurs more commonly following autologous transplant of stem cells then allogeneic transplant of stem cells E: zoster occurs more frequently during the first month after transplant

C reactivation zoster is almost a creditable event after SCT, occurring in 40% of allogeneic transplants and in 25% of autologous transplants. Patients can develop zoster immediately, but the highest risk period is several months after transplant. Usually just a very painful local infection in the immunocompetent host, transplant recipients' zoster can disseminate systemically from local disease and cause multi organ disease with effects on the lungs, liver, and CNS. Therefore, acyclovir or ganciclovir ppx is the standard of care at most transplant centers. Some data suggest that low doses of acyclovir for a year post-transplant is effective and may eliminate most cases of post-transplant zoster. Acyclovir is still extremely reliable for ppx and treatment of Varicella, with resistance being a very rare event. Foscarnet would be the DOC under these very rare circumstances

30 YO F with ESRD who receives her dialysis though a tunneled catheter in her shoulder presents with fever and severe lower back pain. On exam, she is uncomfortable and diaphoretic but HD stable. She has a soft 2/6 early systolic flow murmur. Her line site is red and warm with no pustular exudates. She is very tender over her lower back. Neurologically, she is completely intact. There is no evidence of Janeway lesions, Osler nodes, or Roth spots. Her WBC 16.7 with 12% bands. Immediate evaluation should include all of the following, except: A: MRI of the lumbar spine B: removal of the dialysis catheter C: TEE D: two sets of blood cx followed by Vancomycin as well as gram-negative coverage

C the major clinical concern is an epidural abscess or vertebral osteomyelitis, as well as a line infection 2/2 Staph aureus. Mets seeding during S aureus bacteremia has been estimated to occur as often as 30% of the time. Bones, joints, kidneys, and lungs are the MC sites. Mets infection to the spine should be evaluated in emergent fashion with MRI. The catheter should be removed as it is infected, based on exam. Infective endocarditis is a major concern. This dx is based on positive blood cx results and either a vegetation on ECHO, new pathologic murmur, or evidence of septic embolization on PE. A TEE is warranted in the evaluation for endocarditis (a disease they are at risk for). However, it need not be ordered emergently as it will not impact management during the initial phase of hospitalization. Moreover, because the dx can only be established in the presence of positive blood cx (or in rare cases serology of a difficult to culture organisms), a rational approach is to await positive blood cx before ordering the ECHO

In the urgent care clinic, you are evaluating a 47 YO F with poorly controlled DM who has a c/o of "sinusitis." She does not have a hx of atopy. She first noticed a HA x2 days ago and now feels very congested in her upper nasal passages. She has hyperesthesia over her nasal bridge as well and is inquiring about ABX to treat her infection. She has a bloody nasal d/c with occasional black specks. On exam, the sinuses are full and tender. She has a T 38.3C. Oral exam shows a black eschar on the roof of her mouth surrounded by discolored hyperemic areas on the palate. What is the most appropriate intervention at this time? A: ciprofloxacin and quarantine for possible anthrax B: ENT consult if no improvement with oral abx C: immediate bx of the involved areas and lipid amphotericin D: immediate bx of the lesion and voriconazole E: intranasal decongestants and close follow-up

C the patient has ssx of Mucormycosis. Although it is relatively uncommon fungal infection, patients with poorly controlled DM, patients receiving GC, immunocompromised patients, or patients with iron overload syndromes receiving desferrioxamine have an enhanced susceptibility to this devastating infection. The "gold standard" dx is tissue cx, but a common hallmark is the black eschar noted on the palate, which represents invasion of the fungus into tissue, with necrosis. The black eschar in this scenario should prompt the clinician to do more than prescribe treatment for sinusitis. Black eschar on the extrems can also be found with anthrax infection or spider bites. Given the mortality a/w this infection and the rapidity with which it progresses, it is not prudent to wait for ENT consultation after a course of ABX. The infection is usually fatal. Successful therapy requires reversal of the underlying predisposition (glucose control in this case), aggressive surgical debridement, and early initiation of anti fungal therapy. Voriconazole is not thought to be effective in the treatment of Mucormycosis. Posaconazole, an experimental azole anti fungal, has been shown to be effective in mouse models of the disease and has been used in patients unable to tolerate amphotericin.

You are the on-call physician practicing in a. suburban community. You receive a call from a 28 YO F with PMH of sarcoidosis who is currently on no meds. She is c/o the acute onset of crampy diffuse ABD pain and multiple episodes of emesis that are non-bloody. She has not had any light-headedness with standing or LOC. When questioned further, she states that her last meal was 5 hours previously, when she joined her friends for lunch at a local Chinese restaurant. She ate from the buffet, which included multiple poultry dishes and fried rice. What should you do? A: ask the patient to go to the nearest ED for resuscitation with IVF B: initiate ABX with Azithromycin C: reassure the patient that her illness is self-limiting and no further treatment is necessary if she can maintain adequate hydration D: refer for CT to assess for appendicitis E: refer for admission for IV Vancomycin and Ceftriaxone because of her immunocompromised state from sarcoidosis

C the patient likely has food poisoning from contamination of fried rice with Bacillus cereus. This toxin-mediated disease occurs when heat resistant spores germinate after boiling. Frying before serving may not destroy the toxin. The emetic form of illness occurs within 6 hours of eating and is self limited. No therapy is needed unless the patient develops severe hydration. This patient currently has no ssx c/w volume depletion; therefore, she does not need IVF at present. Sarcoidosis does not predispose to ID.

Which of the following is most likely a/w the lowest risk of HIV transmission to a healthcare provider after an accidental needle stick from a patient with HIV? A: needle is visibly contaminated with the patient's blood B: the injury is a deep tissue injury C: the patient whose blood is on the contaminated needle has been on ART for many years with a hx of resistance to many agents, but most recently had a successful viral suppression on current therapy D: the patient whose blood was on the contaminated needle was diagnosed with HIV 2 weeks ago

C the quoted risk for HIV transmission via a needlestick is 0.3%. However, this number is likely highly variable according to RF. Large-bore needle sticks where infected patient blood is visible are higher risk, as are deep tissue puncture to the provider. The patient's degree of virology control is generally inferred to be critical as well. Patients with viral load <1500/mL are considerably less likely to transmit via a needle stick than those with high viral loads. An extension of this point is that during acute and end-stage HIV infection, viral loads are extremely high and contagion by needle stick is likely to be much higher. In addition, during end-stage disease, virulent viral forms predominate, which may increase the risk to an even greater extent. Each of these variables must be assessed rapidly after an accidental high-risk needle stick. ART is effective at preventing HIV transmission via needle stick if given before viral RNA incorporates into the host genome as proviral DNA. This is thought to occur within ~48 hours, but under the best scenario, ART should be given within an hour of the needle stick. Circumstances are often murky, with key information such as viral load, viral resistance history, and even HIV aerostats of the patient variably available: therefore, urgent consult with an HIV and/or occupational health specialist is imperative after needle stick. HCV and HBV should also be considered.

Which of the following statements regarding epidemiology in the US is true as of 2005? A: HIV incidence is currently decreasing among MSM B: heterosexual contact accounts for the majority of HIV cases C: minority women 13-19 YO from the SE USA account for a growing proportion of prevalent HIV cases D: the proportion of cases due to high risk heterosexual contact has decreased dramatically over the past 20 years E: the proportion of prevalent HIV causes due to IVDA is currently increasing

C there is unfortunately a worsening HIV-1 epidemic in the SE USA among heterosexual women, with an alarming increase in the incidence amongst minority adolescent females. Despite fairly widespread knowledge among the US about HIV spread, there has been a gradual increase in the prevalence over the past decade in MSM, as well as those who acquire it via high-risk heterosexual intercourse. This last group is notable for major increases among the long-term female partners of men who engage in, or previously engaged in, high risk behavior. MSM still account for the largest proportion of cases in the country. The proportion of cases 2/2 IVDA is still significant, but decreasing.

26 YO F comes to clinic c/o 3-4 weeks of malodorous white vaginal d/c. She recently began having unprotected sex with a new male partner. He is asymptomatic. Her only med is po OCP. Exam shows a thin, white d/c that evenly coats the vagina. Further exam of the d/c reveals that it has pH 5.0 and has a "fishy" odor when 10% KOH is added. Microscopic exam reveals vaginal cells coated with coccobacillary organisms. Which is indicated? A: Acylovir, 400 mg po TID x 7 days B: Metronidazole, 2 g po x1 C: Metronidazole, 500 mg po bid x 7 days D: Fluconazole, 100 mg po x1 E: Vaginal douching

C this patient has a classic presentation and microscopic exam of bacterial vaginosis. BV, which is linked with HIV acquisition, HSV 2 shedding and acquisition, gonorrhea/Chlamydia acquisition, increased risk of preterm delivery, and subacute PID, is unfortunately very difficult to treat. With the best available regimens, women recur at a rate of 25%. Metronidazole, either as po or vaginal gel, is recommended for at least 7 days for primary infection and 10-14 for recurrence. Intravaginal Clindamycin for this duration is also an option but has been a/w more anaerobic drug resistance. Tx of males with Metronidazole does not prevent recurrence. Metronidazole 2 g po x 1 is the standard treatment for Trichomoniasis, but is too short for bacterial vaginosis. Fluconazole is used for vaginal candidiasis. Douching has no proven role in BV. Acylovir is recommended treatment for HSV2.

39 YO IVDA with hx of R-sided endocarditis and HIV notes back pain and fevers over the past week. He had an abscess recently on his R arm that he drained on his own. He is part of a needle-exchange program and always cleans his arm before shooting heroin into the vein in his antecubital fossa. On PE, T 38.1C, HR 124, BP 75/30. He is in great deal of distress and is slightly confused. He has a 4/6 LLSB murmur that varies with the respiratory cycle. His JVP is monophasic and to the jaw when seated at 90 degrees. Lung exam is clear. ABD is benign. He is very tender over his lower spine. His extremities are warm. Leg strength is 5/5 on the R, with 4/5 L-hip flexion and extension, 3/5 L-knee flexion and extension, and 3/5 L-foot extension. His Babinski is upgoing on the Left and downgoing on the R. What is the next step? A: avoidance of ABX until more definitive cx is obtained; serial Neuro exam B: urgent MRI and neurosurgery consult; Vancomycin after blood cx C: urgent MRi and neurosurgery consult; Vancomycin plus Cefepime after blood cx D: urgent MRI and Neurosurgery consult; avoid abx until more definitive cx obtained E: vancomycin plus cefepime after blood cx are drawn; serial neuro exam

C this patient has at minimum severe sepsis and has a very high pretest prob of an epidural abscess compressing his spinal cord, based on the development of weakness and UMN signs. Both represent true emergencies. From a sepsis standpoint, the most likely organisms are gram-positive skin flora with MRSA or MSSA repressing a distinct possibility. Vancomycin given IV is therefore imperative. However, other gram-negative organisms such as Pseudomonas and HACEK are sometimes causes of bacteremia and endocarditis in IVDA. Given this patients unstable HD state, it would be sensible to empirically cover gram-negative rods as well with Cefepime. As the infection is life threatening, it would not be prudent to await operative cx data prior to starting broad-spectrum abx. An epidural abscess needs to be dx and surgically decompressed as rapidly as possible to prevent permanent loss of neuro function

40 YO M with HIV (CD4 180, viral load 1000/mL) was treated for secondary syphilis based on generalized painless LAD, a diffuse maculopapular rash that includes his palms and soles, and a preceding primary genital chancre. He reported no neuro or ophthalmic ssx at the time and received 1 dose of IM PCN G. At the time of dx, his RPR was 1:64 and FTA-ABS was positive. He follows up 1 year later and si found to have RPR 1:64 and FTA-ABS remains positive. What is the appropriate intervention now? A: Aqueous PCN G 24 mU/day IV given as 4 mU q/4h x 10 days B: Doxycycline, 100 mg po bid C: LP D: PCN desensitization E: PCN G 2.4 mU IM weekly x 3 doses

C this patient has failed therapy for syphilis as his RPR has not decreased 4x over the course of the year. The FTA-ABS may remain positive even after effective treatment. There ei snow indication for PCN desensitization. A LP is indicated when there is not a 4x decrease in the RPR titer 6-12 months after appropriate treatment, particularly in HIV patients. A LP showing pleocytosis, elevated protein, and/or a positive VDRL will confirm the dx and the need for 10-14 days of IV-PCN. If the CSF is negative, re-treatment with 3 doses of IM PCN for late latent syphilis is adequate. There is no reason to begin treatment for neurosyphilis until the dx is made. Doxycycline, 100 mg po bid for 30 days, is an alternative treatment for syphilis of unknown duration or >1 year duration, but not for neurosyphilis

75 YO patient presents with fevers and wasting. He describes fatigue and malaise over the past several months and is concerned that he has been losing weight. On exam, he is noted to have a low-grade fever and a soft diastolic heart murmur is appreciated. Labs reveal a normocytic, normochromic anemia. (3) separate blood cx grow Cardiobacterium hominis. Which of the following statements is true about this patient's clinical condition? A: abx are not likely to improve his condition B: ECHO will likely be wnl C: he has a form of endocarditis with a high risk of emboli D: he will likely need surgery E: the positive blood cx are likely a skin contaminant

C this patient has subacute bacterial endocarditis due to infection with one of the HACEK organisms. The HACEK organisms (Haemophilus, Actinobacillus, Cardiobacterium, Eikenella, and Kingella) are gram-negative rods that reside in the oral cavity. They are responsible for about 3% of cases of IE in most series. They are the MC cause of gram-negative endocarditis in non-drug abusers. Most patients have a hx of poor dentition or recent dental procedure. Often, patients are initially diagnosed with cx-negative endocarditis, as these organisms may be slow growing and fastidious. Cx must be specified for prolonged cx of fastidious organisms. HACEK endocarditis is typically subacute, and the risk of embolic phenomena to the bone, skin, kidneys, and vasculature is high. Vegetations are seen on ~85% of TEE. Cure rates are excellent with abx alone; native valves require 4 weeks and prosthetic valves require 6 weeks of treatment. Ceftriaxone is DOC, with ampicillin/gentamicin as an alternative. Sensitivities may be delayed due to organism's slow growth.

68 YO M is brought to the ED with AMS, fever, and leg pain. His wife reports that he first c/o pain in his leg yesterday, and there was some slight redness in this area. Over the night, he developed a fever to as high as 39.8C and became obtunded this morning. At that point, his family brought him to the ED. Upon arrival, he is unresponsive to voice and withdraws to pain. The VS: BP 88/40, HR 126, RR 28, T 39.3C, and SaO2 95% on RA. Exam of the R-Leg shows diffuse swelling and brawny edema. The patient grimaces in pain when the area is touched. There are several bullae filled with dark blue to purple fluid. Labs show pH 7.22, PaCO2 28, PaO2 93. The Cr 3.2. WBC is elevated at 22.6 with a differential of 70% PMNs, 28% bands, and 2% lymphocytes. A bulla is aspirated and the gram stain shows gram-positive cocci in chains. What is the most appropriate therapy? A: ampicillin, clindamycin, and gentamicin B: clindamycin and PCN C: clindamycin, PCN, and surgical debridement D: PCN and surgical debridement E: vancomycin, PCN, and surgical debridement

C this patient is presenting with septic shock 2/2 necrotizing fasciitis with GAS. NF presents with fever and pain of the affected area that progresses rapidly to severe systemic ssx. Swelling and brawny edema may be present early in the disease, progressing rapidly to dark-red induration with bullae filled with bluish to purple fluid. Pathologically, the underlying dermis shows extensive thrombosis of vessels in the dermis. NF is commonly caused by GAS, specifically S pyogenes, or mixed aerobic-anaerobic infections. In this patient, the gram-positive cocci in chains suggest S pyogenes as the cause. The initial treatment of patients is surgical debridement of the the affected area. The area of debridement is frequently very large. During surgery, all necrotic tissue should be removed and any increased compartment pressure should be relieved. In addition, appropriate abx should be initiated. For GAS, the combination of clindamycin and PCN should be used. PCN is bactericidal for Streptococcus as is Clindamycin. Clindamycin also neutralizes the toxins produced by GAS. ABX therapy alone should not be used as NF is rapidly fatal w/o surgical intervention. Vancomycin is not a first line abx in NF and should be used only for PCN allergy

Which of the following individuals with a known hx of prior latent TB (w/o therapy) has the greatest likelihood of developing reactivation TB? A: 28 YO F with anorexia nervosa, a BMI 16, and a serum Alb 2.3 B: 36 YO IVDA who does not have HIV but is homeless C: 42 YO M who is HIV positive with CD4 350 on ART D: 68 YO M who worked as a stone mason for many years and has silicosis E: 73 YO M who was infected while stationed in Korea in 1958

C while all the patients listed have increased risk of developing reactivation TB, the greatest RF for development of active TB is HIV+. The risk of developing active TB is greatest in those with the lowest CD4 counts; however, having a CD4 above a threshold value does not negate the risk of developing an active infection. The reported incidence of developing active TB in HIV+ individuals is 100x that of an immunocompetent individuals. All of these listed have RF for developing active TB. Malnutrition and severe underweight confers a twofold greater risk of developing active TB, whereas IVDA increases the risk 10-30x. Silicosis also increases the risk of developing active TB in 30 times. While the risk of developing active TB is greatest in the first year after exposure, the risk also increases in elderly.

Which virus is the leading cause of respiratory disease in infants and children? A: adenovirus B: enterovirus C: human RSV D: parainfluenza virus E: rhinovirus

C while rhinovirus likely represents one of the most common viral illnesses in human, RSV is the most prevalent respiratory pathogen in young children and infants. Adenovirus is another common cause of the common cold and pharyngitis in children. Enterovirus can cause an undifferentiated febrile illness and occasionally URTI. Parainfluenza typically causes croup or tracheobronchiolitis among children.

Which of the following represents an emergent (same day) indication for cardiac surgery in a patient with IE? A: cx-proven fungal endocarditis B: cx-proven resistant organism with septic pulmonary emboli C: prosthetic valve endocarditis 4 months after surgery D: sinus of Valsalva abscess ruptured into the R heart E: Staphy lugdunensis in a patient with previous history of endocarditis

Cx-proven fungal (particularly mold) endocarditis and sinus of Valsalva abscess represent situations where strong evidence supports cardiac surgery; a ruptured sinus of Valsalva abscess represents a surgical emergency, while fungal endocarditis is usually considered more of an elective case, with earlier surgical management preferred. Acute aortic regard plus preclosure of the mitral valve is also an indication for immediate sx. Septic pulmonary emboli alone do not necessitate surgery; however, if a 10-mm vegetation is also seen this would best be treated surgically.k. Staph prosthetic valve endocarditis (PVE) or PVE within 2 months of surgery is an example of reasonable indications for surgical management. In contrast to fungal endocarditis, there are no particular spp of bacteria that merit surgical treatment, independent of other factors

In the inpatient setting, extended spectrum B-lactamase producing gram-negative infections are most likely to occur after frequent use of which of the following class of ABX? A: Carbapenems B: Macrolides C: Quinolones D: third gen Cephalosporins

D B-lactamases are a major source of ABX resistance in gram-negative bacilli. Many produce broad spectrum B-lactamases that confer resistance to PCN and 1st gen Cephalosporins. The addition of clavulanate, a B-lactamase inhibitor, to an antibiotic regimen is often enough to overcome resistance. ESBL, however, lead to resistance to all B-lactam drugs including 3rd and 4th gen Cephalosporins. ESBL-producing genes can be acquired by gram-negative bacilli via plasmids and are becoming increasingly prevalent in hospitals worldwide. Klebsiella and E coli are the MC bacteria which acquire ESBLs, though it can be seen in many other gram-negatives including Serratia, Proteus, Enterobacter, and Citrobacter. The MC clinical scenario for this development is prevalent use of 3rd gen Cephalosporins. Carbapenems should be considered 1st line for these bacteria. Macrolides and quinolones have different MOA than B-lactam abx and do not apply selective pressure to generate ESBLs

27 YO M presents to clinic with 2 weeks of ST, malaise, myalgia, night sweats, fevers, and chills. He visited an urgent care center and was told that he likely had the flu. He was told he had a "negative test for mono." The patient is homosexual, states that he is in a monogamous relationship and has unprotected receptive and insertive anal and oral intercourse with one partner. He had several partners prior to his current partner 4 years ago but none recently. He reports a negative HIV-1 test 2 years ago and recalls being diagnosed with Chlamydia infection 4 years ago. He is otherwise healthy with no medical problems. You wish to r/o acute HIV. Which blood test should you order? A: CD4 count B: HIV ELISA/Western blot combination C: HIV resistance panel D: HIV RNA PCR E: HIV RNA by ultra sensitive PCR

D Acute HIV should be suspected in any at-risk person who presents with a mono-like illness: it is diagnosed by positive plasma RNA PCR. Patients typically have not developed sufficient ABS to the virus yet to develop a positive ELISA, and the diagnosis of HIV is usually missed if this test is sent within the first 2 months of acquisition. It is tempting for clinicians to send an ultra sensitive PCR, but this only decreases specificity (FP tests with detection of very low levels of HIV are possible due to cross contamination of the lab) with no other benefit. There is typically a massive amount of HIV in the plasma during acute infection, and the ultra sensitive assay is never required for detection at this stage of disease. Ultra sensitive assays are helpful in the context of therapy to ensure there is not persistence of low-level viremia. CD4 decreases during many acute infections, including HIV, and is therefore not diagnostically appropriate. CD4 counts are useful to risk stratify for opportunistic infections in stable patients with known HIV infection. Resistance tests are sent only when the diagnosis is confirmed.

Which of these statements about C diff-associated disease relapses is true? A: a first recurrence does not imply greater risk of further recurrence B: most recurrences are 2/2 antibiotic resistance C: recurrent C diff disease has been a/w higher risk of CRC D: recurrent disease is a/w serious cx E: testing for CL of C diff is warranted after treating recurrences

D C-diff associated disease recurrences are most often due to reinfection (because patients carry similar RF as they did before infection) or relapse (due to persistence of spores in the bowel). Approximately 15-30% of patients have at least 1 relapse. Recurrent disease has been a/w ~10% risk of serious cx including shock, megacolon, perforation, colectomy, or death at 30 days. Metronidazole resistance occurs but is actually very rare. Metronidazole and Vancomycin have similar efficacy in the first episode of recurrence. Repeat courses of Metronidazole should be avoided for neurotoxic risk. Unfortunately, patients who recur are more likely to recur again, and many patients receive multiple cycles of ABX and are even candidates for more extreme measures such as IV-Ig or fecal transplant via stool enema. Testing for CL is not likely to be informative. A negative stool antigen would not change management, as symptomatic improvement is the true goal of therapy. A positive stool antigen and toxin test in a patient whose ssx have improved after standard therapy implies colonization, not disease. It can therefore be needlessly discouraging to patients and again does not impact management. There is no known associated between C -diff and CRC.

Empirical ABX for continuous ambulatory peritoneal dialysis (CAPD) patients with peritonitis should be directed towards which organism? A: enteric gram-negative rods B: enteric gram-negative rods and yeast C: gram-positive cocci D: gram-positive cocci plus enteric gram-negative rods E: gram-positive cocci plus enteric gram-negative rods plus yeast

D CAPD-associated peritonitis is different from primary or 2nd peritonitis in that most infections are caused by skin flora rather than gut pathogens. Therefore, ABX should be directed towards Staphylococcal spp, especially S aureus. These spp, including coagulase-negative Staph, account for 40-50% of cases. Recently, S aureus has increased in frequency. Typical ssx include diffuse pain and peritoneal signs. Peritoneal fluid will be cloudy with >100 WBCs/uL with >50% PMNs. Vancomycin is necessary in area where MRSA is common. Intraperitoneal loading doses of this drug are typically given. Though gram-negative and Candida infections do occur and should be covered prior to the return of cx data, they are less common. The presence of more than one spp in cx should prompt an evaluation for secondary peritonitis. Once definitive cx data return, then ABX can be narrowed towards the offending agent. If there is no symptomatic improvement within 48 h or the patient appears septic, then catheter removal is standard. These infections are in many ways similar to vascular catheter infections, and their management therefore has many parallels

All of the following are common manifestations of CMV infection s/p lung transplant, except: A: bronchiolitis obliterans B: CMV esophagitis C: CMV PNA D: CMV retinitis E: CMV syndrome (fever, malaise, cytopenia, transaminitis, and CMV viremia)

D CMV retinitis, a common CMV infection in HIV patients, occurs very rarely in solid organ transplant patients. CMV does affect the lung in a majority of transplant patients if either donor or recipient is CMV-seropositive pretransplant. It rarely occurs within 2 weeks of transplant. CMV very commonly causes a pneumonitis that clinically is difficult to distinguish from acute rejection. Prior CMV infection has been a/w bronchiolitis obliterans syndrome (chronic rejection) in lung transplant. As with HIV, the GIT is commonly involved with CMV infection. Endoscopy with biopsy showing characteristic giant cells, not serum PCR, is necessary to make this diagnosis. The CMV syndrome is also common in lung transplant. Serum CMV PCR should be sent as part of the workup for all nonspecific fevers, worsening lung function, liver function abnormalities, or falling WBC counts occurring more than a couple of weeks s/p transplant.

54 YO F presents to the ED c/o pain and redness of her L face and cheek. The area of redness began abruptly yesterday. At that time, the area was about 5 mm2 near the nasolabial fold. There was rapid progression of the redness to an area that is now about 5cm2. In addition, she is c/o intense pain in this area. On exam, there is a well-demarcated 5cm2 area of erythema along her L nasolabial fold. The borders are raised and indurated. The entire area is very tender to touch. Over the next 24 h, the affected area begins to develop a flaccid bullae. What is the most appropriate treatment for this patient? A: acyclovir B: clindamycin C: clindamycin and PCN D: PCN E: TMP/SMX

D Erysipelas is a soft tissue infection caused by Strep progenies that occurs more frequently on the face or extremities. The infection is marked by abrupt onset of fiery-red swelling with intense pain. The infection progresses rapidly and is marked by well-defined and indurated margins. Flaccid bullae may develop on the 2nd or 3rd day. Only rarely does the infection involve the deeper soft tissues. PCN is the treatment of choice. However, swelling may progress despite appropriate treatment with desquamation of the affected areas

a patient presents to clinic c/o nausea/vomiting, crampy ABD pain, and markedly increased flatus. He has not experienced diarrhea, but notes that he has been belching more than usual and describes a "sulfur like" odor when he does so. He returned from a 3-week trip to Peru and Ecuador several days ago and notes that his symptoms began about 1 week ago. Giardiasis is considered in the ddx. Which is true regarding Giardia? A: boiling water prior to ingestion will not kill cysts B: Giardia is a disease of developing nations; if this patient had not travelled, there would be no likelihood of giardiasis C: hematogenous dissemination and eosinophilia are common D: ingestion of as few as 10 cysts may cause human disease E: lack of diarrhea makes the diagnosis very unlikely

D Giardia is one of the most common parasitic diseases, with worldwide distribution. It occurs in developed and developing countries. Infection follows ingestion of environmental cysts, which excyst in the small intestine releasing flagellated trophozoites. Giardia does not disseminate hematogenously; it remains in the SI. Cysts are excreted in stool, which accounts for person-person contact; however, they do not survive for long periods in the feces. Ingestion of contaminated water sources is another major form of infection. Giardia cysts can thrive in cold water for months. Filtering or boiling water will remove the cysts. As few as 10 cysts can cause human disease, which has a broad spectrum of presentations. Most infected patients are asymptomatic. Ssx in infected patients are due to small intestinal dysfunction. Typical early ssx include diarrhea, aBD pain, bloating, nausea, vomiting, flatus, and belching. Diarrhea is very common, particularly early, but in some patients constipation will occur. Later, diarrhea may resolve, with malabsorption ssx predominating. The presence of fever, eosinophilia, blood or mucus in stools, or colitis should suggest an alternative diagnosis. Dx is made by demonstrating parasite antigens, cysts, or trophozoites in the stool

Helicobacter pylori colonization is implicated in all the following conditions, except: A: duodenal ulcer disease B: gastric ADENO C: gastric mucosa-associated lymphoid tissue (MALT) lymphoma D: GERD E: PUD

D H pylori is thought to colonize ~50% (30% in developed, >80% in developing) of the World's population. The organisms induces a direct tissue response in the stomach, with evidence of mononuclear and PMNs infiltrates in all of those with colonization, regardless of whether or not ssx are present. Gastric ulceration and ADENO of the stomach arise in a/w gastritis. MALT is specific to H pylori and is due to prolonged B-cell activation. Though it does not directly infect the intestine, it does diminish somatostatin production, indirectly contributing to the development of duodenal ulcers. GERD is not caused by H pylori, some early, controversial research may suggest it is in fact protective against this condition.

Which of the following STIs is the MC in the US? A: gonorrhea B: HSV-2 C: HIV-1 D: HPV E: syphilis

D HPV is thought to be the most common STI in the US. A recent study of initially seronegative college-aged women found 60% became infected within 5 years. This underscores the importance of the recent development of effective vaccines and continued cervical cancer screening strategies. Approximately 20-25% of the US past 11-12 YO is seropositive for HSV-2. The high prevalence of this infection in the gen pop is due to several factors including lifelong infection, ongoing transmission during latent infection due to asymptomatic shedding of hSV-2 in gate genital mucosa, and high rates of transmission within monogamous couples. These features are markedly different than those with bacterial STIs such as gonorrhea and syphilis, which require high rates of partner change to persist in subpopulations. HIV-1 infections are still concentrated with high-risk populations (MSM, IVDA, high risk heterosexuals, and immigrants from high prevalence regions)

Indinavir is a protease inhibitor that carries which AE unique for HIV ART agent? A: abnormal dreams B: benign hyperbilirubinemia C: hepatic necrosis in pregnant women D: nephrolithiasis E: pancreatitis

D Indinavir is the only agent to cause nephrolithiasis. NRTIs, particularly stavudine and didanosine (d4T and ddI) are a/w mitochondrial toxicity and pancreatitis. Nevirapine can cause hepatic necrosis in women, particularly with CD4 >350. Efavirenz, a very commonly used agent, causes dream disturbances that usually, but not always, subside after the first month of treatment. Both indinavir and atazanavir cause a benign indirect hyperbilirubinemia reminiscent of Gilbert syndrome

In an HIV-infected patient, Isospora belli infection is different from Cryptosporidium infection in which of the following ways? A: Isospora causes a more fulminant diarrhea syndrome leading to rapid dehydration and even death in the absence of rapid rehydration B: Isospora infection may cause biliary tract disease, whereas Cryptosporidiosis is strictly limited to the lumen of the SI and large bowel C: Isospora is more likely to infect immunocompetent hosts than Crypto D: Isospora is less challenging to treat and generally responds well to TMP-SMX E: Isospora occasionally causes large outbreaks among the gen popu

D Isospora and Cryptosporidium cause very similar disease in AIDS patients that range from intermittent, self-resolved watery diarrhea with ABD cramping and sometimes nausea, to potentially fatal cholera-like presentation in the most immunocompromised hosts. Crypto may cause biliary disease and can lead to cholangitis. Isospora is limited to the gut lumen. Crypto is not always opportunistic and has led to widespread community outbreaks. Isospora is not seen in immunocompetent hosts. Finally, treatment for Isospora is usually successful. In fact, this infection is rarely seen in the developed world because TMP-SMX, which is commonly used for PCP, tends to eradicate Isospora. Crypto, on the other hand, is very difficult to cure and interventions are controversial. Some clinicians favor nitazoxanone, but cure rates are mediocre and immune reconstitution with ART is ultimately critical to cure the GIT disease

72 YO F admitted to ICU with respiratory failure. She has fever, obtundation, and b/l parenchymal consolidation on CXR. Which of the following is true about Legionella PNA? A: acute and convalescent antibodies are not helpful due to the presence of multiple serotypes B: Legionella can never be seen on Gram stain C: Legionella cx grow rapidly on the proper media D: Legionella urinary antigen maintains utility after ABX use E: PCR for Legionella is the "gold standard" diagnostic test

D Legionella urine antigen is detectable within 3 days of SSX and will remain positive for 2 months. It is not affected by ABX. The urinary antigen test is formulated to detect only Legionella pneumophilia (~80% of infections), but cross-reactivity with other spp has been reported. The urinary test is sensitive and highly specific. Typically, Gram's staining from sterile sites such as pleural fluid show numerous WBCs but no organisms. However, Legionella may appear as faint, pleomorphic gram-negative bacilli. It may be cx'd from sputum even when epithelial cells are present. Cx, grown on selective media, take 3-5 days to show visible growth. Antibody detection using acute and convalescent serum is an accurate means of diagnosis. A 4x rise is diagnostic, but this takes up to 12 weeks so is most useful for epidemiology. Legionella PCR has not been shown to be adequate sens/spec for clinical use. It is used for environmental sampling.

87 YO M nursing home patient is brought to local ED obtunded and ill-appearing. Per staff, he has low-grade temperatures, poor appetite, and lethargy over several days. A LP is performed, and the gram stain returns gram-positive rods and many WBCs. Listeria meningitis is diagnosed and ABX are started. Which of the following best describes a clinical difference between Listeria and other causes of bacterial meningitis? A: more frequent nuchal rigidity B: more PMNs are present on CSF ddx C: photophobia is more common D: presentation is often more subacute E: WBC count is often more elevated in CSF

D Listeria meningitis typically affects the elderly and the chronically ill. It is frequently a more subacute (developing over days) illness than other etiologies of bacterial meningitis. It may be mistaken for aseptic meningitis. Meningeal signs, including nuchal rigidity, are less common, as is photophobia, than in other, more acute causes of bacterial meningitis. Typically WBC counts in CSF range from 100-5000 with a less pronounced neutrophilia. 75% of patients will have a WBC count <1000. Grams stain is positive in 30-40% of cases. Case fatality rates are ~20%

Previously healthy 19 YO M presents with several days of HA, cough with scant sputum, and fever 38.6C. On exam, pharyngeal erythema is noted and lung fields are clear. CXR reveals focal bronchoPNA in the lower lobes. His HCT is 24, down from baseline of 46. The only other lab abnormality is indirect bilirubin of 3.4. A PBS shows no abnormalities. A cold agglutinin titer is measured at 1:64. What is the most likely agent? A: Coxiella burnetti B: Legionella pneumophila C: MRSA D: Mycoplasma pneumoniae E: Strep pneumoniae

D Mycoplasma pneumoniae is a common cause of PNA that is often under diagnosed based on difficult and time-consuming cx techniques, it likely causes mild respiratory ssx, and because it is adequately treated with standard ABX for CA-PNA. It is spread easily person-person, and outbreaks in crowded conditions are common. Most patients develop a cough w/o radiographic abnormalities. Pharyngitis, rhinitis, and ear pain are also common. M pneumoniae commonly induces the production of cold agglutinins, which in turn can cause an IgM and complement-mediated intravascular hemolytic anemia. The presence of cold agglutinins is specific for M pneumoniae infection only in the context of a consistent clinical picture for the infection. Cold agglutinins are more common in children. Blood smear shows no abnormality, which is in contrast to IgG or warm-type hemolytic anemia where spherocytes are seen. Since there is no easy diagnostic test, empiric treatment is often given.

68 YO F seeks evaluation for an ulcerative lesion on her R hand. She reports that the area on the back of her R hand was initially red and not painful. There appeared to be a puncture wound in the center of the area, and she thought she had a simple scratch acquired while gardening. Over the next several days, the lesion became verrucous and ulcerated. Now, the patient has noticed several nodular areas along the arm, one of which ulcerated and began draining a serous fluid today. She is also noted to have an enlarged and tender epitrochlear LN on the R-arm. A biopsy of the edge shows ovoid and cigar-shaped yeasts. Sporotrichosis is diagnosed. What is the best treatment? A: Amphotericin B IV B: Caspofungin IV C: Clotrimazole topical D: Itraconazole po E: Selenium sulfide topical

D Sporothrix schenkii is a thermally dimorphic fungus found in soil, plants, and moss and occurs mostly in gardeners, farmers, florists, and forestry workers. Sporotrichosis develops after inoculation of the organisms into the skin with a contaminated puncture or scratch. The disease typically presents as a fixed cutaneous lesion or with lymphocutaneous spread. The initial lesion typically ulcerates and becomes verrucous in appearance. The draining lymphatic channels become affected in up to 80% of cases. This presents as a painless nodule along the lymphatic channel, which ulcerates. A ddx is made by cx of the organism. A biopsy of the skin lesion may show ovoid or cigar shaped yeasts. Treatment is systemic therapy. Options include po Itraconazole, saturated solution of KI, and terbinafine. However, terbinafine has not been approved for this indication in the US. Topical antifungals are not effective. In cases of serious systemic disease such as pulmonary sporotrichosis, amphotericin B is the DOC. Caspofungin is not effective.

24 YO F with diffuse arthralgia and morning stiffness in her hands, knees, and wrists. Two weeks earlier she had a self-limited febrile illness notable for a red facial rash and lacy reticular rash on her extremities. On exam, her b/l wrists, MCP joints, and PIP joints are warm and slightly boggy. What test will reveal her dx? A: ANA B: Chlamydia trachomatis ligase chain reaction of the urine C: Joint aspiration for crystals and cx D: Parvo B19 IgM E: RF

D The most likely dx based on her antecedent illness with a facial rash is Parvovirus infection. Parvo commonly leads to diffuse systemic arthritis in the immune phase of the illness when IgM antibodies are developed. Occasionally the arthritis persists over months and can mimic RA. The acute nature of these complaints makes SLE and RA less likely. Reactive arthritis due to Chlamydia or a list of other bacterial pathogens tends to affect large joints such as the SI joints and spine. It is also sometimes a/w uveitis and urethritis. The large number of joints involved with a symmetric distribution argues against crystal or septic arthropathy.

17 YO F presents to clinic c/o vaginal itchiness and malodorous d/c. She is sexually active with multiple partners, and she is interested in getting tested for STDs. A wet-mount microscopy shows trichomonal parasites. Which is true about trichomoniasis? A: a majority of women are asymptomatic B: no treatment is necessary as it is self limited C: the patients sexual partners need not be treated D: it can only be spread sexually E: it is 100% sensitive to metronidazole

D Trichomoniasis is spread via sexual contact with an infected partner. Many men are asymptomatic but may have ssx of urethritis, epididymitis, or prostatitis. Most women will have SSX of infection which include vaginal itching, dyspareunia, and malodorous d/c. These ssx do not distinguish from other forms of vaginitis, such as BV. Trich is not a self-limited infection and should be treated for symptomatic and public health reasons. Wet-mount for mobile trichomonads has a sensitivity of 50-60% in routine exam. Direct immunofluorescent antibody staining of secretions is more sensitive and can be performed immediately. Cx is not widely available and takes 3-7 days. Treatment should consist of Metronidazole either as single 2-g dose or 500 mg bid for 7 days; all sexual partners need treatment. Trich resistant to metronidazole has been reported and is managed with increased doses of metronidazole or with tinidazole

When given as a first line agent for invasive Aspergillus infection, voriconazole commonly causes all of the following AE, except: A: drug-drug interactions B: hepatotoxicity C: photosensitivity skin rashes D: renal toxicity E: visual disturbances

D Voriconazole is an azole anti fungal with a broader spectrum of activity than fluconazole against Candida and has activity against Aspergillus. It is available in po and IV forms. Its visual disturbances are common, transient, and harmless, but patients should be warned to expect them. Voriconazole interacts significantly with many other meds, including immunosuppressive agents, such as tacrolimus, that are often used in patients at risk for systemic fungal infections. Voriconazole may also cause liver toxicity and photosensitivity. Renal toxicity is an issue with amphotericin B products rather than azole.

All of the following anti fungal meds may be used to treat Candida albicans fungemia, except: A: Amphotericin B B: Caspofungin C: Fluconazole D: Terbinafine E: Voriconazole

D all patients with Candida fungemia should be treated with ssx antifungals. Terbinafine is an agent used for dermatophyte infection of the foot. Fluconazole has been shown to be an effective agent for candidemia with equivalence to amphotericin products and caspofungin. For e.g., Candida glabrata is typically resistant to fluconzale. Voriconazole is also active against C albicans, but has many drug interactions which make it less desirable to use. Azoles such as fluconazole and voriconazole are less active against glabrata and krusei. Many practitioners therefore prefer to initiate treatment with caspofungin or amphotericin products in a patient with candidemia until the yeast isolate is definitively ID'd as albicans. Caspofungin and other echinocandins are gaining popularity due to their broad efficacy against most yeast isolates and benign AE profile. Amphotericin B is effective in fungemia but frequently causes rigors, -lyte wasting, and renal insufficiency. Newer lipid formulations mitigate these effects to varying extents.

20 YO F is 36 weeks pregnant and presents for her first evaluation. She is diagnosed with Chlamydia trachomatis infection of the cervix. Upon delivery, what cx is her infant most at risk for? A: jaundice B: hydrocephalus C: hutchinson triad D: conjunctivitis E: sensorineural deafness

D congenital infection from maternal transmission can lead to severe consequences for the neonate; thus, prenatal care and screening for infection are very important. C trachomatis is a/w up to 25% of exposed neonates who develop inclusion conjunctivitis. It can also be a/w PNA and otitis media int he newborn. PNA in the newborn has been a/w later development of bronchitis and asthma. Hydrocephalus can be a/w toxoplasmosis. Hutchinson triad, which is Hutchinson teeth (blunted upper incisor), interstitial keratitis, and CN 8 deafness, is due to congenital syphilis. Sensorineural deafness can be a/w congenital rubella exposure. Treatment of C trachomatis in the infant consists of oral Erythromycin.

36 YO M with HIV/AIDS (CD4 112) develops a scaly, waxy, yellowish, patchy, crusty, pruritic rash on and around his nose. The rest of the skin is wnl. Which is the diagnosis? A: Molloscum contagiosum B: Psoriasis C: Reactive herpes zoster D: Seborrheic dermatitis

D dermatologic problems occur in >90% of patients with HIV infection. Seborrheic dermatitis is perhaps the MC rash in HIV patients, affecting up to 50% of patients. The prevalence increases with falling CD4 count. The rash involves the scalp and the face, appearing as described. Therapy is standard topical treatment, although often a topical anti fungal is added because of concomitant infection with Pityrosporum. Herpes zoster reactivation is painful and dermatomal, with progression of papule to vesicles to small pustules and then crusting. Molloscum typically appears as one or many small pearly umbilicated asymptomatic papule occurring anywhere on the body. They can be a significant cosmetic issue in patients with AIDS. Psoriasis is not more common in HIV infection but may be more severe and generalized. It would be uncommon to only involve the face

the standard starting regimen for AFB smear-positive active TB is: A: INH B: INH, rifampin C: INH, moxifloxacin, pyrazinamide, ethambutol D: INH, rifampin, pyrazinamide, ethambutol E: rifampin, moxifloxacin, pyrazinamide, ethambutol

D drugs used to treat MtB are classified as 1st or 2nd line. First line agents, which are proven most effective and are necessary for any short-course treatment regimen, including RIPE therapy: Rifampin, INH, Pyrazinamide, Ethambutol. First line supplemental agents, which are highly effective with acceptable toxicity, include rifabutin, rifapentine, and streptomycin. 2nd line agents, which are either less clinically active or have greater toxicity, including para-aminosalicyclic acid, ethionamide, cycloserine, amikacin, and capreomycin. The fluroquinolones, Levo and Moxifloxacin, are active against MtB, but are not yet 1st line. While not approved in the US, promising trials are underway. Some experts consider moxifloxacin a supplemental first line therapy. It is necessary to have at least 3 active agents during the 2-month induction phase of active TB. Ethambutol is initially used as a 4th agent to account for the possibility of drug resistance to one of the other agents. Consolidation includes rifampin and INH, and is 4-7 months in length, depending on anatomic location of infection as well as clearance of sputum cx at 2 months.

44 YO M presents to the ED for evaluation of severe ST. His ssx began this morning with mild irritation on swallowing and have gotten progressively severe over the course of 12 hours. He has been experiencing a fever to as high as 39C at home and reports progressive SOB. He denies antecedent rhinorrhea or tooth or jaw pain. He has had no ill contacts. On PE, the patient appears flushed and in respiratory distress with use of accessory ms of respiration. Inspiratory stridor is present. He is sitting leaning forward and is drooling with his neck extended. His VS are 39.5C, BP 116/60, HR 118, RR 24, SaO2 95% on RA. Exam of the oropharynx shows erythema of the posterior oropharynx without exudates or tonsillar enlargement. The uvula is midline. There is no sinus tenderness and no cervical LAD. His lungs are CTA, and CVS exam reveals a regular tachycardia with a II/VI systolic ejection murmur heard at the RUSB. ABD, extrem, and neuro exams are normal. Labs show WBC 17 with 87% PMNs, 8% bands, 4% lymphos, and 1% monocytes. Hgb 13.4, HCT 44%. ABG on RA has pH 7.32, CO2 48, PaO2 92. A lateral XR shows an edematous epiglottis. What isa he most appropriate step in evaluation and treatment? A: Ampicillin, 500 mg IV q6h B: Ceftriaxone, Ig IV q24h C: endotracheal intubation and Ampicillin, 500 mg IV q6h D: endotracheal intubation, Ceftriaxone 1 g IVq24, and Clindamycin, 600 mg IV q6h E: laryngoscopy and close observation

D generally thought of as a disease of children, epiglottis is increasingly becoming an adult disease since the wide use of HiB vaccination. Epiglottitis can cause life-threatening airway obstruction due to cellulitis of the epiglottis an supraglottic tissues, classically from HiB. However, other organisms are also known to cause this, including non-typeable Hinfluenza, S pneumonia, H parainfluenza, S aureus, and viral infection. The initial evaluation and treatment in adults include airway management and IV abx. The patient presented here is demonstrating signs of impending airway obstruction with stridor, inability to swallow secretions, and use of accessory ms. A lateral neck XR shows typical thumb sign indicative of a swollen epiglottis. In addition, he has evidence of hypoventilation with CO2 retention. Thus, in addition to the abx, he should be intubated and mechanically ventilated electively under a controlled setting as he is high risk for obstruction. ABX should cover the typical organisms outlined above and include coverage for oral anaerobes. In adults presenting w/o overt impending airway obstruction, laryngoscopy would be indicated to assess airway latency. ET intubation would be recommended for those with >50% airway obstruction. In children, ET intubation is often recommended as laryngoscopy in children has provoked airway obstruction to a much greater degree than adults, and increased risk of mortality has been demonstrated in some series in children when the airway is managed expectantly.

42 YO M with AIDS and CD4 of 23 presents with SOB and fatigue in the absence of fevers. On exam, he appears chronically ill with pale conjunctiva. HCT 16. MCV is 84. RBC distribution width is wnl. Bilirubin, LDH, and haptoglobin are all within normal limits. Reticulocyte count is wnl. WBC 4.3, with ANC 2500. PLT 105. Which of the following tests is most likely to produce a diagnosis? A: BM aspirate and biopsy B: Iron studies C: Parvo B19 IgG D: Parvo B19 PCR E: Parvo B19 IgM F: PBS

D immunocompromised patients occasionally can't clear Parvo infection due to lack of T cell function. As Parvo B19 selectively infects RBC precursors, persistent infection can lead to prolonged RBC aplasia and persistent drop in HCT, with low or absent reticulocytes. Pure red cell aplasia has been reported in HIV infection, lymphoproliferative disorders, and after transplant. Fe studies will show adequate iron but decreased utilization. The PBS usually shows no abnormalities other than normocytic anemia and the absence of reticulocytes. Antibody tests are not useful in this setting as immunocompromised patients do not produce adequate antibodies against the virus. Therefore, a PCR is the most useful diagnostic test. BM biopsy may be suggestive as it will show no red cell precursors, but usually a less invasive PCR test is adequate. Immediate therapy is with red cell transfusion, followed by IV-Ig, which contain adequate titers of antibody against Parvo B19.

Which of the following statements regarding severe acute respiratory syndromes (SARS) is true? A: SARS displays poor human-human transmission B: SARS is more severe among children than adults C: The etiologic agent is in the Adenovirus family D: there have been no reported cases of SARS since 2004 E: There is no known environmental reservoir for virus causing SARS

D in 2002, an outbreak of SARS began in China. Ultimately 8,000 cases were recorded in 28 countries. The etiologic agent was a/w Coronavirus family, now named SARS-CoV. The natural reservoir appears to be the horseshoe bat, though human exposure may have come from domesticated animals such as the civet. While some patients acquire the virus from animals or the environment, the majority appear to have contracted the illness from other people. Human-human transmission (aerosol or fecal-oral) is efficient. Environmental transmission (water, sewage) may also have played a role. Children had much less severe clinical course compared to adults. In 2003 the outbreak ceased and no new cases since 2004.

Which clinical entity is the most difficult to distinguish from osteomyelitis in a diabetic foot on any currently available medical imaging (XR, CT, MRI, US, and 3-phase bone scan)? A: abscess B: cellulitis C: fracture D: neuropathic osteopathy E: tumor

D neuropathic osteopathy, or Charcot's foot, represents characteristic destruction of the joints and bones of the foot in the setting of prolonged, poorly controlled diabetes mellitus. Characteristic chronic findings on PE include hammer-toes and rocker bottom feet. More acutely the foot is often red and warm, with bounding pulses followed by the initial joint deformities. This condition can therefore be difficult to ddx from cellulitis, osteomyelitis, and deep tissue infection, particularly in the presence of an ulcer. Unfortunately, all available imaging modalities can confuse osteomyelitis with the acute and chronic changes of neuropathic osteopathy as well. To add to the challenge, the presence of a Charcot's foot likely predisposes to ulceration and subsequent contiguous osteomyelitis. Generally, MRI is adequate for ddx cellulitis and tumor; US for abscesses; MRi and XR for fractures. Appropriate samples for microbiologic studies should be obtained to determine the presence and cause of chronic osteomyelitis

19 YO F from Guatemala presents to your office for routine screening PE. At 4 YO she was diagnosed with acute rheumatic fever. She does not recall the specifics of her illness and remembers only that she was required to be on bed rest for 6 months. She has remained on PCN V orally at a dose of 250 mg bid since that time. She asks if she can safely d/c this med. She has had only one other flare of her disease, at age 8, when she stopped PCN at the time of her emigration to the US. She is currently working as a day care provider. Her PE is notable for normal PMI with a grade III/VI holosytolic murmur best heard at the apex which radiates to the axillae. What do you advise? A: ECHO should be done to determine the extent of the valvular damage before deciding if PCN should be d/c'd B: PCN ppx can be d/c'd because she has had no flares in 5 years C: She should change her dosing regimen to IM PCN G q/8 weeks D: She should continue on PCN indefinitely as she had a previous recurrence, has presumed rheumatic heart disease, and is working in a field with high occupational exposure to GAS E: She should replace PCN ppx with polyvalent Pneumococcal vaccine q/5 years

D recurrent episodes of rheumatic fever are MC in the first 5 years after initial diagnosis. PCN ppx is recommended for at least this period. After the 5 years, 2nd ppx is determined on an individual basis. Ongoing ppx is currently recommended for patients who have had recurrent disease, have rheumatic heart disease, or work in occupations that have a high risk for preexposure to GAS infection. PPX regimens are PCN V, po 250 mg bid, PCN G 1.2 M U IM q/4 weeks, and Sulfadiazine, 1 g po daily. Polyvalent pneumococcal vaccine has no cross reactivity with GAS.

38 YO F pigeon keeper who has no significant PMH, is taking no meds, has no allergies, and is HIV negative presents to the ED with a fever, HA, and mild nuchal rigidity. Neuro exam is normal. Heat CT is normal. LP is significant for an opening pressure of 20 cmH2O, wBC 15 (90% monocytes), protein of 0.5 g/l (50 mg/mL), glucose of 2.8 mmol/L (50 mg/dL), and positive Indian ink stain. What is the appropriate therapy? A: Amphotericin B for 2 weeks B: Amphotericin B with flucytosine for 2 weeks C: Amphotericin B for 2 weeks, then po Fluconazole 400 mg daily D: Amphotericin B for 10 weeks, then po Fluconazole, 400 mg daily for 6-12 months E: Ceftriaxone and Vancomycin for 2 weeks

D the goal of therapy for cryptococcal meningoencephalitis in an HIV negative patient is cure of the fungal infection, not simply control of the ssx. Thus, the course of Amphotericin is recommended to be 10 weeks with negative CSF cx, a decreasing CSF cryptococcal antigen titer, and a normal CSF glucose value. Once this is completed and clinical response has been achieved, therapy is followed by Fluconazole to complete 6-12 months. The 2-week Amphotericin course can be used in clinically responding HIV positive patients, who then will require 8 weeks of Fluconazole, 400 mg daily, followed by lifelong suppressive therapy with Fluconazole, 200 mg daily. Flucytosine has been used to accelerate a negative cx response, but its use exposes the patient to potentially severe toxicities. Ceftriaxone and Vancomycin are the recommended treatments for bacterial meningitis in an immunocompetent patient < 50 YO

17 YO F with PMH of intermittent asthma presents to clinic in Feb with several days of cough, fever, malaise, and myalgia. She notes her ssx started 3 days ago with a HA and fatigue, and that several students and teachers at her high school have been recently diagnosed with the "flu". She did not receive a flu shot this year. Which of the following meds is the best option for her? A: ASA and cough suppressant with codeine B: Oseltamivir, 75 mg po bid for 5 days C: Rimantadine, 100 mg po bid for 1 week D: Symptom-based therapy with OTC agents E: Zanamivir, 10 mg inhaled bid for 5 days

D the majority of influenza infections are clinically mild and self-limited. Treatment with OTC suppressants and analgesics such as Acetaminophen is often adequate. Patients <18 YO are at risk of developing Reye's syndrome if exposed to salicylates such as ASA. The neuraminidase inhibitors oseltamivir and zanamivir have activity against Influenza A and B. They can be used within 2 days of ssx onset and have been shown to reduce the duration of ssx by a day or two. This patient has had ssx for >48 hours, therefore neither drug is likely to be effective. The patient's hx of asthma is an addition CONTRA to zanamivir, as this drug can ppt bronchospasm. The M2 inhibitors, amantadine and rimantadine, have activity against Influenza A only. However, in 2005 > 90% of A/H3N2 viral isolates demonstrated resistance to amantadine, and these drugs are no longer used for Flu A.

23 YO previously healthy F letter carrier works in a suburb in which the presence of rabid foxes and skunks has been documented. She is bitten by a bat, which then flies away. Initial exam reveals a clean break in the skin in the RU forearm. She has no hx of receiving treatment for rabies and is unsure about vaccination against tetanus. The physician should: A: clean the wound with 20% soap soln B: clean with 20% soap solo, and administer tetanus toxoid C: clean with 20% soap soln, give tetanus toxoid, and give human rabies immunoglobulin IM D: clean with 20% soap soln, give tetanus toxoid, give human rabies immunoglobulin IM, and give human diploid cell vaccine E: clean with 20% soap soln, and give human diploid cell vaccine

D the patient has been bitten by a member of a species known to carry rabies in an area in which rabies is endemic. Based on the animal vector and the facts that the skin was broken and that saliva possibly containing rabies virus was present, post-exposure rabies ppx should be given. If an animal involved in an unprovoked bite can be captured, it should be killed humanely and the head should be sent immediately to an appropriate lab for exam by fluorescent antibody staining for viral antigen. If a healthy dog or cat bites a person in an edemic area, the animal should be captured, confined, and observed for 10 days. If the animal remains healthy, the bite is unlikely to have transmitted rabies. Post-exposure ppx includes vigorous cleaning of the wound with 20% soap solution to remove any virus particles that may be present. Tetanus toxoid and ABX should be given. Passive immunization with anti-rabies antiserum in the form of human rabies immunoglobulin (rather than corresponding equine antiserum because of the risk of serum sickness) is indicated at a dose of 10 U/kg into the wound and 10 U/kg IM into the gluteal region. Second, one should actively immunize with an anti-rabies vaccine (either diploid cell vaccine or rabies vaccine absorbed in 5x 1-mL doses given IM, preferably in the deltoid or anterior lateral thigh). The five doses are given over 28 days period. The administration of either passive or active immunization w/o the other modality results in a higher failure rate than does the combination therapy

Which of the following has resulted in significant decrease in the incidence of trichinellosis in the US? A: adequate therapy which allows for eradication of infection in index cases before person-person spread can occur B: earlier dx due to a new cx assay C: federal laws limiting the import of foreign cattle D: laws prohibiting the feeding of uncooked garbage to pigs E: requirements for hand washing by commercial kitchen staff who handle raw meat

D there are roughly 12 cases of trichinellosis reported each year in the US. Since most are asymptomatic, this may be an underestimate. Heavy infections cause enteritis, periorbital edema, myositis, and infrequently, death. This infection is caused by ingestion of Trichinella cysts, coccus when infected meat from pigs or other carnivorous animals is eaten. Laws that prevent feeding pigs undercooked garbage have been an important PUBH measure in reducing infections. Person-person transmission has not been described. The majority of infections are mild and resolve spontaneously.

25 YO M is seen in the ED for ssx of fever and ABD swelling, early satiety, and weight loss. His ssx began abruptly 2 weeks ago. He was previously healthy and is taking no meds. He denies illicit drug use and recently immigrated from Bangladesh. On PE, T 102.2F, HR 120, with normal BP and RR. The remainder of the exam is notable for cachexia and a distended abdomen with a massively enlarged spleen. The spleen is tender and soft. The liver is not palpable. Mild peripheral adenopathy is present. Which of the following statements is correct regarding the patient with presumed kala azar leishmaniasis. A: he probably has normal cell counts on PBS B: L donovani is not endemic to Bangladesh C: Leishmania-specific cell-mediated immunity probably is present D: splenic aspiration offers the highest diagnostic yield E: treatment can be delayed until the dx is confirmed

D this patient comes from an area endemic for visceral leishmaniasis that includes Bangladesh, India, Nepal, Sudan, and Brazil. Although many spp can cause cutaneous or mucosal disease, L donovani complex generally is a/w visceral leishmaniasis. The organism is transmitted via the bite of a sandfly in the majority of cases. Although many patients remain asymptomatic, malnourished persons are at particular risk for progression to symptomatic disease or kalazar, the life threatening form. The presentation of this disease generally includes fever, cachexia, and splenomegaly. Hepatomegaly is rare compared with other tropical diseases a/w organomegaly, such as malaria, miliary TB, and schistosomiasis. Pancytopenia is a/w severe disease, as are hypergammaglobulinemia and hypoalbuminemia. Although active investigation is under way to determine a means of diagnosing leishmania by molecular techniques, the current standard remains demonstration of the organism on a stained slide or in tissue cx of a biopsy. Splenic aspiration has the highest yield, with reported sens 98%. In light of high mortality a/w the disease, treatment should not be delayed. The mainstay of therapy is a pentavalent antimonial, but newer therapies including Amphotericin and pentamidine can be indicated in certain situations. In this case it is prudent to r/o malaria with thick/thin smear. Rarely, the intracellular amastigote forms of Leishmania can be seen on PBS.

48 YO F presents to her physician with a 2-day hx of fever, arthralgia, diarrhea, and HA. She recently returned from an ecotour in tropical sub-Saharan Africa, where she went swimming in inland rivers. Notable findings on PE include T 38.7 (101.7F); 2-cm tender mobile LNs in the axilla, cervical, and femoral regions; and a palpable spleen. Her WBC is 15 with 50% eosinophils. She should receive treatment with which med? A: chloroquine B: mebedazole C: metronidazole D: praziquantel E: thiabendazole

D this patient has Katayama fever caused by infection with Schistosoma mansion. Approximately 4-8 weeks after exposure to the parasite, it migrates through the portal and systemic circulation. This phase of the illness may be asymptomatic but in some cases evokes a HSN reaction and a serum-sickness type illness. Eosinophilia is usual. Since there is not a large enteric burden of parasites during this phase, stool studies may not be positive and serology may be helpful, particularly in patients from non endemic areas. Praziquantel is the treatment of choice because Katayama fever may progress to include neurologic cx. Chloroquine is used for malaria; Mebendazole for ascariasis, hookworm, trichinosis, and visceral larval migrans; metronidazole for amebiasis, giardiasis, and trichomoniasis; and thiabendazole for strongyloides

42 YO M with poorly controlled DM (A1c 13.3%) presents with thigh pain and fever over several weeks. PE reveals erythema and warmth over the thigh with notable woody, non pitting edema. There are no cutaneous ulcers. CT of the thigh reveals several abscesses located between the ms fibers of the thigh. Orthopedics is consulted to drain and cx the abscesses. Which of the following is the most likely pathogen? A: C perfringens B: GAS C: Polymicrobial flora D: S aureus E: Strep milleri

D this patient has infectious pyomyositis, a disease of the tropics and of immunocompromised hosts such as patients with poorly controlled DM or AIDS. The pathogen is usually S aureus. Management includes aggressive debridement, abx, and attempts to reverse the patient's immunocompromised status. C perfringens may cause gas gangrene, particularly in the devitalized tissues. Strep infections may cause cellulitis or an aggressive fasciitis, but the presence of abscesses in a patient with poorly controlled DM makes Staph more likely. Polymicrobial infections are common in diabetic ulcers, but in this case the imaging and PE show intramuscular abscesses

35 YO M is seen 6 months after a cadaveric renal allograph. The patient has been on azithroprine and prednisone since that procedure. He has felt poorly for the past week with fever 38.6 (101.5F), anorexia, and a productive cough of thick sputum. CXR reveals a LLL (5-cm) nodule with central cavitation. Exam of the sputum reveals long, crooked, branching, beaded gram-positive filaments. The most appropriate initial therapy would include the administration of which ABX? A: Ceftazidime B: Erythromycin C: PCN D: Sulfisoxazole E: Tobramycin

D this patient is chronically immunosuppressed from his anti rejection prophylactic regimen, which includes both glucocorticoids and azathioprine. However, the finding of a cavitary lesion on CXR considerably narrows the possibilities and increases the likelihood of Nocardiasis. The other clinical findings, including productive sputum, fever, and constitutional ssx, are also quite common in patients with pulmonary nocardiosis. The gram stain, which demonstrates filamentous branching gram positive organisms, is characteristic. Most species are AF if a weak acid is used. These organisms can also be visualized by silver staining. They grow slowly in cx, and the lab must be alerted to the possibility of their presence on submitted specimens. Once the diagnosis is made, sulfonamides are the drug of choice. Sulfadiazine or Sulfisoxazole from 6-8 g/day in 4 divided doses generally is administered, but doses up to 12 g/d have been given. The combination of SMX and TMP has also been used, as have the po alternative Minocycline and Ampicillin and IV Amikacin. There is little experience with the newer B-lactam abx, including the 3rd gen cephalosporins and impinem. Erythromycin alone is not effective, although it has been given successfully along with Ampicillin. In addition to the appropriate abx, the possibility of disseminated nocardiosis must be considered; sites include brain, skin, kidney, bone, and ms.

26 YO F presents to ED with fever, chills, backache, and malaise. She reports a habit of active IVDA; last use was 2 days ago. Her VS show T 38.4C, HR 106, RR 22, BP 114/61, and O2 sat 98% on 2L per NC. A CXR and subsequent Chest CT demonstrates multiple peripheral nodular infiltrates with cavitation. Blood cx are sent to the labs and are pending. At this point in the workup, how many minor criteria are met from the Duke criteria for the clinical diagnosis of IE? A: 0 B: 1 C: 2 D: 3 E: 5

D this patient meets 3/5 minor criteria: - fever >= 38C - predisposition of IVDA - evidence of vascular phenomena (septic pulmonary infiltration on CXR) two remaining criteria are not met or not described above: - immunologic phenomena (glomerulonephritis, Osler's nodes, Roth's spots, RF) - microbiologic phenomena (positive blood cx that do not meet major criteria or positive serology for org likely to cause endocarditis) Major Criteria: 1. + blood cx 2. evidence of endocardial involvement (echo or new valvular regurgitation) with 3 minor and no major, this patient should be considered to possibly have IE, and the remainder of the workup should be pursued to rule in or r/o this dx. A complete PE with particular attention to the joints, skin, and CVS and an ECHO would be crucial for next step.

A previously healthy 17 YO F presents in early October with profound fatigue and malaise, as well as fevers, HA, nuchal rigidity, diffuse arthralgia, and a rash. She lives in a small town in MA and spent her summer as a camp counselor at a local day camp. She participated in daily hikes in the woods, but did not travel outside of the area during the course of the summer. PE reveals a well-developed young woman who appears extremely fatigued but not in extremis. Her T 37.4C, HR 86, BP 96/54, RR 12. PE documents clear breath sounds, no cardiac rub or murmur, normal BS, a NT abdomen, no organomegaly, and no evidence of synovitis. Several cutaneous lesions are noted on the LE, b/l axillae, R thigh and L-groin. All of these are possible cx of her current disease state, except: A: bell's palsy B: large joint oligoarticular arthritis C: meningitis D: progressive dementia E: 3rd degree heart block What is the appropriate therapy?

D this patients rash is characteristic erythema migrans lesion and is diagnostic for Lyme disease. In the US, it is due to borrelia burgdorferi. Partial central clearing, a bright red border, and a target center are suggestive of this lesion. The fact that multiple lesions exist implies disseminated infection, rather than a primary tick bite inoculation where only one lesion is present. Potential cx of 2nd Lyme include migratory arthritis, meningitis, CN neuritis, mononeuritis multiplex, myelitis, varying degrees of AV block, and less commonly, myopericarditis, splenomegaly, and hepatitis. 3rd degree or persistent Lyme disease is a/w oligoarticular arthritis of large joints and subtle encephalopathy but no frank dementia. Borrelia garinii infection is seen only in Europe and can cause a more profound encephalomyelitis Acute Lyme disease involving the skin and/or joints is treated with po Doxycycline 100 mg po BID, unless the patient is pregnant or <9 YO. Amoxicillin and Macrolides (Azithromycin) are less effective. Ceftriaxone is indicated for acute disease in the CNS (meningitis, facial n palsy, encephalopathy, radiculoneuritis) or a 3rd degree heart block. It may also be used for treatment of patients with arthritis who do not respond to po therapy. First gen cephalosporins are not active against Borrellia. While the rash may look like cellulitis due to staphylococcus or streptococcus, there is no proven efficacy of Vancomycin for Lyme disease.

In a patient who has undergone a traumatic splenectomy, what test may be ordered to establish lack of splenic function? A: CT abdomen B: Neutrophil migration studies C: peripheral blood flow cytometry D: peripheral blood smear

D traumatic splenectomy does not necessarily result in loss of splenic function as functioning splenules can implant within the peritoneum in the setting of trauma. A PBS showing Howell-Jolly bodies implies loss of splenic function. This is important as asplenic patients are at considerably higher risk of overwhelming sepsis and warrant vaccination against encapsulated pathogens.

19 YO M presents to urgent care with urethral d/c. He reports (3) new female sexual partners over the past 2 months. What should his management be? A: NAT for gonorrhea and chlamydia and RTC in 2 days B: Cefpodoxime, 400 mg po x1, and Azithromycin 1 g po x1 for the patient and his partners C: NAT for gonorrhea and chlamydia plus Cefpodoxime, 400 mg po x1 and Azithromycin, 1 g po x1 for the patient D: NAT for gonorrhea and chlamydia, Cefpodoxime, 400 mg po x1, and Azithromycin 1 g po x1 for patient and his partners E: NAT for gonorrhea and chlamydia, Cefpodoxime 400 mg po x 1, Azithromycin 1 g po x1, and Flagyl 2 g po x1 for patient and partners

D urethritis in men causes dysuria with or w/o d/c, usually w/o frequency. The MC causes in men include gonorrhea, chlamydia, mycoplasma, ureaplasma, trichomonal, HSV, and possibly Adenovirus. Until recently, chlamydia accounted for 30-40% of cases; however, this number may be falling. Recent studies suggest that Mycoplasma is a common cause of non-chlamydial cases. Currently the dx of urethritis is men includes specific tests only for gonorrhea and chlamydia. Tenets of d/c treatment include providing treatment for the MC causes with the assumption that patient may be LTFU. Therefore, prompt empirical treatment for G/C should be given on day of presentation. Azithromycin will also be effective for Mycoplasma. If pus can be milked from the urethra, cx should be sent for ddx and to allow for contacting the health dept, as both are reportable diseases. Urine NAT are an acceptable sub in the absence of pus. It is also critical to provide empiric treatment for at-risk sexual contacts. If ssx do not respond to initial treatment, patients should be reevaluated for compliance, preexposure, or Trichomoniasis.

3 YO M is brought by his parents to clinic. They state that he has experienced fevers, anorexia, weight loss, and, most recently, has started wheezing at night. He has been completely healthy until these ssx started 2 months ago. The family has traveled through Europe several months previously and reported no unusual exposures or exotic foods. They have a puppy at home. On exam, he is ill-appearing and is noted to have HSM. Labs show panic value of 82% eosinophilia. Total WBC are elevated. A CBC is repeated to r/o lab error and eosinophilia is 78%. Which is the most likely organism? A: cysticercus B: giardiasis C: staph lugdunensis D: toxocariasis E: trichinellosis

D visceral larva migrans, caused in this case by the canine roundworm Toxocara Canis, MC affects young children who are exposed to canine stool. Toxocara eggs are ingested and begin their life cycle in the SI. They migrate to many tissues in the body. Particularly characteristic of this illness are HSM and profound eosinophilia, at times close to 90% of total WBC count. Staphylococci will not typically cause eosinophilia. Trichinellosis, caused by ingestion of meat from carnivorous animals that has been infected with trichinella cysts, does not cause HSM and is uncommon w/o eating a suspicious meal. Giardiasis has profound diarrhea and ABD pain w/o systemic features of eosinophilia. Cysticercosis typically causes myalgia and can spread to the brain, where it is often asymptomatic but can lead to seizures

In a patient with bacterial endocarditis, which of the following ECHO lesions is most likely to lead to embolization? A: 5-mm mitral valve vegetation B: 5-mm tricuspid valve vegetation C: 11-mm aortic valve vegetation D: 11-mm mitral valve vegetation E: 11-mm tricuspid valve vegetation

D while any valvular vegetation can embolism, vegetations located on the Mitral valve and vegetations >10 mm are at greatest risk. Hematogenously seeded infection from an embolized vegetation may involve any organ, but particularly those with the highest blood flow. They are seen in up to 50% of patients with endocarditis. Triscuspid lesions will lead to septic pulmonary emboli, common in IVDA. Mitral and aortic lesions can lead to embolic infections in the skin, spleen, kidneys, meninges, and skeletal system. A dreaded neurologic cx is mycotic aneurysm, focal dilations of arteries at points in the arterial wall that have been weakened by infection in the vasa vasorum or septic emboli, leading to hemorrhage

19 YO M college student presents to ED with crampy ABD pain and watery diarrhea that has worsened over 3 days. He recently returned from a volunteer trip o Mexico. He has no PMH and felt well throughout the trip. Stool exam shows small cysts containing four nuclei, and stool antigen immunoassay is positive for Giardia. Which of the following is an effective treatment regimen? A: Albendazole B: Clindamycin C: Giardisis is self limited and requires no ABX D: Metronidazole E: Paromomycin F: Tinidazole

D or F Giardiasis is diagnosed by detection of parasite antigens, cysts, or trophozoites in feces. There is no reliable serum test for this disease. As a wide variety of pathogens are responsible for diarrheal illness, some degree of diagnostic testing beyond the H&P is required for ddx. Colonoscopy does not have a role in diagnosing Giardia. Giardiasis can persist in symptomatic patient and should be treated. Cure rates with 5 days of po Metronidazole TID are >90%. A single po dose of Tinidazole is reportedly at least as effective as Metronidazole. Paromomycin, an po poorly absorbed amino glycoside, can be used for symptomatic patients during pregnancy, but its efficacy for eradicating infection is not known. Clindamycin and Albendazole do not have a role in treatment. Refractory disease can be treated with a longer duration of Metronidazole

Abacavir is a nucleoside transcription inhibitor that carries which AE unique for HIV treatment? A: fanconi's anemia B: granulocytopenia C: lactic acidosis D: liopatrophy E: severe HSN reaction

E Abacavir use is a/w potentially severe HSN reaction in about 5% of patients. There is likely a genetic component, with HLA-B*5701 being a significant RF for HSN. SSX, which usually occur within 2 weeks of therapy but can take >6 weeks to emerge, include fever, maculopapular rash, fatigue, malaise, GIT ssx, and/or SOB. Once a diagnosis is suspected, the drug should be stopped and never given again because rechallenge can be fatal. For this, both the diagnosis and patient education once the diagnosis is made must be performed thoroughly and carefully. It is important to note that two available combination pills contain Abacavir (epzicom, trizivir), so patients must know to avoid these as well. Fanconi's anemia is rare and a/w tenofovir. Zidovudine causes anemia and sometimes granulocytopenia. Stavudine and other NRTIs are a/w lipodystrophy of the face and legs

The MC clinical presentation of infection with Babesia microti is A: acute hepatitis B: chronic meningitis C: generalized LAD D: overwhelming hemolysis, high-output CHF, respiratory failure, and DIC E: self-limited flulike illness

E Babesiosis due to B microti is transmitted to humans by the hard-bodied tick. The infection occurs mostly in coastal southern New England and Eastern Long Island; however, cases have been reported in NY, PA, WI, and MN. Most cases are probably never recognized because the MC (25% of adults) are either asymptomatic or indistinguishable from other acute febrile illnesses which are self-limiting. After an incubation of 1-6 weeks after a tick bite, patients may develop fever (intermittent or sustained as high as 40C), malaise, shaking chills, myalgia, and arthralgia. Severe infection is MC in asplenic patients and the elderly and in immunocompromised patients (HIV, malignancy, immunosuppressive meds). Patients co-infected with Borrelia burgdorferi (Lyme) are also at severe risk of infection. It is notable for an enormous parasitemia that can reach as high as 85% and is a/w hemolysis, high output CHF, and renal and respiratory failure

19 YO M presents with 4 days of watery diarrhea, nausea/vomiting, and low grade fever. He recalls no unusual meals, sick contacts, or travel. He is hydrated with IVF, given antiemetics, and d/c home after feeing better. 3 days later, two of three blood cx are positive for C perfringens. He is called at home and says he feels fine and is back to work. What should your next instruction to patient be? A: return for IV PCN B: return for IV PCN plus ECHO C: return for IV PCN plus Colonoscopy D: return for surveillance blood cx E: reassurance

E Clostridia are gram-positive, spore-forming obligate anaerobes that reside normally in the GIT. Several spp can cause severe disease. C perfringens, which is the 2nd MC clostridial spp to normally colonize the GIT, is a/w food poisoning, gas gangrene, and myonecrosis. C septicum is seen often in conjunction with GIT tumors. C sordellii is a/w septic abortions. All can cause a fulminant overwhelming bacteremia, but this condition is rare. The fact that this patient is well several days after his acute complaints r/o this fulminant course. A more common scenario is transient, self-limited bacteremia due to transient gut translocation during an episode of gastroenteritis. There is no need to treat when this occurs, and no further workup is needed. Clostridium spp sepsis rarely causes endocarditis because overwhelming DIC and death occur so rapidly. Screening for GIT tumors is warranted when C septicum is cx from blood or deep wound

Which of these statements regarding the currently licensed HPV vaccine (Gardasil) is true? A: it does not protect against warts B: it is an inactivated live virus C: it is targeted towards all oncogenic strains of HPV, but is only 70% effective at decreasing infection in an individual D: once sexually active, women will derive little protective benefit from the vaccine E: vaccinees should continue to receive standard Pap smear testing

E HPV 6 and 11 cause 90% of anogenital warts. HPV 16 and 18 cause 70% of cervical cancers. Both vaccines consist of virus-like particles w/o any viral nucleic acid, therefore are not active. Both provide nearly 100% protection against two common ongenic strains of HPV (16, 18) but neglect to cover the other strains that cause up to 30% of cervical cancer. Because they do not protect against all oncogenic strains, it is recommended that the Pap screening of women continue according to prior schedules. The vaccine should be given to girls and young women between the ages of 9-26 YO provided they do not have evidence of infection with both 16 and 18 already.

Which of the following statements regarding ppx of human respiratory syncytial virus (H-RSV) infection in children is true? A: all children who are admitted to the hospital more than 2x/year should be vaccinated against H-RSV B: barrier precautions remain the only effective means of ppx C: children should be vaccinated at birth D: inactivated, whole-virus vaccine should be considered in children <2 YO E: RSV immune globulin should be given monthly to children <2 YO who were born prematurely

E HRSV, previously known as RSV, is an RNA paramyxovirus. It is the major respiratory pathogen in children, the foremost cause of lower respiratory illness in infants, and a cause of common cold-like symptoms. It is a common and important nosocomial pathogen. RSV Ig, aka Palivizumab, has been approved as a monthly injection for children <2 YO who have congenital heart disease or lung disease or who were born prematurely as a means of preventing RSV infection. It has not shown to be beneficial in HRSV PNA. Barrier precautions should be used, especially in locations where there are high transmission rates; however, with the advent of RSV Ig this is not the only means of prevention. An inactivated, whole-virus RSV vaccine trials found that patients receiving the vaccine appeared more likely to acquire RSV infection. An adequate vaccine has not been developed to date

45 YO F with known HIV and medical non adherence to therapy is admitted to the hospital with 2-3 weeks of increasing DOE and malaise. CXR shows b/l alveolar infiltrates and induced sputum is positive for PCP. Which of the following clinical conditions is an indication for admission and adjunct glucocorticoids? A: ARDS B: CD4 <100 C: No clinical improvement 5 days into therapy D: Pneumothorax E: Room air PaO2 <70

E PCP lung infection is known to worsen after initiation of treatment, likely due to lysis of the organism and immune response to its intracellular contents. It is thought that the adjunct administration of GC may reduce inflammation and subsequent lung injury in patients with moderate-severe PNA due to PCP. Adjunct administration of GC in patients is determined by RA PaO2 <70 or A-a gradient >35 mm Hg. GC should be given for a total duration of 3 weeks. Patients do not improve until may days into therapy and often initially worsen; steroids should be used as soon as hypoxemia develops rather than wait for lack of improvement. PTX and ARDS are common feared complications of PCP. If patients present with aRDS 2/2 PCP PNA, they would meet criterion for adjunct GC just due to the severe nature of disease

All of these are a/w increased risk of PID, except: A: bacterial vaginosis B: hx of salpingitis C: IUD D: recent sexual exposure to a man with urethritis E: ssx beginning on days 14-21 of the menstrual cycle

E PID refers to an ascending infection from the vagina/cervix to the endometrium and/or the Fallopian tubes. Infectious cx include peritonitis, perihepatitis, perisplenitis, or pelvic abscess. It is an important cause of infertility. Gn/Cl are the microbes most commonly implicated in PID. Mycoplasma, anaerobes, and facultative organisms have also been isolated in the peritoneal fluid or Fallopian tubes of women with PID. BV, a hx of salpingitis, the presence of IUD, or recent exposure to a male partner with urethritis are RF for the development of PID. PID is more likely to develop during or soon after menses, suggesting that the menstruation is a RF for ascending infection

Which of the following meds use as antimycobacterial drugs require dose reduction for patients with an estimated GFR < 30? A: INH B: Pyrazinamide C: Rifabutin D: Rifampin E: Streptomycin

E Streptomycin is an amino glycoside first-line supplemental agent for treatment of MTB, M marinarium, and M kansasii. It is only available IM or IV and is not commonly used in the US because of toxicity. AE occur in 10-20% of patients. Renal toxicity and ototoxicity are most common. Ototoxicity may involve hearing and vestibular function. Like other ahminoglycosides, it is eliminated almost exclusively by the kidney, so drug levels must be followed along with renal function. INH and Rifampin must be avoided in patients with severe hepatotoxicity. Pyrazinamide is also metabolized by the liver and should be used carefully in patients with liver disease.

Which of the following pathogens are cardiac transplant patients at unique risk for acquiring from the donor heart early after transplant when compared to other solid organ transplant patients? A: Cryptococcus neoformans B: CMV C: PCP D: S aureus E: Toxoplasma

E Toxoplasma commonly achieves latency in cysts during acute infection. Reactivation in the CNS in AIDS patients is well known. However, cysts also reside in the heart. Thus, transplanting a Toxo-positive heart into a negative recipient may cause reactivation in the months after transplant. Serology screening of cardiac donors and recipients for Toxo is important. To account for this, ppx dose of TMP-SMX, which is also effective ppx against PCP and Nocardia, is a standard after cardiac transplant. Cardiac transplant recipients, similar to all other solid organ transplant patients, are at risk of developing infections related to impaired cellular immunity, particularly >1 month to 1 year post-transplant. Wound infections or mediastinitis from skin organisms may cx the early transplant (<1 month) period

A patient comes to clinic and describes progressive ms weakness over several weeks. He has also experienced nausea, vomiting, and diarrhea. One month ago he had been completely healthy and describes a bear hunting trip in Alaska, where they ate some of the game they killed. Soon after he returned, his GIT ssx began, followed by ms weakness in his jaw and neck that has now spread to his arms and lower back. Exam confirms decreased ms strength in the UE and neck. He also has slowed extra ocular movements. Labs show panic values for elevated eosinophils and serum CPK. Which of the following organisms is most likely? A: Campylobacter B: CMV C: Giardia D: Taenia solium E: Trichinella

E Trichinellosis occurs when infected meat products are eaten, most frequently pork. The organism may also be transmitted through ingestion of meat from dogs, horses, and bears. Recent outbreaks in the US and Canada have been related to the consumption of wild game, particularly bear meat. During the first week of infection, diarrhea, n/v are prominent features. As the parasites migrate from the GIT, fever, and eosinophilia are often present. Larvae encyst after 2-3 weeks in ms tissue, leading to myositis and weakness. Myocarditis and maculopapular rash are less common features. Giardia and Campy are organisms that are frequently acquired by drinking contaminated water; neither will produce this pattern. While both cause GIT ssx (and Campy causes fever), neither cause eosinophilia or myositis. Taenia, or pork tapeworm, shares a similar pathogenesis but does not cause myositis. CMV has varied presentations, but none that lead to this.

Which of the following clinical features can be used to r/o malaria in favor of another tropical febrile illness in a returning traveler? A: diarrhea B: lack of paroxysmal nature of the fever C: lack of splenomegaly D: severe myalgia and retroorbital HA E: none of these

E all febrile travelers returning from, or immigrants arriving from, Plasmodium falciparum endemic regions should be assumed to have infection with this most severe form of malaria until proven otherwise. P falciparum is the MC infection in returning travelers, may be fatal if not treated, and none of the listed features have sufficient predictive value to r/o malaria. Splenomegaly is a common feature of malaria but is not present in all cases. Vivax and ovale often have paroxysmal fevers but not enough so as to carry significant predictive value; this feature is rarely present in falciparum infected persons. Severe myalgia and rtetroorbital HA are appropriately prompt interest in a dx of dengue fever, but these ssx are common in malaria as well. ABD pain is a very common feature of malaria, and diarrhea can also occur.

During the last 1990s, there was a resurgence of all of the following bacterial STIs among homosexual men, except: A: chlamydia B: gonorrhea C: lymphogranuloma venereum D: syphilis E: all of these had a resurgence

E all of these have had an impressive resurgence among homosexual men in North America and Europe since 1996. This is in part due to a phenomenon of serosorting, an imperfect process among many homosexual men who seek sexual partners of the same HIV aerostats. This method allows for no protection against other STIs, and in fact may allow for concentration of these among high-risk networks of men. HIV prevalence has unfortunately also increased among homosexual men. Lymphogranuloma venereum, an uncommon chlamydia infection that has virtually disappeared prior to AIDS era, has been reported as outbreaks in homosexual men

26 YO asthmatic continues to have coughing fits and SOB despite numerous steroid tapers and frequent use of albuterol over the past few months. Persistent infiltrates are seen on chest roentgenogram. A pulmonary consult suggests an evaluation for allergic bronchopulmonary aspergillosis. What is the test of choice? A: bronchoalveolar lavage with fungal cx B: galactomannan enzyme immunoassay C: high-res CT D: pulmonary function tests E: serum IgE

E allergic bronchopulmonary aspergillosis (ABPA) is not a true infection but rather a HSN immune response to colonizing Aspergillus spp. It occurs in ~1% of patients with asthma and in up to 15% of CF. Patients typically have wheezing that is difficult to control with usual agents, infiltrates on CXR due to mucus plugging of airways, a productive cough often with mucus casts, and bronchiectasis. Eosinophilia is common if GC have not been administered. The total IgE is of value if >1000 IU/mL in that it represents a significant allergic response and is very suggestive of ABPA. In the proper clinical context, a positive skin test forAspergillus antigen or detection of serum Aspergillus-specific IgG or IgE precipitating antibodies are supportive of the diagnosis. Galctomannan EIA is useful for invasive aspergillosis but has not been validated for ABPA. There is no need to try to cx an organism via BAL to make the diagnosis. Chest CT, which may reveal bronchiectasis, or PFT, which will reveal an obstructive defect, will not be diagnostic.

All of the following regarding HSV-2 infection are true, except: A: approximately 1 in 5 Americans harbors HSV-2 antibodies B: asymptomatic shedding of HSV-2 in the genital tract occurs nearly as frequently in those with no ssx as in those with ulcerative disease C: asymptomatic shedding of HSV-2 is a/w transmission of the virus D: HSV-2 seropositivity is an independent RF for HIV transmission E: seroprevalence rates of HSV-2 are lower in Africa than in the US

E antibodies to HSV-2 are not routinely detected until puberty, c/w typical sexual transmission of the virus. Serosurveys suggest 15-20% of American adults have HSV-2 infection. However, only 10% report a hx of genital lesions. Seroprevalence is similar or higher in Central America, South America, and Africa. Recent studies in African OB clinics have found seroprevalence rates as high as 70%. HSV-2 infection is felt to be so pervasive in the general population based on ease of transmission, both in symptomatic and asymptomatic states. Therefore, this STD is significantly more common in individuals who less frequently engage in high-risk behavior than other STDs. HSV-2 is an independent RF for HIV acquisition and transmission. HIV virion is shed from herpetic lesions, thus promoting transmission.

Which of the following ABX has the weakest association with the development of C diff? A: Ceftriaxone B: Ciprofloxacin C: Clindamycin D: Moxifloxacin E: Pip/Tazo

E clindamycin, ampicillin, and cephalosporins (including Ceftriaxone) were the first abx a/w C diff. More recently, broad-spectrum fluoroquinolone, including moxifloxacin and ciprofloxacin, have been a/w outbreaks of C diff, including outbreaks in some locations of a more virulent strain that has caused severe disease among elderly outpatients. For unclear reasons, B-lactams other than the later generation Cephalosporins appear to carry a lesser risk of disease. PCN/B-lactamase combo antibiotics appear to have lower risk of C diff. Cases have even been reported a/w Metronidazole and Vancomycin. Nevertheless, all patients initiating abx should be warned to seek care if they develop diarrhea that is severe or persists for more than a day, as all abx carry some risk of C diff

Deficits in complement membrane attack complex (C5-8) are a/w recurrent infections of what variety? A: pseudomonas B: catalase-positive bacteria C: strep pneumo D: salmonella spp E: neisseria meningitis

E deficiencies in complement predispose patients to a variety of infection. Most of these are congenital. Patients with SCD have acquired functional defects in the alternative complement system. They are at risk of infection from S pneumonia and Salmonella spp. Patients with liver disease, nephrotic syndrome, and SLE may have defects in C3. They are at particular risk for infections with S aureus, S pneumonia, Pseudomonas, and Proteus. Patients with congenital or acquired (usually SLE) def in the terminal complement cascade (C5-8) are at particular risk of infection from Neisseria spp, such as meningitides or gonorrhoeae

19 YO college student is brought to ED by friends from his dorm for confusion and AMS. They state that many colleagues have URTIs. He does not use ETOH or illicit drugs. His PE is notable for confusion, fever, and a rigid neck. CSF exam reveals a WBC 1800 with 98% PMNs, glucose 1.9 (35 mg/dL) and a protein 1.0 g/L (100 mg/dL). Which abx is most appropriate as initial therapy? A: ampicillin plus vancomycin B: ampicillin plus gentamicin C: cefazolin plus doxycycline D: cefotaxime plus doxycycline E: cefotaxime plus vancomycin In addition to these, which adjuvant therapy should be given to improve the chance of a favorable outcome?

E in a previously healthy student, particularly one in a dorm, Strep pneumoniae and Neisseria meningitides are the pathogens most likely to be causing CA-bacterial meningitis. As a result of increasing prevalence of PCN and Cephalosporin-resistant Streptococci, initial therapy should include a 3rd/4th gen Cephalosporin plus Vancomycin. DEX has been shown in children and adults to decrease meningeal irritation and unfavorable outcomes in acute bacterial meningitis. In a recent study of adults the effect on outcome was most notable in patients with S pneumonia infection. The first dose (10 mg IV) should be given 15-20 min before or with the first dose of ABX and is unlikely to be of benefit unless it is begun 6 hours after the initiation of abx. Ddx may decrease the penetration of vancomycin into the CSF.

19 YO F comes to your office after being bitten by a bat on the ear while camping in a primitive shelter. She is unable to produce a vaccination record. On PE, she is afebrile and appears well. There are two small puncture wounds on the pinna of the L-ear. What is an appropriate vaccination strategy in this context? A: IV-Ribavirin B: No vaccination C: Rabies immunoglobulins D: Rabies inactivated virus vaccine E: Rabies inactivated virus vaccine plus immunoglobulins

E in recent years, rabies virus has been most frequently transmitted by bats in the US. Usually a bite is noted, but not always. Therefore, patients who have expected, unmonitored (i.e., while they are asleep) close contact with bats should be told to seek medical attention and likely vaccination. A bite is a clear indication for the most effective strategy involving both active (inactivated virus) and passive (human rabies immunoglobulin) immune activation, unless the offending bat is captured and found to be rabies negative. The vaccination schedule for nonimmunes is intensive, with doses at 0, 3, 7, 14, and 28 days. While there has been at least one report of successful antiviral treatment of rabies, there is no indication for ppx antiviral therapy.

39 YO F received a liver transplant 2 years ago and is maintained on Prednisone, 5 mg, and cyclosporine A, 8 mg/kg per day. She has had (2) episodes of rejection since transplant, as well as an episode of CMV and Nocardia PNA. She intends on taking a 2-week gorilla-watching trip to Rwanda and seeks advice regarding traveling abroad. Which potential intervention is strictly CONTRA? A: malaria ppx B: meningococcal vaccine C: rabies vaccine D: typhoid purified polysaccharide vaccine E: yellow fever vaccine

E live attenuated vaccines are generally CONTRA as vaccines for immunocompromised hosts for fear of vaccine-induced disease. The MC example of this is smallpox vaccine resulting in disseminated vaccinia infection. However, Yellow fever vaccine is also a live vaccine. The other examples listed here are inactivated organisms (rabies, IM typhoid) or polysaccharides (meningococcal) and are therefore noninfectious. Oral typhoid vaccine is a live attenuated strain, so the IM form is preferable. Malaria ppx currently involves chemoprophylaxis rather than vaccination. While safe from an infection standpoint, potential interactions with cyclosporine should be monitored

36 YO M with hx of HTN presents c/o 3-year hx of constant fatigue, diffuse myalgia, and memory deficits. He also notes trouble with routine tasks at work. He was diagnosed with Lyme disease 4 years ago and was briefly admitted to a CCU for transient 3rd degree heart block. SSX at that time included fever, malaise, arthralgia, diffuse erythema migrans, and facial n palsy. He received Ceftriaxone, 2g/d for 28 days, and has complete resolution of SSX for several months but then developed his new constellation of problems that have gradually worsened over time. PE is wnl. Which is the best next step? A: B burgdoferi ELISA B: Ceftriaxone, 2 g daily x 1 month C: Doxycycline, 100 mg po daily for life D: Prednisone, 60 mg po daily E: Symptomatic treatment

E post-Lyme syndrome is an uncommon but debilitating syndrome similar to fibromyalgia and chronic fatigue syndrome and sometimes occurs after appropriate treatment for Lyme. Its etiology is not well understood, but it does not appear to be due to ongoing replication or presence of Borrelia at sites of patient discomfort. Clinical trials have compared long courses of IV-Ceftriaxone followed by po Doxy with placebo and found no differences in outcomes. Prendisone would likely be indicated only the presence of documented inflammatory arthritis, a condition that is not evident here. The patient has already received adequate treatment for Lyme, and a positive IgG serology would either indicate past infection or serofast state. In either case, the test is not indicated because it would not be helpful in the current situation and could be misleading

23 YO F is newly diagnosed with HSV-2 infection. What can you tell her that the chance of reactivation disease will be during the first year after infection? A: 5% B: 25% C: 50% D: 75% E: 90%

E primary genital herpes due to HSV-2 is characterized by fever, HA, malaise, inguinal LAD, and diffuse genital lesions of varying stage. The cervix and urethra are usually involved in women. While both HSV-2 and 1 can involve the genitals, the recurrence of 2 is much higher (90% in the first year) than with 1 (55% in the first year). The rate of reactivation for HSV-2 is very high. Acyclovir (or its congeners Valacyclovir and Famciclovir) is effective in shortening the duration of SSX and lesions in herpes. Chronic daily therapy can reduce the frequency of recurrences in those with frequent reactivation. Valacyclovir has been shown to reduce transmission between sexual partners.

Which of these factors is most important determinant of the rate of disease progression from HIV to clinical diagnosis of AIDS? A: age B: CD4 count 6 months after infection C: CMV IgG status D: HIV resistance panel at infection E: HIV viral load set point 6 months after initial infection

E rate of progression from HIV to AIDS is likely determined by many factors, but viral load set point, measured ~2-6 months after acute infection, is the most predictive factor. Most persons have a median time of 10 years, but rapid Progresso's with high viral loads can develop AIDS over 5 years whereas some "long term nonprogressors" with low viral load set points can be asymptomatic with HIV for decades. CD4 count is a measure of how close one is to developing AIDS rather than a measure of progression: it tends to be close to normal once viral load set point is achieved. Resistance of the virus is more of a determinant of response to specific therapy than disease progression in the absence of therapy. CMV serology is important as a predictor for CMV disease once a person reaches CD4 < 50

33 YO F is undergoing consolidation CTX for AML with cytarabine plus daunorubicin. She developed a fever 5 days prior which has persisted despite the addition of cefepime and vancomycin to her ppx abx regimen of norfloxacin, fluconazole, and acyclovir. Other than diaphoresis and chills during her periodic fevers, she remains largely asymptomatic except for a general sense of malaise and nausea a/w CTX, as well as oral pain due to mucositis. She remains neutropenic despite administration of heme growth factors. Blood, urine, and sputum cx all remain negative. What is the next step in her management? A: add Metronidazole B: add Tobramycin C: change fluconazole to caspofungin D: chest roentgenogram E: high res CT plus serum galactomannan enzyme assay

E the MC infection in a patient with prolonged neutropenia and fever once acute bacterial infections have been r/o is invasive Aspergillus. Thoracic CT and serum galactomannan testing are indicated because the initial stages of invasive disease are frequently not symptomatic. CT scan may show subtle new nodules or a halo sign (hemorrhagic infarct surrounding a nodule) that is suggestive, though not diagnostic, of this condition. Aspergillus antigen is detected by galactomannan release during growth of mold. FP are possible for both chest CT and antigen tests. However, over diagnosis is preferable to late or missed as this infection disseminates to the skin and brain and can be difficult to treat at this stage. As a result, many cases are treated presumptively with serial chest CTs and galactomannan levels, rather than awaiting definitive diagnosis by cx or consistent hematopathology. Galactomannan levels may be falsely elevated in the presence of B-lactam/B-lactamase combo abx such as Pip/Tazo. Chest roentgenogram is not sens for detecting early disease. Bronchoscopy with lavage is indicated for an attempt at definitive diagnosis only if abnormalities are detected on chest CT. There is no reason to suspect C diff (and hence need Metronidazole) in the absence of diarrhea. Similarly, in the absence of documented bacterial infection, there are no data to support the addition of an amino glycoside. There is no reason to suspect fluconazole-resistant yeast infection requiring caspofungin in the absence of detectable fungemia. While caspofungin has activity against Aspergillus, it is approved only for salvage therapy

All of the following patient characteristics are included in the calculation of PNA Patient Outcomes Research Team (PORT) score that is used in the evaluation of patients with CA-PNA, except: A: age B: co-existing illness C: labs D: XR findings E: smoking history

E the PORT score is a system used to classify CA-PNA from mild to severe. The score assigns points to 20 items a/w mortality, including age, nursing home residence, comorbid conditions (neoplasia, liver disease, CHF, CVA, renal disease), PE findings (mental status, RR, BP, T, HR), and labs/XR findings (pH, BUN, Na, glucose, HCT, PaO2, pleural effusion). The resulting scores are used to determine five classes with progressively increasing mortality. These classes correlate with mortality and have been used to derive suggested management of treatment (home v hospital). Smoking hx is a RF for PNA, but is not used in the prognostic scoring system.

Per-coital rate of HIV acquisition in a man who has unprotected sexual intercourse with an HIV-infected female partner is likely to increase under which of the following: A: acute HIV in the female partner B: female HSV-2 positive serostatus C: male nongonococcal urethritis at the time of intercourse D: uncircumcised male status E: all of the above

E the biologic determinants of HIV transmission and acquisition are complex and have been difficult to study. However, several RF are now known to increase the per-coital rate of HIV transmission, at least for heterosexual couples. In discordant couples there is a dose-dependent relationship between serum viral load and HIV transmission. In fact, in carefully done studies there was virtually no transmission between discordant couples when serum viral load was low (<400). It is likely that this is due to a fairly tight correlation between serum and genital viral load. A corollary is that during acute HIV or AIDS, the viral load, and therefore, transmissibility are high. There are strong clinical data from RCT that circumcised men are less likely to acquire HIV because the interior of the foreskin is replete with cellular targets for HIV. Nonulcerative STI cause mucosal breakdown which allows for greater acquisition of HIV. HSV-2 (not necessarily active ulcer) leads to increase in HIV genital shedding as well as HIV-1 target cell migration to the genital mucosa, making both transmission and acquisition of HIV higher in HSV-2 positive persons.

45 YO M from Western KY presents to ED in September c/o fever, HA, and ms pain. He recently had been on a camping trip with several friends during which they hunted for their food, including fish, squirrels, and rabbits. He did not recall any tick bites during the trip, but does recall having several mosquito bites. For the past week, he has had an ulceration on his R hand with redness and pain surrounding it. He also has noticed some pain and swelling near his elbow. None of his friends he camped with have been ill. His VS: BP 106/65, HR 116, RR 24, and T 38.7. O2 Sat is 93% on RA. He appears mildly tachypneic and flushed. His conjunctiva are not injected and his mucous membranes are dry. The chest exam reveals crackles in the R mid-lung field and L base. His HR is tachycardic but regular. There is a II/VI systolic ejection murmur heard best at the LLSB. His ABD exam is unremarkable. On the R hand, there is erythematous ulcer with a punched out center covered by black eschar. He has no cervical LAD, but there are markedly enlarged and tender LNs in the R-axillae and epitrochlear regions. The epitrochlear nodes has some fluctuance with palpation. CXR shows fluffy b/l alveolar infiltrates. Over the first 12 hours, he become progressively hypotensive and hypoxic, requiring intubation. What is the most appropriate therapy? A: Ampicillin, 2 g IV q6h B: Ceftriaxone, 1 g IV daily C: Cipro, 400 mg IV bid D: Doxycycline, 100 mg IV bid E: Gentamicin, 5 mg/kg bid

E the most likely organism in this patient is Francisella tularensis. Gentamicin is the DOC for the treatment of tularemia. Fluoroquinolone have shown in vitro activity against it and have successfully been used in a few cases of tularemia. Currently, however, it cannot be recommended as 1st line as data are limited in regards to its efficacy relative to gentamicin, but can be considered if an individual is unable to tolerate gentamicin. To date, there have been no clinical trials of fluoroquinolone to definitively demonstrate equivalency with gentamicin. 3rd gen cephalosporins have in vitro activity. However, use of Ceftriaxone in children with tularemia resulted in almost universal failure. Likewise, tetracycline and chloramphenicol also have limited usefulness with a higher relapse rate (up to 20%) when compared to gentamicin. F tularensis is a small, gram-negative, pleomorphic bacillus that is found both intra- and extracellularly. It is found in mud, water, and decaying animal carcasses, and ticks and wild rabbits are the source for most human infection in the SE US and Rocky Mtn states. In western states, tabard flies are the MC vectors. The organisms usually enter the skin through the bite of a tick or through an abrasion. On further questioning, the patient reported that during the camping trip he was primarily responsible for skinning the animals and preparing dinner. He did suffer a small cut on his R hadn't at the site where the ulceration is apparent. The MC clinical manifestations of F tularensis are ulceroglandular and glandular disease, accounting for 75-85% of cases. The ulcer appears at the site of entry of the bacteria and lasts for 1-3 weeks and may develop a black eschar at the base. The draining LNs become enlarged and fluctuant. They may drain spontaneously. In a small % of patients, the disease becomes systemically spread, as in this case, with PNA, fever, and sepsis syndrome. When this occurs, the MR approaches 30% if untreated. However, with appropriate abx the prognosis is very good. Dx requires a high clinical suspicion as demonstration of the organism is difficult. It rarely is seen on Grams stain because they stain weakly and are so small that they are difficult to distinguish from background. On polychromatically stained tissue, they may be seen both intra- and extracellularly, singly or in clumps. Moreover, it is a difficult organism to culture and requires cystein-glucose-blood agar. However, most labs do not attempt to cx the organism because of the risk of infection in lab workers, requiring biosafety level 2. Usually the dx is confirmed by agglutination testing with titers >1:160 confirming the dx

46 YO veterinary researcher who frequently operates on rats presents to the ED with jaundice and scant hemoptysis. She recalls having a fairly deep cut on her hand during an operation 14 days ago. She has had no recent travel or other animal exposures. Her illness started ~9 days ago with fever, chills, severe HA, intense myalgia, and nausea. She also noted b/l conjunctival injection. Thinking that she had influenza, she stayed home from work and started to feel better 5 days into the illness. However, within a day her ssx had returned with worsening HA, and soon after she developed jaundice. On initial exam, her T 38.6C, HR 105, BP 156/89, with O2 sat 92% on RA. She appears acutely ill and is both icteric and profoundly jaundice. Her liver is enlarged and tender, but there are no palpable masses and she has no splenomegaly. Labs show BUN 64, Cr 3.6, total bili 64.8 (direct 59.2), AST 84, Alt 103, ALP 384, WBC 11 (13% bands and 80% PMNs), HCT 33, PLT 142. UA shows 20 WBCs, 3+ protein, and granular casts. Coags wnl. LP reveals sterile pleocytosis. CT scan shows diffuse flamelike infiltrates c/w pulmonary hemorrhage. What is the diagnosis? A: acute interstitial PNA B: AML C: PAN D: rat bite fever (Streptobacillus moniliformis) E: Weil's syndrome (Leptospira interrogans)

E the patient has Weil's syndrome due to infection with Leptospira as evidence by flulike prodrome followed by profound hyperbilirubinemia with only minor hepatocellular dysfunction as well as renal failure. Rats are the most important reservoir. The organisms is excreted in the urine and can survive in water for months. Important sources of exposure include occupational, recreation in contaminated waters, and being homeless in contaminated living areas. Weil's represents the immune phase of the disease in its most severe form. A bleeding diathesis is common, as is pulmonary hemorrhage. The conjunctival suffusion during the initial spirochetemic phase is an important diagnostic clue. PCN G is appropriate therapy for severe leptospirosis, but its comparative efficacy is not yet proven in literature. AML would most likely cause more characteristic abnormalities in blood counts with this degree of illness. Acute interstitial pneumonitis (Haman-Rich syndrome) affects only the lung and would not be a/w severe increases in bilirubin. PAN rarely involves the lung and would not be expected to cause such a high bilirubin, even in the setting of hepatic ischemia. Rat-bite fever causes intermittent Fev era, polyarthritis, and a nonspecific rash.

62 YO M returns from vacation to AZ with fever, pleurisy, and NP cough. All of the following factors on Hx and labs favor a diagnosis of pulmonary Coccidioidomycosis rather than CA-PNA, except: A: eosinophilia B: erythema nodosum C: mediastinal LAD on Chest Roentgenogram D: positive Coccidioides complement fixation titer E: travel limited to Northern AZ (Grand Canyon area)

E there is no Coccidioides in Northern AZ (i.e., the Grand Canyon region). The organism can be cx'd from dry top soil in the high desert of Southern Arizona surrounding Phoenix and Tucson. Eosinophilia is a common finding in acute coccidioidomycosis and erythema nodosum is a common cutaneous feature. Mediastinal LAD is more commonly seen on XR for all acute PNA due to endemic mycoses, including Coccidioides, rather than due to bacterial PNA. A positive complement fixation test is one method to dx acute infection

28 YO F presents with fevers, HA, diaphoresis, and ABD pain 2 days after returning from an aid mission to the coast of Papua New Guinea. Several of her fellow aid workers developed malaria while abroad, and she stopped her doxycycline ppx due to a photosensitivity reaction 5 days prior. You send blood cx, routine labs, and a thick/thin smear to evaluate the source. Which of the following statement is accurate in reference to her diagnosis of malaria? A: a thick smear is performed to increase sens in comparison to a thin smear but can only be performed in centers with experienced lab personnel and has a longer processing time B: careful analysis of thin blood film allows for prognostication based on estimation of parasitemia and morphology of the RBCs C: in the absence of rapid diagnostic information, empirical treatment for malaria should be strongly considered D: Morphology on blood smear is the current criterion used to ddx the four spp of Plasmodium that infect humans E: all of the above are true

E thick and thin smears are a critical part of the evaluation of fever in a person with recent time spent in a Plasmodium-endemic region. Thick smears take a longer time to process but increase sens in the setting of low parasitemia. Thin smears are more likely to allow for precise morphologic evaluation to ddx between the 4 types and also allow for prognostic calculation of parasitemia. If clinical suspicion is high, repeat smears should be performed if initially negative. If personnel are not available to rapidly interpret a smear, empirical therapy should be strongly considered to ward off the most severe manifestations of P falciparum infection. Antibody-based diagnostic tests that are sens and spec for falciparum infection have been introduced. They will remain positive for weeks after infection and do not allow quantification of parasitemia

38 YO F presents to ED with severe ABD pain. She has no PMH or SURGhx. She recalls no recent hx of ABD discomfort, diarrhea, melena, BRBPR, nausea/vomiting prior to this acute episode. She ate ceviche (lime marinated raw fish) at a Peruvian restaurant 3 h prior to presentation. On exam, she is in terrible distress and has dry heaves. T 37.6C, HR 128, BP 174/92. Exam is notable for extremely tender ABD with guarding and rebound tenderness. BS are present and hyperactive. Rectal exam is wnl and guiac is negative. Pelvic exam wnl. WBC 6.7, HCT 42. CMP and lipase/amylase are all wnl. CT of the abdomen shows no abnormalities. What is the next step? A: CT angio of the abdomen B: Pelvic US C: PPI and observation D: RUQ US E: Upper endoscopy

E this patient's likely diagnosis is Anisakiasis. This is a nematode infection where humans are an accidental host. It occurs hours to days after ingesting eggs that previously settled into the ms of fish. The main RF for infection is eating raw fish. Presentation mimics an acute abdomen. Hx is critical as Upper Endoscopy is both diagnostic and curative. The implicated nematodes burrow into the mucosa of the stomach causing intense pain and must be manually removed by endoscope or, on rare occasion, surgery. There is no medical management.

During the first 2 weeks after solid organ transplant, which family of infection is MC? A: CMV and EBV reactivation B: humoral immunodeficiency associated infections (e.g., meningococcemia, invasive Strep pneumoniae) C: neutropenia-associated infection (e.g., Aspergillosis, candidemia) D: T-cell def associated infections (e.g., PCP PNA, Nocardiosis, Cryptococcosis) E: Typical hospital-acquired infections (e.g., central line infection, hospital-acquired PNA, UTI)

E ultimately, solid organ transplant patients are at highest risk of infection due to T cell immunodeficiency from anti rejection meds. As a result, they are also at risk for reactivation of many of the viruses from the Herpes family, most notably CMV, VZV, and EBV. However, immediately after transplant, these deficits have net yet developed in full. Neutropenia is not common in solid organ, as in BMT. In fact, patients are most at risk of typical hospital-acquired infections, including wounds, UTI, PNA, C diff, and line-associated infection. Therefore, a standard eval of a febrile patient in the first weeks after a solid organ transplant should include detailed PE, blood and urine cx, UA, CXR, C diff stool antigen/toxin, if warranted, in addition to the transplant-specific eval

All of the following infections a/w sexual activity correlate with increased acquisition of HIV in women, except: A: bacterial vaginosis B: Chlamydia C: gonorrhea D: HSV-2 E: Trichomoniasis F: all of the above are associated with increased acquisition

F HIV is the leading cause of death in some developing countries. Efforts to decrease transmission include screening and treatment of STI. All of these have been linked with higher acquisition of HIV, based on epidemiology and high biologic plausibility. Up to 50% of women of reproductive age in developing countries have BV. All of the bacterial infections are curable, and treatment can decrease the frequency of genital herpes recurrence. This highlights an additional reason that primary care doctors should screen for each of these infections in female patients with detailed history, GU and rectal exams, and evidence based screening for these infections based on age and risk category

In a patient with known HIV infection, all of the following are an AIDS-defining criterion except: A: active pulmonary TB B: CD4 count <200 C: Cryptococcal meningitis D: CMV retinitis E: disseminated MAC F: HZV involving more than one dermatome G: Kaposi sarcoma H: PCP PNA

F Patients with HIV and CD4 <200 are given the dx of AIDS as this is a rough threshold for an increased risk for several life-threatening opportunistic infections. Different opportunistic infections and HIV-related cx are seen most frequently below certain CD4 count thresholds: 1. disseminated CMV/MAC (<50) 2. cryptococcosis (<100) 3. pneumocystis (<200) These are all considered AIDS-defining cx. Certain dx, such as active TB and Kaposi are considered AIDS-defining even though they may present with higher CD4 counts because they are historically linked with higher MR. Multidermatomal zoster starts to occur with more frequency when CD4 <500 and is not AIDS defining. If multi dermatomal zoster occurs in a previously healthy person, it should raise suspicion for new HIV infection

One month after receiving a 14-day course of Omeprazole, Clarithromycin, and Amoxicillin for H pylori-associated gastric ulcer disease, a 44 YO F still has mild dyspepsia and pain s/p meals. What is the best next step? A: empirical long-term PPI B: endoscopy with bx to r/o gastric ADENO C: H pylori serology D: reassurance E: 2nd line treatment with Omeprazole, Bismuth subsalicylate, tetracycline, and Metronidazole F: urea breath test

F it is impossible to know whether the patient's continued dyspepsia is due to persistent H pylori as a result of treatment failure or to some other cause. A quick noninvasive test to look for the presence of H pylori is the urea breath test. This test can be done as outpatient and gives a rapid, accurate response. Patients should not have received any PPI or antimicrobials in the meantime. Stool antigen test is another good option if urea breath testing is not available. If the urea breath test is positive >1 month after completion of 1st line therapy, 2nd line therapy with the above may be indicated. If the test is negative, the remaining ssx are unlikely due to H pylori. Serology is useful only for dx infection initially, but it can remain positive and therefore misleading in those who have cleared infection. Endoscopy is a consideration to r/o ulcer or UGI malignancy but is generally preferred after two failed attempts to eradicate.

There is wide concern among many new members of the general pop regarding which of the following vaccines as a source of autism?

MMR There has been particular attention among the public and medical field regarding the MMR vaccine due to use of thimerosal, a mercury-containing preservative widely used in vaccines since the 30s. Several, large scale, carefully performed epidemiological studies in the US and N Europe have shown no association between the use of these vaccines and autism or other brain development disorders. Nevertheless, autism incidence is increasing and the proximity in age between development and administration of vaccine has allowed the debate to continue. CDC, WHO, and FDA have formally rejected any causal link between vaccines and autism, but as a cautionary measure, thimerosal has been d/c in pediatric vaccines in the US, most notably MMR. It is noteworthy that when vaccine coverage rates go below a certain threshold, outbreaks of vaccine preventable lesions occur. It is important to be able to communicate this complex information accurately to patients.

Which organisms is most likely to cause infection of a shunt implanted for the treatment of hydrocephalus?

S epidermidis Probably because of its ubiquity and ability to stick to foreign surfaces, S epidermidis is the MC cause of infections of central nervous system shunts as well as an important cause of infections on artificial heart valves and orthopedic protheses. Corynebacterium spp (diphtheroids) just like S epid colonize the skin. When they are isolated from cx of shunts, it is often difficult to be sure if they are the cause or simply contaminants. Leukocytosis in CSF, consistent isolation of the same organism, and the character of a patient's symptoms are all helpful in deciding whether treatment for infection is indicated

Candida albicans is isolated from the following patients. Rate the likelihood in order from greatest to least that the positive cx represents true infection rather than a contaminant or noninfectious colonization. Patient X: 63 YO M admitted to ICU with PNA who has recurrent fevers after 5 days of Levofloxacin for PNA. UA drawn from the Foley shows +LE, negative nitrite, 15 WBCs, 10 RBCs, and 10 epithelial cells. UCX grows Candida albicans Patient Y: 38 YO F on HD with low-grade fever and malaise. PBS grow Candida in 1 out of 3 sets of blood cx in the aerobic bottle only Patient Z: 68 YO M presents with 2-day hx of fever, productive cough, and malaise. Chest roentgenogram reveals a LLL infiltrate. A sputum stain shows many PMNs, few epithelial cells, moderate gram-positive cocci in chains, and yeast c/w Candida

Y > X > Z Isolation of yeast from the blood stream can virtually never be considered a contaminant. Presentation may be indolent with malaise only, or fulminant with overwhelming sepsis in the neutropenic host. All indwelling catheters need to be removed to ensure CL of the infection, and evaluation for endocarditis and endopthalmitis should be considered, particularly in patients with persistent positive cx or fever. Both of these cx of fungemia often entail surgical intervention for cure. A positive yeast cx in the urine is often difficult to interpret, particularly if on ABX or in the ICU. Most frequently, a positive cx for yeast represents contamination, even if the UA suggest bladder inflammation. An attractive option is to remove the Foley and recheck a cx. Antifungals are indicated if the patient appears ill, in the context of renal transplant where fungal balls can develop in the graft, and often in the neutropenic patient. Candida PNA is uncommon, even in immunocompromised host. A positive yeast cx of the sputum is usually representative of commensal oral flora and should not be managed as infection as in this case where acute bacterial PNA is likely.


Related study sets

The Essential 427: AP Psychology Review

View Set

Completing the Application, Underwriting, and Delivering the Policies 14%

View Set

Psychology Chapter Quizzes 13, 14, 15

View Set

English Grammar: The colon, semicolon, and more

View Set